File:  [Local Repository] / db / baza / tyumar09.txt
Revision 1.15: download - view: text, annotated - select for diffs - revision graph
Tue Jun 5 10:39:42 2018 UTC (5 years, 11 months ago) by rubashkin
Branches: MAIN
CVS tags: HEAD
*** empty log message ***

Чемпионат:
Тюменский марафон - 2009

URL:
/znatoki/boris/reports/200905Tyumen.html

Дата:
02-May-2009

Редактор:
Евгений Ярков (Тюмень)

Тур:
1 тур

Вопрос 1:
(pic: 20090818.jpg)
   Внимание, мы начинаем марафон - игру для скучающих и неугомонных,
которую немногие из вас смогут доиграть до конца.
   Перед вами изображение предмета, функциональное назначение которого
неизвестно. Одни считали, что это черный ящик утонувшего в болоте
самолета, другие - что это крышка скворечника, третьи полагали, что это
ящик Пандоры. Еще одно предположение содержит слово, обозначающее
название вымышленного золотого города. Напишите это слово.

Ответ:
Джуманджи.

Комментарий:
Джуманджи - название игры и города в этой игре. В игре есть
предостережение, что если игра начнется, то закончится она не раньше,
чем один из игроков достигнет золотого города Джуманджи. Завершив игру,
дети Питер и Джуди утопили ее в реке. А наше Джуманджи еще только
начинается.

Источник:
   1. http://community.livejournal.com/hrenovina/704936.html
   2. http://books.google.ru/books?id=HDi2AwYRJZAC&dq=jumanji+txt&printsec=frontcover&source=bl&ot=sycYBaHVJbe&sigJ54eAg3BgCuVhIz7f_nrRjwfLGc&hl=ru&ei=PgjsScq5NMKL_Qa5xInVAw&sa=X&oi=book_result&ct=result&resnum=4#PPT11,M1

Автор:
Евгений Ярков (Тюмень)

Вопрос 2:
Алишер Навои писал, что ЕЙ голову сжигает страсть, а ноги держит медь
оков. Мы же надеемся, что ОНА первая и последняя. Назовите ЕЕ.

Ответ:
Свеча.

Комментарий:
Второй вопрос - и сразу свеча. :-) Надеемся, что дальше свечек не будет.

Источник:
А. Навои. Газели.

Автор:
Евгений Ярков (Тюмень)

Вопрос 3:
Сравнивая советскую литературу с НИМ, писатель Григорий Свирский
сетовал, что на виду сияющие, гладкие, отшлифованные цензурой
произведения, в то время как острый, порой бесформенный, самиздат скрыт
от глаз читателей. Назовите ЕГО словом немецкого происхождения.

Ответ:
Айсберг.

Комментарий:
Свирский сравнивал советскую литературу с айсбергом, основная часть
которого не видна. От немецкого Eisberg, "ледяная гора".

Источник:
http://lib.ru/NEWPROZA/SWIRSKIJ/svirsky1.txt

Автор:
Евгений Ярков (Тюмень)

Вопрос 4:
В 1920 году в Витебске можно было увидеть плакат, на котором ПЕРВЫЙ
сравнивался с мировой революцией, а второй - с мировой экономией. В
названиях ПЕРВОГО и ВТОРОГО есть одно и то же слово. Напишите это слово.

Ответ:
Квадрат.

Комментарий:
Этот плакат включал в себя логотип листка Утвердителей Нового искусства
- общества, основанного Казимиром Малевичем. Под логотипом было
написано: "Носите черный квадрат как знак мировой экономии. Чертите в
ваших мастерских красный квадрат как знак мировой революции искусств".

Источник:
http://www.ec-dejavu.net/b/Black_square.html

Автор:
Евгений Ярков (Тюмень)

Вопрос 5:
После интронизации Патриарх Московский и всея Руси Кирилл практически
заменил бывшую администрацию Российской православной церкви на своих
сподвижников. Заголовок статьи об этом в газете "Коммерсант" представлял
собой фразеологизм из трех слов, в котором буквы "н" были заменены на
буквы "р". Воспроизведите этот заголовок.

Ответ:
Клир клиром вышибают.

Комментарий:
Клиром называют церковное сословие, а исходная идиома - "Клин клином
вышибают".

Источник:
http://www.kommersant.ru/doc.aspx?DocsID1147712

Автор:
Евгений Ярков (Тюмень)

Вопрос 6:
Древнеримский сестерций весил две с половиной единицы веса и официально
обозначался как Libra-Libra-Semis [либра-либра-семис]. По одной из
версий, именно это обозначение легло в основу символа, который впервые
использовал бухгалтер Оливер Поллок. Изобразите этот символ.

Ответ:
$.

Зачет:
[Изображение символа доллара с одной или двумя вертикальными чертами,
пересекающими букву S.]

Комментарий:
По этой версии, Libra-Libra-Semis сокращалось до llS, эти буквы Оливер
Поллок наложил друг на друга и получил нечто вроде современного символа
доллара.

Источник:
   1. http://www.gif.ru/themes/dates-of-the-week/dollar/view_print/
   2. http://ru.wikipedia.org/wiki/Символ_доллара

Автор:
Евгений Ярков (Тюмень)

Вопрос 7:
Персонаж произведения английского писателя опечален тем, что два
джентльмена тУзят друг друга. Назовите это произведение.

Ответ:
"Клуб самоубийц".

Зачет:
"Новые сказки тысячи и одной ночи".

Комментарий:
В произведении Стивенсона "Клуб самоубийц", если джентльмен вытягивает
трефового туза, он назначается исполнителем казни того джентльмена,
который вытягивает туза пикового.

Источник:
http://lib.ru/STIVENSON/stiven02.txt

Автор:
Евгений Ярков (Тюмень)

Вопрос 8:
По другой версии, изобретая символ доллара, Оливер Поллок обратил
внимание на испанский герб, на котором ОНИ обвиты лентой с надписью "nec
plus ultra" [нек плюс ультра], что означает "дальше некуда". Назовите ИХ
двумя словами.

Ответ:
Геркулесовы столбы.

Комментарий:
На испанском гербе изображены геркулесовы столбы, которые считались
краем земли.

Источник:
   1. http://www.gif.ru/themes/dates-of-the-week/dollar/view_print/
   2. http://www.bibliotekar.ru/encSlov/5/74.htm

Автор:
Евгений Ярков (Тюмень)

Вопрос 9:
ЕГО судьба, по мнению персонажа Владимира Хлумова, определена
конструктором ИКСА и состоит лишь в том, чтобы каждые несколько минут
умирать и возрождаться. В другом произведении ОН покидает ИКС и начинает
ходить в гости к разным людям, а когда те рассказывают о его визитах, им
мало кто верит. Мы не спрашиваем, кто ОН. Назовите ИКС.

Ответ:
Светофор.

Комментарий:
ОН - это зеленый человечек из светофора.

Источник:
   1. http://lib.ru/ZHURNAL/prelest.txt
   2. http://www.proza.ru/2003/05/26-134

Автор:
Евгений Ярков (Тюмень)

Вопрос 10:
В произведении Вячеслава Шишкова ОНИ сопровождают масленичные гулянья,
при этом малыши клянчат "Четвертьблина", середнячки просят "Полблина", а
самый главный требует "Блин". Полагают, что ОНИ появились благодаря
святому Павлинию, которому глас божий явился в виде полевых цветов.
Назовите ИХ.

Ответ:
Колокола.

Источник:
   1. http://lib.ru/HIST/SHISHKOW/shishkov_reka.txt
   2. http://www.valday.com/museum_bells

Автор:
Евгений Ярков (Тюмень)

Вопрос 11:
Один писатель отмечал, что ненависть к знакам препинания у него
появилась тогда, когда одно его произведение вышло в печать пЕристым
из-за шести страниц сплошных точек. Назовите это произведение.

Ответ:
"Облако в штанах".

Комментарий:
А писатель - В.В. Маяковский, не любивший ни точек, ни запятых.

Источник:
В. Маяковский. Избранное.

Автор:
Евгений Ярков (Тюмень)

Вопрос 12:
Один "язычник" назвал ЭТО ЗАНЯТИЕ бесплатной эпиляцией. А кто, согласно
фольклору, исчез в результате ЭТОГО ЗАНЯТИЯ?

Ответ:
Снегурочка.

Комментарий:
ЭТО ЗАНЯТИЕ - прыжки через костер.

Источник:
   1. http://vzhdanov.livejournal.com/271020.html
   2. http://www.amik.ru/Jokes/gid1294/pg7.html
   3. http://glagol.kneparhia.ru/snegurochka-nedobraya-boginya-ili-roman/

Автор:
Евгений Ярков (Тюмень)

Вопрос 13:
В произведении Виктора Мбо герой детского произведения живет в шикарном
доме на плаццо дель Марко, ездит на черном "Ламборджини" и вообще
считается настоящим римским аристократом. И всем очевидно, что некое
слово обозначает вовсе не помещение, а связь с итальянской мафией.
Назовите это слово.

Ответ:
Каморка.

Комментарий:
Герой детского произведения - Папа Карло. А каморку Папы Карло персонажи
произведения связывают с названием итальянской мафии - kamorra.

Источник:
В. Мбо. Педагогическое полено.

Автор:
Евгений Ярков (Тюмень)

Вопрос 14:
В произведении греческого писателя Лукиана ЕМУ выставляют довольно
приличный счет за якорь, ремень для весел, иглу для паруса и другие
предметы, а ОН говорит, что рассчитается, когда будет чума. Назовите
ЕГО.

Ответ:
Харон.

Комментарий:
Харону полагался обол за перевоз умершего. Выставили же ему счет на
несколько десятков драхм, и этой суммы у него не оказалось. Но как
только будет чума, умершие люди пополнят его финансы.

Источник:
http://lib.ru/POEEAST/LUKIAN/lukian1_4.txt

Автор:
Евгений Ярков (Тюмень)

Вопрос 15:
В советские времена розничная ИХ стоимость составляла десятую часть
копейки. В одном фантастическом рассказе ОНИ стали причиной смерти из-за
чрезмерных усилий. Назовите ИХ двумя словами.

Ответ:
Шесть спичек.

Комментарий:
В СССР коробок стоил одну копейку, при этом в коробке должно было быть
60 спичек. В рассказе Стругацких "Шесть спичек" инженер Комлин поднимал
силой мысли различные предметы, однако умер, поднимая шесть спичек.

Источник:
   1. http://www.jans.ru/answer12-259.html
   2. А. и Б. Стругацкие. Шесть спичек.

Автор:
Евгений Ярков (Тюмень)

Вопрос 16:
Заголовок статьи об итогах выступления малозаметного клуба российской
футбольной премьер-лиги лишь одной буквой отличался от вопроса из
произведения российского революционного писателя. Напишите этот
заголовок.

Ответ:
"А был ли Нальчик?".

Комментарий:
Речь в статье шла о нальчикском "Спартаке". Известная фраза - "А был ли
мальчик?" - взята из произведения Максима Горького "Жизнь Клима
Самгина".

Источник:
   1. http://www.sports.ru/blog/newsweek/4284743.html
   2. http://www.bibliotekar.ru/encSlov/a/2.htm

Автор:
Алексей Усанов (Киров)

Вопрос 17:
В статье "Эти непослушные слова" Евгений Гладилин пишет: "Нормальный
человек на вопрос "Что дает солнце?" ответит: "Свет", в то время как
составители кроссвордов ждут другой ответ". Скажите, что, по мнению
Гладилина, так наводят.

Ответ:
Тень на плетень.

Комментарий:
Составители кроссвордов, по мнению Гладилина, ждут слово "Тень".
Кроссворд похож на плетень. "Тень на плетень" - расхожая фраза.

Источник:
http://www.rojdenierus.ru/doc/mifi/neposlushnie-slova.shtml

Автор:
Евгений Ярков (Тюмень)

Вопрос 18:
В произведении Анатолия Мариенгофа фигурирует девушка с ровным, белым
лицом и накрашенными губами. При этом в качестве сравнения фигурирует
ОН. А по мнению Андрея Бажина, ОН символизирует большую любовь. Назовите
ЕГО.

Ответ:
Туз червей.

Комментарий:
Белое лицо и красный рот напоминают туза червей. В гадании туз червей
символизирует любовь.

Источник:
   1. http://lib.ru/RUSSLIT/MARIENGOF/cynix.txt
   2. http://magia-taro.ucoz.ru/news/2008-05-12-29

Автор:
Евгений Ярков (Тюмень)

Вопрос 19:
В произведении Гарольда МАзура свидетель со стороны обвинения дает
лживые показания, оправдывающие преступника. В этом произведении
упоминается автоматический кольт и еще одно оружие. Назовите это оружие.

Ответ:
Бумеранг.

Комментарий:
Оружие обвинение оказалось направленным против себя самого.

Источник:
Г. Мазур. Бумеранг.

Автор:
Евгений Ярков (Тюмень)

Вопрос 20:
Женское имя Алла принято производить от греческого слова, означающего
"Другая". Владимир Перцель рассуждая о странных основаниях канонизации
святой Аллы, упоминает некоего офицера. Назовите этого офицера.

Ответ:
Подпоручик Киже.

Зачет:
Поручик Киже; Киже; Киж.

Комментарий:
Киже (или Киж) - персонаж исторического анекдота времен царствования
императора Павла I, появившийся в документах из-за ошибки писаря,
который ошибся и вместо слов "прапорщики ж [такие-то] - в подпоручики"
написал "прапорщик Киж". Похожим на этот анекдот случаем В. Перцель
считает канонизацию святой Аллы.

Источник:
   1. http://ru.wikipedia.org/wiki/Поручик_Киже
   2. http://ru.wikipedia.org/wiki/Алла

Автор:
Евгений Ярков (Тюмень)

Вопрос 21:
Об одном своем современнике Федор Сологуб писал так: "Стихи его, такие
капризные, легкие, сверкающие и звенящие, льются потому, что переполнен
ИКС". Назовите ИКС двумя словами.

Ответ:
Громокипящий кубок.

Комментарий:
"Громокипящий кубок" - название первого сборника Игоря Северянина.

Источник:
http://history.yar.ru/vestnik/novye_Issledovaniy/29_8/

Автор:
Евгений Ярков (Тюмень)

Вопрос 22:
В произведении Дмитрия Нагишкина шаман загадывает такую загадку: "Белые
люди рубят, красный человек возит". В каком фразеологизме упоминаются
понятия, являющиеся ответом на эту загадку?

Ответ:
Держать язык за зубами.

Зачет:
Любой фразеологизм с упоминанием языка и зубов.

Комментарий:
Белые люди - зубы, красный человек - язык.

Источник:
Д. Нагишкин. Золотое кольцо.

Автор:
Евгений Ярков (Тюмень)

Вопрос 23:
Злые языки утверждали, что ОН попал в компанию "великих" только потому,
что Джона Борглума привлекала возможность продемонстрировать свои
таланты и похвастаться трудновыполнимым пенсне. Назовите ЕГО.

Ответ:
[Теодор] Рузвельт.

Комментарий:
Поговаривали, что Джон Борглум, создатель монумента "Четырех
президентов", включил Теодора Рузвельта только для того, чтобы
похвастаться каменным изваянием пенсне.

Источник:
http://www.vestnik.com/issues/2002/0415/koi/kiseleva.htm

Автор:
Евгений Ярков (Тюмень)

Вопрос 24:
В первом куплете пародии на известную песню упоминается полочка и ручка
для весла. А во втором куплете - лесенка и постройка. Назовите эту
постройку.

Ответ:
Сарай.

Комментарий:
Это пародия на песенку "В лесу родилась елочка". Так, первый куплет
звучит следующим образом: "В лесу родилась елочка, в лесу она росла, для
книжек выйдет полочка и ручка для весла". Со словом "песенка" во втором
куплете рифмуется лесенка, а с "бай-бай" - сарай.

Источник:
http://www.hohmodrom.ru/project.php?prid=11636

Автор:
Евгений Ярков (Тюмень)

Вопрос 25:
В одном сообщении, описывающем событие 2001 года, говорится, что еще
никто так, как русские, не глушил рыбу! Завершающий этап этого события
часто связывают с местом, имеющим аббревиатуру ККК. Расшифруйте эту
аббревиатуру.

Ответ:
Кладбище космических кораблей.

Комментарий:
Речь идет о потоплении ОКС "Мир". Кладбищем космических кораблей
называют участок Земли недалеко от Фиджи, куда падают остатки
космических аппаратов.

Источник:
   1. http://www.eku.ru/aforism/aforism2.html
   2. http://ru.wikipedia.org/wiki/Кладбище_космических_кораблей

Автор:
Евгений Ярков (Тюмень)

Вопрос 26:
В одном памфлете предлагается количество неких острых предметов
увеличить до сотни, чтобы ОНА превратилась в дикобраза. Мы не
спрашиваем, что это за предметы. Назовите ЕЕ.

Ответ:
Революция.

Комментарий:
В политическом памфлете Аркадия Аверченко "Дюжина ножей в спину
революции" количество ножей предлагалось увеличить до сотни, чтобы ее
превратить в дикобраза.

Источник:
http://az.lib.ru/a/awerchenko_a_t/text_0140.shtml

Автор:
Евгений Ярков (Тюмень)

Вопрос 27:
Футбольный комментатор, говоря о том, что смуглого защитника Рио
Фердинанда трудно сдвинуть с места, назвал его ИМ. Кто назвал так же
свой роман, опубликованный в середине XX века?

Ответ:
[Эрих Мария] Ремарк.

Источник:
   1. Трансляция матча "Манчестер Юнайтед" - "Тоттенхэм", 7ТВ,
01.03.2009 г.
   2. http://ru.wikipedia.org/wiki/Чёрный_обелиск_(роман)
   3. http://ru.wikipedia.org/wiki/Рио_Фердинанд

Автор:
Дмитрий Кукулин (Тюмень)

Вопрос 28:
В произведении Анатолия Мариенгофа описан страшный танец с ИКСОМ, в
котором узкое и розовое тело ИКСА извивается и одна американка ломает
ИКСУ хребет и сдавливает беспокойными пальцами горло. Надо полагать, что
ИКС отомстил американке в 1927 году. Назовите ИКС.

Ответ:
Шарф.

Комментарий:
Американская танцовщица - это Айседора Дункан, описан танец с шарфом.
Дункан погибла задушенная собственным шарфом, намотавшимся на колесо.

Источник:
   1. http://lib.ru/RUSSLIT/MARIENGOF/roman.txt
   2. http://ru.wikipedia.org/wiki/Дункан,_Айседора

Автор:
Евгений Ярков (Тюмень)

Вопрос 29:
В древнегреческой мифологии персонификацию голода звали Эфоном. Как
свидетельствует грек Ферекид, имя Эфон носило также существо, которое
Иосиф Бродский считал метафорой алкоголя. А к кому являлось это
существо?

Ответ:
К Прометею.

Комментарий:
Орла, каждый день клюющего печень Прометея, звали Эфоном.

Источник:
   1. http://www.astromyth.tau-site.ru/Subjects/Prometeus.htm
   2. http://ru.wikipedia.org/wiki/Эфон

Автор:
Евгений Ярков (Тюмень)

Вопрос 30:
(pic: 20090819.jpg)
   На этом изображении мы скрыли от вас семибуквенное слово. Напишите
это слово.

Ответ:
Перекур.

Комментарий:
У нас первый перекур.

Источник:
http://2opena.livejournal.com/54726.html

Автор:
Евгений Ярков (Тюмень)

Тур:
2 тур

Вопрос 1:
Александр Сергеевич Пушкин в поэме "Медный всадник" характеризует
Петербург как "юный град, полночных стран краса и диво". Прилагательное
"полночный" в этой цитате никак не связано со временем суток, а
обозначает лишь указание на ИКС. В одной энциклопедической статье
говорится, что ИКС последнюю свою войну вел в Британии, где он победил
каледонцев и восстановил Адрианов вал. Какое слово мы заменили на "ИКС"?

Ответ:
Север.

Комментарий:
Эпитет "полуночный" Пушкин употребляет вместо слова "северный" (сравните
полночь на часах и стрелку на розе ветров). Септимий Север, один из
римских императоров, восстановил Адрианов вал.

Источник:
   1. http://la-dy-ashley.livejournal.com/159826.html
   2. http://ru.wikipedia.org/wiki/Септимий_Север

Автор:
Евгений Ярков (Тюмень)

Вопрос 2:
Японского моряка Кодаи весьма приветливо принимали в России времен
Екатерины II и даже отдавали ему предпочтение перед многими европейскими
титулованными особами. Происходило это потому, что в те времена в мире
было всего семь ИХ. Одна из НИХ находится между "надеждой" и
"возвращением". А что она делает?

Ответ:
Наносит ответный удар.

Комментарий:
В те времена в мире было всего семь империй, и часто даже к
простолюдинам империй относились лучше, чем к важным персонам из
"обычных" королевств. В гексалогии "Звездные войны" фильм "Империя
наносит ответный удар" является продолжением фильма "Новая надежда" и
предшествует фильму "Возвращение джедая".

Источник:
   1. http://simonff.livejournal.com/395735.html
   2. http://sciuro.livejournal.com/221143.html
   3. http://ru.wikipedia.org/wiki/Звёздные_войны

Автор:
Евгений Ярков (Тюмень)

Вопрос 3:
Разъясняя потерянный смысл известной фразы, Сергей Верийкин пишет, что
ОНА сравнивалась с обезболивающим средством потому, что была призвана
создать видимость преодоления человеком гнета природы и общества.
Назовите ЕЕ.

Ответ:
Религия.

Комментарий:
Речь идет о фразе Маркса "Религия - опиум народа". В наши дни многие эту
фразу понимают неправильно.

Источник:
http://www.atheism.ru/library/verey_1.phtml

Автор:
Евгений Ярков (Тюмень)

Вопрос 4:
В фантастическом рассказе Сандры Сайкс фигурирует детская игрушка
"Цифертон", которая с помощью цветовых и звуковых комбинаций вводит в
состояние транса, после чего происходит нечто вроде телепортации. Какой
немецкий город упоминается в этом рассказе?

Ответ:
Гаммельн.

Зачет:
Хамельн.

Комментарий:
Цифертон, словно Гаммельнский крысолов, гипнотизирует детей и уводит их
от родителей.

Источник:
С. Сайкс. Цифертон.

Автор:
Евгений Ярков (Тюмень)

Вопрос 5:
Известно, что она появилась в период царствования Алексея Михайловича
Романова. Однако существующей информации недостаточно для того, чтобы
определить ЕЕ высоту. Так, она могла равняться как трем, так и пяти
метрам. Назовите ЕЕ двумя словами.

Ответ:
Коломенская верста.

Комментарий:
"Коломна" - вопрос, в котором не так просто выбрать ответ из нескольких
равнозначных. Идиома "Коломенская верста", означающая "верзилу", берет
свое начало со времен царя Алексея Михайловича, который велел расставить
вдоль дороги от Калужской заставы Москвы до летнего дворца в селе
Коломенском столбы на расстоянии 700 саженей друг от друга. Высота
каждого из них была около двух саженей, с орлами наверху. Поскольку
сажени были разные, никто не мог точно назвать высоту версты.
   z-checkdb: Перенесение свойств столбов на свойство версты (а именно
приписывание версте высоты) ничем не обосновано (Антон Губанов).

Источник:
   1. http://www.nkj.ru/archive/articles/10340/
   2. http://ru.wikipedia.org/wiki/Сажень

Автор:
Евгений Ярков (Тюмень)

Вопрос 6:
(pic: 20090820.jpg)
   Перед вами монумент с могилы Владимира Высоцкого. Андрей Вознесенский
пишет, что, смотря прямо на Высоцкого, можно увидеть два ИКСА. ИКС
получил свое название от латинского слова "облако". Назовите ИКС.

Ответ:
Нимб.

Комментарий:
Гитара Высоцкого кажется двойным нимбом.

Источник:
http://www.vsvysotsky.ru/post32023221/

Автор:
Евгений Ярков (Тюмень)

Вопрос 7:
Многие советские имена имели свои расшифровки. Например, имя Даздраподик
означало "Да здравствует победа "Динамо-Киев"", а имя Перкосрак -
"Первая космическая ракета". В расшифровке еще одного подобного имени
упоминаются только Комсомол, Коминтерн и Коммунизм, а само имя совпадает
с названием одежды. Назовите это имя.

Ответ:
Трико.

Комментарий:
"Комсомол, Коминтерн и Коммунизм" сокращалось до Трико.

Источник:
   1. http://ru.wikipedia.org/wiki/Имена_советского_происхождения
   2. http://dic.academic.ru/dic.nsf/ushakov/1058562/

Автор:
Евгений Ярков (Тюмень)

Вопрос 8:
На сайте Знаменской церкви говорится, что ОНО является врагом блуда,
исполняет очи слезами и прогоняет мечтания. ОНО, как правило,
заканчивается первым часом. Автор вопроса полагает, что участникам
марафона тоже вскоре предстоит ОНО. Назовите ЕГО двумя словами.

Ответ:
Всенощное бдение.

Зачет:
По смыслу.

Комментарий:
Всенощное бдение - как правило, служба, продолжающаяся с ночи и до утра.

Источник:
   1. http://www.znamenie.org/content/view/226/86/
   2. http://www.magister.msk.ru/library/bible/zb/zb266.htm

Автор:
Евгений Ярков (Тюмень)

Вопрос 9:
Военный теоретик Николай Николаевич Головин писал, что после смерти
этого человека в апреле 1918 года белое движение уже не могло
рассчитывать на то, чтобы ощутить вкус победы. Назовите имя и фамилию
этого человека.

Ответ:
Лавр Корнилов.

Комментарий:
Какой может быть вкус без Лавра.

Источник:
   1. Н.Н. Головин. Российская контр-революция в 1917-1918 гг. - Таллин,
1937.
   2. http://ru.wikipedia.org/wiki/Корнилов,_Лавр_Георгиевич

Автор:
Евгений Ярков (Тюмень)

Вопрос 10:
Заголовок статьи в газете "Время новостей" об исключении Анастасии
Волочковой из списка кандидатов на пост мэра города Сочи совпадает с
названием российской музыкальной группы. Воспроизведите этот заголовок.

Ответ:
"Танцы минус".

Комментарий:
Волочкова - балерина. Название рок-группы Вячеслава Петкуна - "Танцы
минус".

Источник:
http://www.vremya.ru/2009/57/4/226517.html

Автор:
Евгений Ярков (Тюмень)

Вопрос 11:
Алишер Навои, воспевая круглолицую девушку, сравнивал уста девушки с
НЕЙ. ЕЕ отсутствие в проектной документации, как правило, объясняется
нецелевым использованием ластика. Назовите ЕЕ тремя словами.

Ответ:
Точка от циркуля.

Зачет:
По смыслу.

Комментарий:
Уста, как центр круга лица, Навои сравнил с точкой от циркуля. Ножку
циркуля при черчении иногда втыкают в ластик, чтобы не оставлять след.

Источник:
   1. А. Навои. Газели.
   2. http://www.hobbyhandmade.com/instrumentyi/priemyi-i-metodyi-rabotyi/rezka-orgstekla.html

Автор:
Евгений Ярков (Тюмень)

Вопрос 12:
В одной миниатюре известный иллюзионист бьет ногой зрителя. Название
этой миниатюры на слух не отличается от имени сказочного персонажа.
Напишите название этой миниатюры.

Ответ:
"Пинок Кио".

Комментарий:
Иллюзионист - Кио, а персонаж сказки Карло Коллоди - Пиноккио.

Источник:
http://kvn-dvgu.narod.ru/litera.html

Автор:
Евгений Ярков (Тюмень)

Вопрос 13:
В одной из строк стихотворения Олега Ладыженского говорится, что враг
зарезан и расколот, при этом чуть ранее упоминаются ОНИ. Историк Буркин
утверждает, что ОНИ сливаются в греческую букву омега, означающую конец.
Назовите ИХ.

Ответ:
Серп и молот.

Комментарий:
Буква омега похожа на символы серпа и молота. Собственно, "серп и молот"
соотносятся с фразой "зарезан и расколот".

Источник:
   1. http://lib.ru/OLDI/poez.txt
   2. http://www.rusidea.org/?a25102505

Автор:
Евгений Ярков (Тюмень)

Вопрос 14:
Заглавный персонаж не устаревшего и по сей день романа, отказавшись от
идеи соблазнить простушку Гетти Мертон, утверждает, что стремление
порисоваться иногда толкает на поступки благороднее нас самих. Назовите
этого персонажа.

Ответ:
Дориан Грей.

Комментарий:
В романе Оскара Уайльда "Портрет Дориана Грея" герой не стареет, а
стареет его портрет. Герой думает, что благородные действия сделают уже
ставший мерзким портрет немного приятнее.

Источник:
http://lib.ru/WILDE/doriangray.txt

Автор:
Евгений Ярков (Тюмень)

Вопрос 15:
В Нонсенском толковом словаре синонимом этому слову является слово
"приблизительный". А в толковом словаре Ефремовой этим словом
обозначается чиновник, следящий за соблюдением закона на небольшой
территории города. Напишите это слово.

Ответ:
Околоточный.

Комментарий:
Околоточный надзиратель ведал околотком, небольшой территорией.

Источник:
   1. http://www.dictionary.nonsence.de/inh/14.html
   2. http://dic.academic.ru/dic.nsf/efremova/202489/

Автор:
Евгений Ярков (Тюмень)

Вопрос 16:
На одном произведении искусства можно обнаружить такую запись: "Я гулял
по дороге с двумя товарищами. Над сине-черным фьордом и городом лежали
кровь и языки пламени. Мои друзья продолжали гулять, а я остался позади,
трепеща от страха, и я услышал бесконечный [пропуск], пронзающий
природу". Какое слово мы в этом вопросе пропустили?

Ответ:
Крик.

Комментарий:
Фьорды могли подсказать вам, что это норвежский художник Эдвард Мунк,
автор картины "Крик".

Источник:
Н.А. Ионина. 100 великих картин.

Автор:
Евгений Ярков (Тюмень)

Вопрос 17:
Внимание, в вопросе словами "ДОМЕН" и "ОМЕН" заменены другие два слова.
   В 2007 году в результате сильного шторма в акваторию Керченского
залива из разорванного стихией танкера "Волгонефть-139" попало несколько
тонн мазута. Для борьбы с надвигающейся экологической катастрофой
предполагалось построить перекрытие между островом Тузла и косой Тузла,
иначе Черному и Азовскому морю был бы нанесен непоправимый ущерб. Статья
об этом в газете "Время новостей" называлась "ДОМЕН или ОМЕН". Какие
слова мы заменили на "ДОМЕН" и "ОМЕН"?

Ответ:
Дамба, амба.

Комментарий:
Нужна насыпь (дамба), иначе наступит конец морям (амба).

Источник:
http://www.vremya.ru/2007/208/46/191953.html

Автор:
Евгений Ярков (Тюмень)

Вопрос 18:
В этимологии многих современных слов скрыт ироничный смысл. Напишите
короткое слово, которым, по мнению историка Гладилина, называли
бритоголового прислужника при передвижном иудейском храме.

Ответ:
Скин.

Зачет:
Скинхед.

Комментарий:
Передвижной храм называется скиния, соответственно скин - прислужник
такого храма. Вот и получается, что "неонацисты" прислуживали иудеям.

Источник:
http://www.rojdenierus.ru/doc/mifi/neposlushnie-bukvi.shtml

Автор:
Евгений Ярков (Тюмень)

Вопрос 19:
В произведении греческого писателя Лукиана Гермес говорит ЕМУ, что топор
не будет хорошей повивальной бабкой. Назовите ЕГО.

Ответ:
Зевс.

Комментарий:
Из разрубленной головы Зевса должна была родиться Афина.

Источник:
http://lib.ru/POEEAST/LUKIAN/lukian1_3.txt

Автор:
Евгений Ярков (Тюмень)

Вопрос 20:
(pic: 20090821.jpg)
   Перед вами вымышленное существо, которое Урсула Ле Гуин назвала
сложносоставным словом из тринадцати букв. Напишите это слово.

Ответ:
Палиндромадер.

Комментарий:
Дромадер-палиндром.

Источник:
http://www.ursulakleguin.com/Poetry-Palindromedary.html

Автор:
Евгений Ярков (Тюмень)

Вопрос 21:
Согласно русскому фольклору, у НЕГО сверху кожа, снизу кожа, а в
середине пусто. ОН дал название объекту, который у взрослых людей имеет
овальную форму, а у детей - круглую. Назовите этот объект двумя словами.

Ответ:
Барабанная перепонка.

Комментарий:
ОН - это барабан.

Источник:
   1. http://klassikpoez.boom.ru/azbuka.htm
   2. http://ru.wikipedia.org/wiki/Барабанная_перепонка

Автор:
Евгений Ярков (Тюмень)

Вопрос 22:
Заголовок статьи в газете "Московский комсомолец" о тюремном педагоге
состоит из трех слов. Второе слово - "для", а первое и третье пишутся
одинаково и отличаются только ударением. Воспроизведите этот заголовок.

Ответ:
"УрОк для Урок".

Комментарий:
Урки - заключенные.

Источник:
http://www.zagolovki.ru/article/01Oct2008/urok

Автор:
Евгений Ярков (Тюмень)

Вопрос 23:
Прослушайте отрывок из стихотворения Игоря Иртеньева о суете московских
улиц, в котором мы пропустили первую строчку.
   [Строка пропущена]
   Толкают тушь по семь рублей,
   Еврей пугливый к водопою
   Спешит с еврейкою своей.
   Пропущенная строчка упоминается в начале известной поэмы наряду с
причерноморской исторической областью. Назовите эту область.

Ответ:
Бессарабия.

Комментарий:
Строчка - "цыгане шумною толпою. В поэме Пушкина "Цыганы" "Цыганы шумною
толпой по Бессарабии кочуют".

Источник:
   1. http://lib.ru/ANEKDOTY/irtenew.txt
   2. http://ru.wikipedia.org/wiki/Бессарабия
   3. http://ilibrary.ru/text/445/p.1/index.html

Автор:
Евгений Ярков (Тюмень)

Вопрос 24:
В пятнадцатой главе романа Умберто Эко "Остров накануне" герой
произносит фразу "Il tuon dal ciel fu dopo" [иль тюон даль сьель фю
допо], дословный перевод которой - "Гром грянул позже". Однако
некоторые, упоминая некое изобретение, указывают и другой перевод этой
фразы, звучащий как "Тифон, к маяку!". Назовите это изобретение.

Ответ:
Маятник Фуко.

Комментарий:
"Il tuon dal ciel fu dopo" - это анаграмма "Il pendolo di Foucault",
названия другого произведения Умберто Эко.

Источник:
http://petro-gulak.livejournal.com/780140.html

Автор:
Евгений Ярков (Тюмень)

Вопрос 25:
В марте 2009 года произошел беспрецедентный случай для шоу "Такси" на
ТНТ - впервые за всю историю проекта участники - они были иностранцами -
воспользовались неким изобретением. Это изобретение впервые применил
авиатор Адольф Пегу, выполняя длительную бочку. Что это за изобретение?

Ответ:
Ремни безопасности.

Зачет:
По смыслу.

Комментарий:
До этого ни разу участники не пристегивались. Иностранцы же, сев в
такси, пристегнулись. Пегу использовал ремни, чтобы не выпасть из
самолета при перевороте.

Источник:
   1. http://apashenko.livejournal.com/90484.html
   2. http://www.popmech.ru/article/1200-derzhite-menya-troe/

Автор:
Евгений Ярков (Тюмень)

Вопрос 26:
Внимание, в вопросе есть замены.
   Идея, пошатнувшая устои шестнадцатого века, привела к появлению
фразы, в которой ПЕРВЫЙ назывался ВТОРЫМ бога. ПЕРВЫЙ - это имя
собственное, а синонимом к слову "ВТОРОЙ" является слово "оппонент", при
этом ПЕРВЫЙ отличается от ВТОРОГО одной буквой. Назовите ПЕРВОГО.

Ответ:
Коперник.

Комментарий:
Коперник - богу соперник, идея - гелиоцентрическая система.

Источник:
http://www.foxfox.ru/child/out/universe/647.html

Автор:
Евгений Ярков (Тюмень)

Вопрос 27:
Первые конструкции этих устройств появились в Москве в середине 30-х
годов и в зависимости от высоты обозначались H-10 [аш десять] или H-30
[аш тридцать]. В 1952 году их сменили более современные конструкции типа
"ЭМ" [Эльдар - Максим]. Расшифруйте первую букву аббревиатуры "ЭМ"
словом латинского происхождения.

Ответ:
Эскалатор.

Комментарий:
ЭМ - эскалатор метрополитена. Речь идет об эскалаторах для московского
метро.

Источник:
http://www.vokrugsveta.ru/vs/article/772/

Автор:
Евгений Ярков (Тюмень)

Вопрос 28:
Газетная статья о читающем с помощью шрифта Брайля человеке содержит
пять слов из популярной песни Валерия Ободзинского. Напишите эти пять
слов.

Ответ:
В каждой строчке только точки.

Комментарий:
Шрифт Брайля - точечный.

Источник:
   1. http://www.t-i.ru/article/8874/
   2. http://www.all-lyrics.biz/songs/obodzinskiy/vostochnaya-pesnya.html

Автор:
Евгений Ярков (Тюмень)

Вопрос 29:
В Лондоне существует множество мужских клубов. Так, например, клуб
"Карлтон" и по сей день является "гнездом" консерваторов. Согласно
устоявшейся традиции, все премьер-министры от консервативной партии по
традиции обязаны быть его членами. В связи с этим одного
премьер-министра даже провозгласили почетным мужчиной. Назовите этого
премьер-министра.

Ответ:
[Маргарет] Тэтчер.

Комментарий:
Женщину в этот мужской клуб допускать было немыслимо, вот и стали
считать Тэтчер мужчиной.

Источник:
http://www.vokrugsveta.ru/vs/article/773/

Автор:
Евгений Ярков (Тюмень)

Вопрос 30:
   <раздатка>
   - Вам звонят из Уфы. - [Пропуск]. -
   Плохо слышно, увы. - [Пропуск]. -
   Все архивы Уфы [пропуск],
   Не нашли мы, а вы? - [Пропуск].
   </раздатка>
   Перед вами часть стихотворения Александра Кушнера о плохом телефонном
соединении, в котором мы сделали пропуски. Восстановите любой из
пропусков одним словом.

Ответ:
Перерыв.

Комментарий:
В первой, второй и четвертой строке "перерыв" - существительное, в
третьей - деепричастие.

Источник:
http://www.litera.ru/stixiya/authors/kushner/u-menya-zazvonil.html

Автор:
Евгений Ярков (Тюмень)

Тур:
3 тур

Вопрос 1:
Персонаж произведения Михаила Успенского, находясь перед НИМ, полагает,
что справа уже давно закончился фураж, а слева наметился острый дефицит
женщин. Прилагательное, используемое в ЕГО названии, больше подходит к
характеристике неправильного образа жизни. Назовите ЕГО двумя словами.

Ответ:
Распутный камень.

Зачет:
Распутный столб и т.п., зачет по слову "распутный".

Комментарий:
На камне, как водится, написано: "Прямо ехать - убиту быти. Налево ехать
- женату быти. Направо ехать - коня потеряти".

Источник:
М. Успенский. Там, где нас нет.

Автор:
Евгений Ярков (Тюмень)

Вопрос 2:
Узник литературы Андрей Вознесенский остроумно замечает, что сам он
[пропуск] языка. Восстановите пропуск двумя буквами и одним символом.

Ответ:
З/к.

Комментарий:
"Я з/к языка", говорит Вознесенский.

Источник:
А. Вознесенский. Улисс улиц.

Автор:
Евгений Ярков (Тюмень)

Вопрос 3:
Многим известна фраза "Принцессы не какают". Остроумный Варлам Барлам в
качестве объяснения этого феномена упоминает другую фразу, которую можно
было встретить и в Моральном кодексе строителя коммунизма, и даже в
Сталинской конституции. Напишите эту фразу.

Ответ:
Кто не работает, тот не ест.

Зачет:
По смыслу.

Комментарий:
Известный лозунг появился сначала в труде Ленина "О голоде". Логично,
что принцессы не работают, а потому не едят.

Источник:
   1. http://sirin.livejournal.com/1129742.html
   2. http://azbyka.ru/hristianstvo/bibliya/novyi_zavet/krylatye_frazy/4g86.shtml

Автор:
Евгений Ярков (Тюмень)

Вопрос 4:
Прослушайте стихотворение Ренаты Мухи, в котором мы сделали несколько
пропусков.
   Был [первый пропуск] дом не нужен,
   Носилась вечно по делам.
   Но, постарев, вернулась к мужу,
   И [второй пропуск] пополам.
   Первый пропуск представляет собой термин, который обычно
отождествляют с грызуном. Восстановите второй пропуск.

Ответ:
Делит угол.

Зачет:
Угол делит.

Комментарий:
Первый пропуск - биссектриса, которая обычно как крыса.

Источник:
http://users.livejournal.com/_katusha_/154228.html

Автор:
Евгений Ярков (Тюмень)

Вопрос 5:
ЕГО находили в Помпеях, в укрепленных замках, во многих памятниках
разных культур. Христиане пытались доказать, что ОН составляет слова
"pater noster", но безуспешно. Следуя греческому алфавиту, надпись,
составленная из ЕГО частей, гласит: "Сеятель, ползанием или из ростков
растения, создает колеса мироздания". Напишите любое из слов ЕГО
составляющих.

Ответ:
Любое из слов: Sator, Arepo, Tenet, Opera, Rotas.

Комментарий:
Речь идет о магическом буквенном квадрате "Sator".

Источник:
Б. Вербер. Энциклопедия относительного и абсолютного знания.

Автор:
Евгений Ярков (Тюмень)

Вопрос 6:
Один критик, негативно отзывающийся о творчестве британского скульптора
Генри Мура, видел в фамилии последнего ЭТО. В серии популярных
произведений ЭТО присваивалось весьма ограниченному кругу лиц. Напишите
ЭТО.

Ответ:
00.

Зачет:
Два нуля.

Комментарий:
"Вот и до двух нулей доигрался", - говорил критик Вознесенскому. Два
нуля - префикс агентов британской разведки, имеющих лицензию на
убийство.

Источник:
   1. А. Вознесенский. Улисс улиц.
   2. http://en.wikipedia.org/wiki/00_Agent#List_of_00s

Автор:
Евгений Ярков (Тюмень)

Вопрос 7:
Прослушайте цитату из анализа стихотворения: "Одним из классических
образцов ранней поэзии этого автора является стихотворение "НЕНАВИСТЬ!".
Название произведения уже режет слух, в нем выражено негодование творца,
которого избалованная публика принимает за раба, готового выполнить
любое ее желание. Мы не просим вас назвать автора, ответьте, какое слово
мы заменили словом "ненависть"".

Ответ:
НАТЕ.

Комментарий:
Слово "HATE" по-английски обозначает ненависть.

Источник:
   1. http://litevv.narod.ru/razbor_stixov/mayakovsky.html
   2. http://www.vmayakovsky.ru/book/72/

Автор:
Евгений Ткаченко (Тюмень)

Вопрос 8:
Внимание, в вопросе есть замена.
   Во время трансляции матча Континентальной хоккейной лиги "Атлант" -
"Динамо" (Минск) комментатор телеканала "Спорт", характеризуя сугубо
оборонительный стиль игры минской команды, говорит, что "Динамо"
показывает ТАКОЙ хоккей. При этом он отмечает, что Белоруссия вообще
ТАКАЯ страна, правда, не в наше время. Припомните историю Белоруссии и
ответьте, какой эпитет мы заменили словом "ТАКОЙ".

Ответ:
Партизанский.

Источник:
Трансляция матча "Атлант" - "Динамо" (Минск), телеканал "Спорт",
13.11.2008 г.

Автор:
Владимир Сорокин (Вологда)

Вопрос 9:
Болельщики футбольного клуба "Зенит" после серии вялых, невыразительных
матчей своей команды вывесили на трибунах баннер с призывом,
общепринятый смысл которого заключается в ограничении передвижения.
Однако в данном случае он, наоборот, требует от футболистов двигаться
активнее. Воспроизведите надпись на этом баннере.

Ответ:
По газону не ходить.

Источник:
Трансляция матча "Зенит" - "Спартак" (Нальчик), телеканал "Спорт",
24.06.2007 г.

Автор:
Дмитрий Кукулин (Тюмень)

Вопрос 10:
Внимание, в вопросе есть замена.
   Российский вратарь Николай Хабибулин за свою непробиваемость получил
в Национальной хоккейной лиге прозвище "Резной палисад". В истории
известен ряд "Резных палисадов". А еще об одном из "Резных палисадов" в
конце 70-х годов нам поведал некий коллектив. Мы не спрашиваем, что мы
заменили на "Резной палисад". Назовите этот коллектив.

Ответ:
"Pink Floyd".

Комментарий:
"Резной палисад" - замена слову "Стена".

Источник:
   1. http://eng.tsure.ru/University/Faculties/Femp/TPPI/downloads/koval.htm
   2. http://ru.wikipedia.org/wiki/The_Wall

Автор:
Владимир Сорокин (Вологда)

Вопрос 11:
Статистика показывает, что 91% беременных женщин в России собираются
кормить своих будущих детишек исключительно грудью. Но, к сожалению,
лишь очень немногим мамам удается сохранить естественное вскармливание
хотя бы до шести месяцев. Статья об этой проблеме в журнале "Мой
ребенок" за декабрь 2008 года называется фразой, произнесенной Василием
Ливановым в 1970 году. Благодаря этой фразе, в частности, его герой
подружился с новой неожиданной знакомой, а впоследствии еще и проявил
свои таланты. Как называется статья?

Ответ:
"У вас молоко убежало".

Комментарий:
Герой Ливанова Карлсон произносит фразу в мультфильме 1970 года "Карлсон
вернулся".

Источник:
   1. "Мой ребенок", 2008, N 12. - С. 122.
   2. http://ru.wikipedia.org/wiki/Карлсон_вернулся

Автор:
Владимир Сорокин (Вологда)

Вопрос 12:
Джордж Харрисон ЕЙ посвятил песню "Something", Пол Маккартни - целый
альбом "Ecce Cor Meum" ("Познай мое сердце"). В 1996 году вышел фильм об
ИХ клубе по роману Оливии Голдсмит. А вот создатели передачи на одном
телеканале, видимо, решили, что такое название лишит их части
потенциальной аудитории. На что они заменили второе слово в названии
фильма?

Ответ:
Бывших.

Комментарий:
Песни посвящены первым женам. Фильм называется "Клуб первых жен",
передача на ТНТ - "Клуб бывших жен".

Источник:
   1. http://lenta.ru/articles/2008/11/13/beatles/
   2. http://www.utro.ru/articles/2006/09/26/587138.shtml
   3. http://www.kinoexpert.ru/index.asp?comm=4&num=495
   4. http://www.tnt-tv.ru/programs/ExWifeClub/

Автор:
Мария Сергушичева (Вологда)

Вопрос 13:
Кастовая система в Индии установилась много веков назад, и появление
новых каст - явление очень редкое. Тем не менее, после колонизации Индии
Великобританией возникла каста баньев. Ответьте абсолютно точно, чем
занимались ее представители.

Ответ:
Торговали с англичанами.

Источник:
"GEO", 2002, N 5. - С. 94.

Автор:
Юрий Денисов (Вологда)

Вопрос 14:
В "Русской правде" - первом отечественном источнике уголовного права -
предусмотрена ответственность за такие виды краж, как кража из закрытого
помещения, кража хлеба, конокрадство, кража холопа, сельскохозяйственных
продуктов и скота, а также кража ЭТОГО. Какое звание незаконно присвоил
себе герой произведения XX века незадолго до совершения кражи ЭТОГО?

Ответ:
Гроссмейстер.

Комментарий:
Предусмотрена ответственность за кражу ладьи. А Бендер украл ладью во
время шахматной партии, назвавшись гроссмейстером.

Источник:
   1. http://www.krugosvet.ru/articles/68/1006870/print.htm
   2. http://law.isu.ru/ru/science/vestnik/20044/georgievskiy.html

Автор:
Дмитрий Кукулин (Тюмень)

Вопрос 15:
В одном из рассказов Виктора Пелевина фигурирует эротическое приложение
к газете "Московский комсомолец". Название этого еженедельника чем-то
похоже на название другого, пожалуй, самого известного издания в этой
сфере и состоит из семи букв, первые две из которых - "МК". Напишите
остальные пять.

Ответ:
Сутра.

Комментарий:
Полное название - "МК-сутра".

Источник:
В. Пелевин. Краткая история пэйнтбола в Москве.
http://pelevin.nov.ru/rass/pe-paint/1.htm

Автор:
Сергей Веселков (Вологда)

Вопрос 16:
Закончите ироническое высказывание американского поэта Роберта Фроста:
"Мозг - удивительный орган. Он начинает работать, как только вы
просыпаетесь утром, и не прекращает до тех пор, пока вы не ...".

Ответ:
"... приходите на службу".

Зачет:
По смыслу.

Источник:
   1. http://www.diary.ru/~geraldine/p41280573.htm
   2. http://ru.wikipedia.org/wiki/Фрост,_Роберт

Автор:
Владимир Сорокин (Вологда)

Вопрос 17:
Кое-кто, услышав сейчас ЭТОТ ПОЭТИЧЕСКИЙ ВОПРОС, отреагировал бы
словами: "Что курил автор вопроса?". Да и сам автор в следующих строчках
стихотворения говорит, что не только курил, но и пил. А ведь ответ на
заданный им вопрос довольно неоднозначен. Христиане и буддисты могли бы
ответить, что "третье", мусульмане - "второе", иудеи - "таки, шестое".
Процитируйте ЭТОТ ПОЭТИЧЕСКИЙ ВОПРОС.

Ответ:
"Какое, милые, у нас тысячелетье на дворе?".

Комментарий:
Борис Пастернак "Про эти стихи":
   "... В кашне, ладонью заслонясь,
   Сквозь фортку крикну детворе:
   Какое, милые, у нас
   Тысячелетье на дворе?
   Кто тропку к двери проторил,
   К дыре, засыпанной крупой,
   Пока я с Байроном курил,
   Пока я пил с Эдгаром По?.."
   У христиан идет 2009 год. У буддистов сейчас 2603 год, у мусульман -
1430-й. Самое "старое" время у иудеев - для них наступил 5769 год.

Источник:
   1. Б.Л. Пастернак. Про эти стихи.
   2. http://www.kostyor.ru/kostyor1-03/archivar1-03.php

Автор:
Сергей Веселков (Вологда)

Вопрос 18:
Мардук, Ра, Аполлон, Индра, Гильгамеш, Беллерофонт, Тор, Кадм, Сигурд
ДЕЛАЛИ ЭТО. А в наше время, около двадцати лет назад, ЭТО смог СДЕЛАТЬ
Александр Абдулов. А что именно?

Ответ:
Убить дракона.

Комментарий:
Все они - драконо- и змееборцы.

Источник:
   1. Электронная энциклопедия "Легенды и мифы". - Издательство "Media",
2002. - Статья "Георгий Победоносец".
   2. http://ru.wikipedia.org/wiki/Убить_дракона

Автор:
Сергей Веселков (Вологда)

Вопрос 19:
Внимание, в вопросе есть замена.
   В 1801 году английский ученый Хэтчет открыл новый элемент. Однако
коллеги с ним не согласились и решили, что на самом деле это уже
известный СИЗИФ. В итоге в 1846 году всё же было доказано, что данный
элемент вовсе не СИЗИФ, а просто является его ближайшим родственником.
Мы не просим вас воспроизвести замену. Скажите, какое название в итоге
получил открытый Хэтчетом элемент.

Ответ:
Ниобий.

Комментарий:
На СИЗИФ заменен Тантал. И тот, и другой персонажи, согласно мифологии,
испытывали муки. Ниоба являлась дочерью Тантала, а элемент Ниобий очень
близок по свойствам Танталу.

Источник:
Айзек Азимов. Занимательная мифология. Новая жизнь древних слов. - М.:
Центрполиграф, 2002. - С. 124.

Автор:
Юрий Денисов (Вологда)

Вопрос 20:
Существует версия, что первоначально в этом устойчивом выражении
присутствовала рыба. В зарослях камыша или там, где в иле тонут коряги,
попавшаяся на крючок рыба легко может оборвать леску и уйти. А в [два
слова пропущено] уйти рыбе будет тяжело. Мы не просим заполнить пропуск.
Воспроизведите это выражение.

Ответ:
Вывести на чистую воду.

Комментарий:
Пропущены слова "чистой воде".

Источник:
http://www.comics.ru/dic/dic47.htm

Автор:
Дмитрий Кукулин (Тюмень)

Вопрос 21:
В опубликованной на сайте www.utro.ru статье Бориса Сурда "В кризисе
виноват всего один человек" рассказывается о первопричинах финансового
кризиса. В том числе упоминается ОН - некий гражданин США, купивший в
2003 году дом в калифорнийском городе Стоктон. Примером ЕГО может
служить женщина-повар, известная как "Тифозная мери". Назовите ЕГО двумя
словами.

Ответ:
Нулевой пациент.

Комментарий:
Медикам очень хорошо знаком такой термин, как "нулевой пациент" - это
человек, с которого начинается эпидемия. Часто получается так, что сам
он не страдает от симптомов того или иного заболевания, являясь при этом
лишь его разносчиком.

Источник:
http://www.utro.ru/articles/2009/02/09/796535.shtml

Автор:
Павел Рогозин (Миасс)

Вопрос 22:
Ингмар Бергман говорил, что счастье - это хорошее здоровье и плохая ОНА.
Муж Елены Дьяконовой создал в XX веке известное произведение, упомянув в
названии ЕЕ. Напишите название этого произведения, состоящее из двух
слов.

Ответ:
"Постоянство памяти".

Комментарий:
Елена Дьяконова - жена Сальвадора Дали.

Источник:
   1. http://ru.wikiquote.org/wiki/Ингмар_Бергман
   2. http://ru.wikipedia.org/wiki/Постоянство_памяти

Автор:
Максим Меляев (Миасс)

Вопрос 23:
Древний Египет - рабы, земля, золотая муха. Древняя Русь - золотая
гривна. Назовите тот, который в СССР был закреплен всего лишь за 17
человеками.

Ответ:
Орден "Победа".

Комментарий:
Перечисленное являлось высшими военными наградами. В СССР высшей военной
наградой в 1943 году стал орден "Победа", он был закреплен (прикреплен к
одежде) в общей сложности за 17 человеками.

Источник:
   1. Современные игры разума. - Ростов-на-Дону, 2007.
   2. http://www.edu-zone.net/show/75163.html
   3. http://www.dazzle.ru/spec/grivna.shtml
   4. http://ru.wikipedia.org/wiki/Орден_%C2%ABПобеда%C2%BB

Автор:
Евгений Горячев (Вологда)

Вопрос 24:
Доказано, что в четвертом тысячелетии до нашей эры на одном острове было
обнаружено большое количество этого металла и началось его применение.
Многим из вас известна произошедшая на этом острове трагедия, в
результате которой погиб его правитель. Мы не спрашиваем, о каком
острове идет речь. Назовите вышеуказанный металл, название которого, по
одной из версий, произошло от названия острова.

Ответ:
Медь.

Комментарий:
Речь идет о Кипре, где происходит действие трагедии Шекспира "Отелло".
Латинское название меди Cuprum происходит от названия острова Кипр.

Источник:
   1. Большая энциклопедия Кирилла и Мефодия - 2005.
   2. http://ru.wikipedia.org/wiki/Отелло

Автор:
Ираклий Назгаидзе (Тбилиси)

Вопрос 25:
Этот небольшой городок, население которого не превышает 32 тысяч
человек, ежегодно принимает сотни тысяч туристов. Во время купального
сезона, когда количество туристов достигает своего пика, город переходит
на активную ночную жизнь, что вызывает восторг у свободомыслящей
молодежи. Поэтому город часто в шутку называют "спальня". Хотя следует
отметить, что туристы посещают этот город скорее всё же из-за огромного
количества исторических памятников, что не удивительно, так как один из
семи находился именно здесь. Назовите современное название этого города.

Ответ:
Бодрум.

Комментарий:
Бодрум (древний Галикарнас) - один из известных курортов Турции. Бодрум
в шутку называют "Bedroom". Там находятся развалины галикарнасского
мавзолея.

Источник:
   1. http://en.wikipedia.org/wiki/Bodrum
   2. http://en.wikipedia.org/wiki/Mausoleum_at_Halicarnassus

Автор:
Ираклий Назгаидзе (Тбилиси)

Вопрос 26:
В 1559 году временный комендант Макао обвинил этого человека в
превышении должностных полномочий, арестовал его и отправил на корабле в
Гоа. По дороге корабль потерпел крушение в устье реки Меконг, в
результате чего на дно отправилось всё, кроме рукописи, которую он спас,
в полубессознательном состоянии добравшись до берега. Добравшись до Гоа,
он потребовал суда и был оправдан. После этого он отправился в Мозамбик,
где жил на протяжении двух лет. Мы не спрашиваем, кто был этот человек.
Ответьте, под каким названием по сей день известна его рукопись.

Ответ:
"Лузиады".

Комментарий:
Речь идет о Луисе де Камоэнсе и его эпической поэме. Макао и Мозамбик
были португальскими колониями, где поэт жил и работал.

Источник:
Большая энциклопедия Кирилла и Мефодия - 2005.

Автор:
Вахтанг Элердашвили (Тбилиси)

Вопрос 27:
Назовите фильм, вышедший в первой половине 2007 года, статья о котором
на сайте www.film.ru называлась "+1".

Ответ:
"13 друзей Оушена".

Комментарий:
Предыдущий был "12 друзей Оушена".

Источник:
http://www.film.ru/article.asp?id4768

Автор:
Дмитрий Кукулин (Тюмень)

Вопрос 28:
По поводу ПЕРВОГО Уолтер Уинчел сказал, что это "красиво оформленный
сборник фактов, о которых никто ничего не знает и которые никому не
интересны". Что касается ВТОРОГО, то Сафир Мориц-Готлиб считает, что это
"совесть женщины: она ничего не делает, не посоветовавшись с ним". Вы же
назовите то, что увидело свет 4 января 1947 года в Ганновере.

Ответ:
Журнал "Шпигель".

Комментарий:
Уинчел говорил о журнале, Мориц-Готлиб - о зеркале. Зеркало по-немецки -
"шпигель".

Источник:
   1. В.Н. Зубков. Мудрость тысячелетий от А до Я.
   2. http://de.wikipedia.org/wiki/Der_Spiegel

Автор:
Ираклий Назгаидзе (Тбилиси)

Вопрос 29:
Когда этому человеку предложили сменить логотип его компании на
швейцарский крест, он ответил: "Крест не заменит мое место жительства,
так как это не только фирменный знак, но и фамильный герб моей семьи".
Швейцарская компания "Herzog und de Meuron Architekten" спроектировала
это "место жительства", и с 2008 года оно превратилось в символ одной
столицы. Догадайтесь, что мы заменили словами "место жительства", и
назовите эту столицу.

Ответ:
Пекин.

Комментарий:
Речь идет о "птичьем гнезде". Основатель компании "Нестле" - Анри Нестле
- говорил о логотипе компании. Пекинский олимпийский стадион "Птичье
гнездо" спроектировала компания "Herzog und de Meuron Architekten".

Источник:
http://www.type-market.ru/tmsite.php?menu=zametki&podmenu=worldexp&action=detail&id=7

Автор:
Вахтанг Элердашвили (Тбилиси)

Вопрос 30:
Внимание, после этого вопроса перерыв будет чуть длиннее.
   В одной миниатюре измотанная крольчиха, со вздохом откинувшись на
мягкую траву, говорит кролику: "Знаешь, я так еще ни разу в жизни...
Потрясающе!". Названием какого бразильского произведения озаглавлена эта
миниатюра?

Ответ:
"Одиннадцать минут".

Комментарий:
"Одиннадцать минут" - произведение Пауло Коэльо.

Источник:
http://kvn-dvgu.narod.ru/litera.html

Автор:
Евгений Ярков (Тюмень)

Тур:
4 тур

Вопрос 1:
На одной пессимистической карикатуре человек с камнем на шее смотрит на
рыбу, привязанную к некоему предмету. А в песенке Сергея Гогина этот
предмет сравнивался с расфасованным выдохом. Назовите этот предмет двумя
словами.

Ответ:
Воздушный шарик.

Зачет:
Надувной шарик.

Комментарий:
Рыба с суицидальными намерениями взлетает вверх на шарике. А у С. Гогина
- "Шарики воздушные, спутники неверные, рожицы нескучные, щеки
безразмерные, выдох, расфасованный в ярких упаковочках, небу адресованы
слезки на веревочках".

Источник:
   1. http://mi3ch.livejournal.com/1321944.html
   2. http://iatp.ulstu.ru/~gogin/poems/cuplet017.rhtml

Автор:
Евгений Ярков (Тюмень)

Вопрос 2:
Андрей Вознесенский вспоминал, как во время блокады по темному небу
блуждали лучи прожекторов и иногда появлялся ОН. ЕГО в 832 году
использовал король Ангус, отдавая должное покровителю его народа.
Назовите его абсолютно точно.

Ответ:
Андреевский крест.

Зачет:
По смыслу. Незачет: Андреевский флаг; Шотландский флаг.

Комментарий:
Лучи прожекторов пересекались, что напомнило Вознесенскому Андреевский
крест. Андреевский крест лег в основу шотландского флага, утвержденного
в 832 году. А вот Андреевский флаг (флаг ВМФ России) не подходит, в том
числе и по цветовой гамме.

Источник:
   1. А. Вознесенский. Улисс улиц.
   2. http://ru.wikipedia.org/wiki/Андреевский_крест

Автор:
Евгений Ярков (Тюмень)

Вопрос 3:
(pic: 20090822.jpg)
   Перед вами часть плаката, опубликованного в 2008 году. Какие шесть
английских букв мы скрыли от вас на этом изображении?

Ответ:
RTH SEA.

Комментарий:
На плакате-карте в виде государств изображены участники финального этапа
чемпионата Европы по футболу 2008 и неудачница Англия. Справа от Англии
- Северное море (North Sea).

Источник:
http://mi3ch.livejournal.com/1435005.html

Автор:
Евгений Ярков, Ирина Яркова (Тюмень)

Вопрос 4:
Во время интервью чешский легионер московского "Динамо" Петр Чаянек был
представлен болельщикам так: один из шести игроков "Динамо", игравших за
"Ак Барс". Какие шесть букв мы пропустили в вопросе?

Ответ:
До того.

Комментарий:
"До-то-го" - это традиционная чехословацкая хоккейная кричалка.

Источник:
Матч "Ак Барс" - "Динамо", четвертьфинал Кубка Гагарина.

Автор:
???

Вопрос 5:
Персонаж Виктора Гюго говорил, что медь, звучащая на крестинах, может
убивать. А всё потому, что безбожники-французы ДЕЛАЮТ ЭТО. В России тоже
ДЕЛАЛИ ЭТО во многом из-за Нарвского конфуза. А что именно делали?

Ответ:
Переливали колокола на пушки.

Зачет:
По смыслу.

Комментарий:
Благовест во время крестин издавал колокол. Колокола в революционной
Франции переливали на пушки. После поражения под Нарвой Петр I приказал
перелить колокола на пушки.

Источник:
   1. В. Гюго. Девяносто третий год.
   2. http://svvpsu.org.ru/smf_forum/index.php?topic=237.420

Автор:
Евгений Ярков (Тюмень)

Вопрос 6:
На одном изображении можно увидеть неподвижную окровавленную руку,
торчащую из-под большого валуна. Название этого изображения лишь одной
буквой отличается от фразеологизма. Напишите название этого изображения.

Ответ:
"Сизифов труп".

Комментарий:
"Сизифов труд" - известная идиома, на картинке Сизиф придавлен камнем.

Источник:
http://methan.livejournal.com/106526.html

Автор:
Евгений Ярков (Тюмень)

Вопрос 7:
Дуплет.
   1. В современном мире домашние животные нередко становятся обузой.
Как отмечает Британское королевское общество защиты животных, особенно
часто встречаются случаи издевательства над собаками. Заголовок статьи
об этом лишь одной буквой отличается от названия фантастического
произведения. Напишите этот заголовок.
   2. ВалерИ ЛесАж в статье о мюзикле "Дракула" пишет: "Воинственный
[ПРОПУСК] на вечные страдания по погибшей любимой, пытается обнаружить
свою Мину спустя века". Пропуск лишь одной буквой и запятой отличается
от названия фантастического произведения. Восстановите пропуск.

Ответ:
   1. "Трудно быть догом".
   2. "... граф, обреченный...".

Источник:
   1. http://www.lobnya-online.ru/news.php?pid=2309
   2. http://lib.ru/STRUGACKIE/be_god.txt
   3. http://forum.frenchmusicals.ru/viewtopic.php?f=21&t=232&st=0&sk=t&sd=a&start=2145
   4. http://lib.ru/STRUGACKIE/grad_obr.txt

Автор:
Евгений Ярков (Тюмень)

Вопрос 8:
(pic: 20090823.jpg)
   Перед вами часть адреса интернет-сайта. Кому этот сайт посвящен?

Ответ:
[Папе Римскому] Бенедикту XVI.

Зачет:
Йозефу Ратцингеру.

Комментарий:
www.benediktxvi.ru, xvi - это не буквы, а цифры. Кстати, шрифт текста -
Pontifex.

Источник:
http://www.benediktxvi.ru

Автор:
Евгений Ярков (Тюмень)

Вопрос 9:
Центральным моментом оперы Андре Гретри, поставленной по мотивам сказки
"Красавица и Чудовище", является эпизод, в котором страшный зверь
подставляет ладонь под брошенный испугавшимся купцом аленький цветок.
Догадавшись, что это был за цветок, назовите имя зверя.

Ответ:
Азор.

Комментарий:
"А роза упала на лапу Азора" - известный палиндром. Чудовище зовут Азор.
Во многих постановках он действительно ловит и рассматривает Розу.

Источник:
   1. http://fr.wikisource.org/wiki/Z%C3%A9mire_et_Azor
   2. http://fandrom.narod.ru/erik_phantomish_zemira_and_azor.html

Автор:
Евгений Ярков (Тюмень)

Вопрос 10:
Во время съемок фильма-ремейка "Фанфан-Тюльпан" исполнительница главной
роли довольно долго не хотела играть пикантные сцены, отчего эпизоды
приходилось снимать снова и снова. В одной из рецензий отмечено, что
этот фильм считается самой заметной [ДВА СЛОВА ПРОПУЩЕНО] периода романа
с Томом Крузом. Восстановите два пропущенных слова.

Ответ:
Работой Пенелопы.

Комментарий:
В греческой мифологии Пенелопа, супруга Одиссея, ждала возвращения мужа
из-под Трои в течение двадцати лет, упорно отвергая домогательства
многочисленных женихов. Отсюда иносказательно: "работа Пенелопы" -
нескончаемая работа.

Источник:
   1. http://www.ntvplus.ru/off-screen?id=29895
   2. http://letter.com.ua/phrase/p2.php

Автор:
Евгений Ярков (Тюмень)

Вопрос 11:
[Ведущему: никоим образом не показывать, что Лось - имя собственное.]
   В фантастическом произведении Михаила Зайцева ОНА - это условное
название группы элитных антипатов. А в другом произведении ОНА
влюбляется в Лося. Назовите ЕЕ.

Ответ:
Аэлита.

Комментарий:
Аэлита - дочь Марсианского вождя в произведении графа Алексея Толстого.
Антипаты элитные в произведении Михаила Зайцева именовались А-элитой,
или просто Аэлитой.

Источник:
   1. М. Зайцев. А-элита.
   2. А. Толстой. Аэлита.

Автор:
Евгений Ярков (Тюмень)

Вопрос 12:
(pic: 20090824.jpg)
   Часто военная форма подбиралась с таким расчетом, чтобы ранение
оставалось незаметным и не снижало боевой дух. Перед вами картина
Роберта Гибба, на которой изображен небольшой отряд английских солдат,
вынужденных прикрывать слишком широкий фронт. Военный корреспондент,
наблюдавший за сражением с холмов, сравнил этих солдат с НЕЙ. Назовите
ЕЕ тремя словами.

Ответ:
Тонкая красная линия.

Комментарий:
По одной из версий, военная форма англичан была сделана красной для
того, чтобы трусливые солдаты не сбегали с поля боя при виде раненых
однополчан. Корреспондент "Таймс" У.Г. Рассел, наблюдавший за
самоотверженной битвой англичан, растянувшихся в длинный ряд, назвал их
"тонкой красной линией, окованной сталью".

Источник:
   1. А. Уланов. Крест на башне.
   2. http://ru.wikipedia.org/wiki/Военная_форма
   3. http://en.wikipedia.org/wiki/The_Thin_Red_Line_(Battle_of_Balaclava)
   4. http://www.directart.co.uk/mall/more.php?ProdID=196

Автор:
Евгений Ярков (Тюмень)

Вопрос 13:
Внимание! В вопросе словами "ПЕРВЫЙ" и "ВТОРОЙ" заменены словосочетания.
   Персонаж ЭрвЕ БазЕна, сжимающий в кулаке голову змеи, так описывает
ее предсмертные конвульсии. Сначала тело умирающей змеи было похоже на
спираль, затем на посох епископа, а потом на ПЕРВЫЙ, который в момент
смерти змеи сменился на неподвижный ВТОРОЙ. В этом вопросе есть и
ПЕРВЫЙ, и ВТОРОЙ. Какие словосочетания мы заменили на "ПЕРВЫЙ" и на
"ВТОРОЙ"?

Ответ:
Вопросительный знак, восклицательный знак.

Зачет:
В любом порядке.

Комментарий:
Предсмертные изгибы змеи напомнили герою Базена вопросительный и
восклицательный знаки.

Источник:
http://lib.ru/INPROZ/BAZEN/rezo_1.txt

Автор:
Евгений Ярков (Тюмень)

Вопрос 14:
Давно известно, что ИКС - это село в Болгарии. Также давно известно, что
в Великом Новгороде существует улица ИКСА. А еще 24 апреля 2007 года
один ИКС сыграл свой первый матч за "Барселону". Назовите ИКС.

Ответ:
Боян.

Комментарий:
Боян - легендарный древнерусский певец-сказитель. Боян - село в
Болгарии. Боян Кркич - игрок ФК "Барселона".

Автор:
???

Вопрос 15:
В журнале "Maxim" есть рубрика, публикующая различные рекомендации
читателей. Об этих читателях известно очень мало - от силы имя или
интернет-ник. Два слова, которыми называется рубрика, некоторое время
занимали четвертое место в списке, созданном в 1722 году. Напишите эти
два слова.

Ответ:
Тайный советник.

Комментарий:
Список - табель о рангах. В 1724 году тайный советник был переименован в
действительного статского советника.
   z-checkdb: Действительный статский советник занял место тайного
советника в 4-м классе, а тайный советник перешел в 3-й (Антон Губанов).

Автор:
Андрей Белкин (Ханты-Мансийск)

Вопрос 16:
Жан-Пьер Люмине в своей книге рассказывает старое персидское предание.
Бабочки пытались познать тайну пламени и пришли к выводу, что узнала всё
об огне только та, что сгорела. Далее автор проводит аналогию с учеными
и "застывшими звездами". Но отмечает, что некоторое представление о них
можно получить из исследований обсерватории Медон. Под каким названием
нам более известны "застывшие звезды"?

Ответ:
Черные дыры.

Комментарий:
Полную информацию может получить только космонавт в самой черной дыре.
Но он уже не вернется.

Источник:
http://www.astronet.ru/db/msg/1180462/node1.html

Автор:
Тимур Кот, Владислав Тайшин (Ханты-Мансийск)

Вопрос 17:
Внимание, в вопросе есть замена.
   Мало какой человек обрадуется, если ему СДЕЛАЮТ ПАКОСТЬ. А вот
тигрята-"отказники" из зоопарка Днепропетровска смогли выжить благодаря
тому, что им СДЕЛАЛИ ПАКОСТЬ. Напишите идиоматическое выражение, которое
мы заменили словами "СДЕЛАЛИ ПАКОСТЬ".

Ответ:
Подложили свинью.

Зачет:
Подложить свинью.

Комментарий:
Тигрятам, от которых отказалась мать, подложили свинью, которая стала их
кормить вместе со своими поросятами.

Источник:
Новости ОРТ от 22.09.2008 г.

Автор:
Елена Богомолова (Нижневартовск)

Вопрос 18:
В гостинице в Тюмени приятельницу автора вопроса расстроил тот факт, что
у нее "сгорел" ИГРЕК. Однако уже на следующий день она была замечена с
работающим ИГРЕКОМ, что послужило причиной для комментария: "Да у тебя
не ИГРЕК, у тебя ИГРЕКикс". Какое слово заменено на "ИГРЕК"?

Ответ:
Фен.

Комментарий:
Фен "сгорел", а затем, как феникс, "восстал из пепла".

Автор:
Юрий Гнатюк (Сургут)

Вопрос 19:
Аббревиатура одной из книг по мотивам сериала "Солдаты" - ТБД. Можно
сказать, что автор книги заменил одно сверхъестественное существо
другим. Напишите полное название книги.

Ответ:
"Трудно быть духом".

Источник:
http://www.books.ru/shop/books/436159

Автор:
Андрей Ситников (Тюмень)

Вопрос 20:
Пословица гласит: "Жена мужу ПЕРВЫЙ, муж жене ВТОРОЙ". ПЕРВЫЙ и ВТОРОЙ
отличаются лишь одной буквой. Первая часть поговорки, видимо,
перефразирует изречение из Книги Бытия, начинающееся так: "Оставит
человек отца своего и мать свою...". Да и в конце пословицы есть нечто
библейское. Назовите ПЕРВЫЙ и ВТОРОЙ.

Ответ:
Пластырь, пастырь.

Комментарий:
"... оставит человек отца своего и мать свою и прилепится к жене своей".

Источник:
   1. В.И. Даль. Пословицы и поговорки русского народа.
   2. Быт. 2:22-24.

Автор:
Ирина Тюшкевич (Тюмень)

Вопрос 21:
Недавно один ресторан предложил своим клиентам два новых гамбургера.
Elephant Burger содержит ингредиенты, популярные в Аризоне: гуакамоле,
островатый соус сальса и мясо жареного ягненка. Второй - O'Burger -
сделан в гавайском стиле: между булочками можно найти креветки, ананасы,
экзотический салат и соус карри. Назовите фамилии людей, в честь которых
сделаны эти блюда.

Ответ:
Маккейн, Обама.

Источник:
http://top.rbc.ru/wildworld/01/11/2008/257677.shtml?print

Автор:
Андрей Ситников (Тюмень)

Вопрос 22:
Однажды Вагрич Бахчанян написал предложение, цитирую: "В этом
предложении [пропуск] буквы". Восстановите два пропущенных слова
абсолютно точно.

Ответ:
Тридцать две. Незачет: 32 (требовались именно слова).

Источник:
В. Бахчанян. Мух уйма.
http://www.zaraz.org/samvydav/thick/muh_ujma/bakhchanian.html

Автор:
Андрей Ситников (Тюмень)

Вопрос 23:
Традиционные пункты маршрута свадебных кортежей в Санкт-Петербурге - это
Медный всадник, Стрелка Васильевского острова, Марсово поле, набережная
Финского залива. Недавно к ним прибавился еще один объект. Некоторые
молодожены останавливаются на Тучковом мосту и прямо с РОЗАМИ
фотографируются на его фоне. Назовите этот объект.

Ответ:
Стадион "Петровский".

Зачет:
Стадион; Стадион "Зенита".

Комментарий:
"Зенит" выиграл чемпионат России в 2007 году, это его домашний стадион.
Розами называются шарфы с символикой клуба.

Источник:
http://www.championat.ru/football/news-78004.html

Автор:
Андрей Ситников (Тюмень)

Вопрос 24:
Одним из символом питерских митьков является скульптура кошки по имени
Тиша. Фамилия ее позаимствована у известного персонажа, которого с
митьками роднит характерная внешняя особенность. Назовите полное имя и
фамилию этой кошки.

Ответ:
Матроскина Тишина.

Комментарий:
Кот Матроскин такой же полосатый, как и Митьки в тельняшках.

Источник:
http://www.fontanka.ru/2006/06/08/166616/

Автор:
Андрей Ситников (Тюмень)

Вопрос 25:
Студия Артемия Лебедева сделала дизайн привычных бытовых предметов в
форме летучей мыши. Когда предмет находится в свободном состоянии, мышь
висит "вниз головой", а когда предмет выполняет свою функцию - мышь
находится "вверх головой". Назовите этот предмет.

Ответ:
Прищепка для белья.

Источник:
http://www.artlebedev.ru/everything/vespertilium/

Автор:
Андрей Ситников (Тюмень)

Вопрос 26:
Вечером 21 ноября 2007 года этот англичанин простоял около двух часов
под огромным зонтом. В этом же году на Рождество лужайку перед его домом
ироничные соотечественники завалили сотнями зонтов. Назовите имя и
фамилию этого англичанина.

Ответ:
Стив Макларен.

Комментарий:
Он стоял под зонтом на матче Англия - Хорватия, после которого англичане
не попали на Евро-2008.

Источник:
   1. http://www.championat.ru/football/news-80820.html
   2. http://www.championat.ru/football/_euro2008/13/calendar/tour/preliminary.html

Автор:
Андрей Ситников (Тюмень)

Вопрос 27:
Представьте себе, что Бриджит Бардо попробовала "Тоблерон". Или что
Грета Гарбо попробовала "Линдт". Или даже что Мэрилин Монро попробовала
"Визит". После всех этих случаев с полным основанием можно утверждать,
что ПЕРВЫЙ во ВТОРОЙ. Назовите имя и фамилию россиянки, которая известна
как ВТОРАЯ в ПЕРВОМ.

Ответ:
Ксения Собчак.

Комментарий:
Первый - шоколад, вторая - блондинка. Перечислены марки шоколада.

Автор:
Андрей Ситников (Тюмень)

Вопрос 28:
Наверняка каждый из вас встречался с F.A.Q. [эф-эй-кью] - ответами на
часто задаваемые вопросы. Пользователь Живого Журнала fritzmorgen
[фрицморген] утверждает в своей статье, что такая система эффективнее
традиционных учебников, так как лучше направлена к ученику: "Когда
ученик читает F.A.Q., он получает ответы именно на те вопросы, которые
его волнуют". Назовите произведение и его создателя, которое автор
статьи назвал далеким предком современных F.A.Q.

Ответ:
"Диалоги", Платон.

Комментарий:
   z-checkdb: Диалоги Платона - не произведение, а литературная форма
(Антон Губанов).

Источник:
http://fritzmorgen.livejournal.com/94553.html

Автор:
Андрей Ситников (Тюмень)

Вопрос 29:
[Ведущему: прочитать цитату, не выделяя интонационно слова в кавычках.]
   Обозреватель сайта Звуки.ру так описала увиденный концерт: "The Best
Of - важный этап для каждого артиста. Это подведение "Итогов", это
значит, что певица на самом деле "Созрела", она "Искала" в себе
"Главное" и нашла его". Назовите имя упомянутого артиста.

Ответ:
Земфира.

Комментарий:
В цитате заключены названия четырех песен Земфиры.

Источник:
http://www.zvuki.ru/R/P/17962/

Автор:
Андрей Ситников (Тюмень)

Вопрос 30:
Однажды в программе "Самый умный" джазмен Игорь Бутман никак не мог
вспомнить имя и фамилию известной женщины, но назвал его после
подсказки, где упоминались алкогольный напиток и знаменитый сыщик. А вы
назовите псевдоним этой женщины.

Ответ:
[Мэрилин] Монро.

Комментарий:
Подсказка звучала так: "Добавьте немного джина и немного Шерлока
Холмса". Тина Канделаки рассудила, что Шерлок Холмс сразу напомнит о
Бейкер-стрит и наведет на фамилию Бейкер. Мэрилин Монро, кстати, снялась
в фильме, который в русском прокате назывался "В джазе только девушки".

Источник:
Телепередача "Самый умный" на канале СТС от 26.01.2008 г.

Автор:
Александр Воробьев (Тюмень)

Тур:
5 тур

Вопрос 1:
Генрих Гейне с болью в сердце писал: "ИКСЫ правят государствами, творят
историю, хотя история запоминает лишь имена ИГРЕКОВ!". Вирджиния Вульф
же сказала, что ИКСЫ веками играли роль зеркала, наделенного волшебным и
обманчивым свойством: отраженный в нем ИГРЕК был вдвое больше
натуральной величины. Назовите ИКС и ИГРЕК в правильном порядке.

Ответ:
Женщина и мужчина.

Источник:
   1. http://www.litportal.ru/genre214/author6084/read/page/2/book29490.html
   2. http://www.aforismo.ru/authors/872/

Автор:
Георгий Мосидзе (Тбилиси)

Вопрос 2:
[Ведущему: при каждом прочтении интонационно указывать на множественное
число слова "президенты", но не слишком явно.]
   В Википедии в категории "Президенты по странам" на букву "С" есть
такие подкатегории:
   Президенты США
   Президенты Сальвадора
   Президенты Сирии
   Президенты Словакии
   Президенты Словении
   Президенты Сомали
   Президенты Суринама
   Напишите абсолютно точно еще одну подкатегорию, которую мы
пропустили.

Ответ:
Президент СССР.

Комментарий:
В единственном числе. В СССР, как известно, был только один президент.

Источник:
http://ru.wikipedia.org/wiki/Категория:Президенты_по_странам

Автор:
Дмитрий Кукулин (Тюмень)

Вопрос 3:
В продолжении известного советского произведения ЭТОТ ЧЕЛОВЕК,
походивший на тонкую изящную ложку или на скрипичный ключ, один заменил
нескольких безымянных диктаторов. Напишите имя этого человека.

Ответ:
Раздватрис.

Комментарий:
В уничтоженном Олешей продолжении "Трех толстяков" учитель танцев
Раздватрис должен был заменить толстяков, которых за неимением имен
называли Первым, Вторым и Третьим. Номер вопроса, кстати,
соответствующий.

Источник:
   1. http://www.proza.ru/addrec.html?2007/01/28-47
   2. http://aleksid.livejournal.com/8025.html
   3. Ю. Олеша. Три толстяка.

Автор:
Евгений Ярков (Тюмень)

Вопрос 4:
(pic: 20090825.jpg)
   Перед вами изображение, на котором мы скрыли четыре слова. Эти четыре
слова упоминает Тол Пенкейк, говоря об инопланетянине-шпионе. Назовите
эти четыре слова.

Ответ:
Не в своей тарелке.

Комментарий:
Рыба на карикатуре находится "не в своей тарелке". Инопланетянин-шпион
прокрался в чужую "тарелку", так что тоже находился не в своей тарелке.

Источник:
   1. http://2opena.livejournal.com/46443.html
   2. http://www.anekdot.ru/id/-2030601006/

Автор:
Евгений Ярков (Тюмень)

Вопрос 5:
Слон, носорог, буйвол, лев, леопард. Эти животные объединены символом. В
одном известном ряду этот символ идет 22-м, в другом - 23-м. Что, по
мнению Дэвида Ллойда и Алана Мура, значит этот символ?

Ответ:
Вендетта.

Комментарий:
Большая африканская охотничья пятерка: лев, слон, леопард, носорог и
буйвол. Охотники, которые официально охотились и убили этих животных,
имеют право носить на груди особый значок в форме латинской буквы "V". В
латинском алфавите V - 22-я буква, в периодической системе V - ванадий -
23-й элемент. Дэвид Ллойд и Алан Мур являются авторами комикса "V for
Vendetta" ("V значит вендетта").

Источник:
   1. http://ru.wikipedia.org/wiki/Большая_пятёрка
   2. http://www.mirf.ru/Reviews/review1931.htm

Автор:
Илья Марченко, Анна Рубцова (Вологда)

Вопрос 6:
Дореволюционная партия "Союз 17 октября" из-за своей программной
невнятности получила ехидное прозвище: "Союз ЕЕ". Назовите ЕЕ двумя
словами, если ОНА - это название русской повести 1831 года.

Ответ:
Пропавшая грамота.

Источник:
http://www.ej.ru/?a=note&id=8478

Автор:
Иван Беляев (Вологда)

Вопрос 7:
Экранизация этого романа была полна сюрпризов. Так, исполнителем одной
из главных ролей стал неугодный советской власти второсортный актер.
Посовещавшись, "верхушка" решила компенсировать нежелательного человека
народным артистом, чья твердая жизненная позиция не вызывала сомнений. А
еще, как ни странно, советская власть пощадила героиню романа.
Требование сохранить ей жизнь мотивировалось тем, что на волне всеобщей
печали производительность труда упадет, и выполнение плановых норм будет
под большой угрозой. Мы не просим вас назвать героиню. Напишите название
этого романа.

Ответ:
"Эра милосердия".

Комментарий:
В романе Вайнеров Вера Синичкина погибает, в фильме "Место встречи
изменить нельзя" - нет.

Источник:
Телепередача ""Место встречи изменить нельзя" двадцать лет спустя".

Автор:
Анастасия Шаркова (Тюмень)

Вопрос 8:
При поиске этого слова в Интернете автор вопроса получил ссылки на
статью в Википедии про реку Фьюмичино, на которую в XX веке было
снаряжено несколько экспедиций, на веб-страницу частной охранной
организации, на сайт учебного центра и на программу по преодолению
психологических проблем. Назовите это слово.

Ответ:
Рубикон.

Комментарий:
Гидрологическая экспедиция на Фьюмичино в 1933 году показала, что именно
эта река с наибольшей вероятностью являлась Рубиконом, в результате
этого реку переименовали обратно в Рубикон.

Источник:
   1. http://ru.wikipedia.org/wiki/Рубикон
   2. http://www.center-rubikon.ru/
   3. http://bb.ct.kz/index.php?showtopic=111011

Автор:
Гога Тариелашвили (Тбилиси)

Вопрос 9:
Если верить Плутарху, эта известная фраза была на самом деле произнесена
по-гречески, а не на родном языке автора. В фильме "Пираты Карибского
моря" эту фразу (только уже по-английски) говорит Билл Тернер своему
сыну, когда терпит поражение от капитана Дэви Джонса и навеки становится
рабом "Летучего голландца". Напишите эту фразу на русском языке.

Ответ:
Жребий брошен.

Комментарий:
Плутарх пишет, что фраза Цезаря "Жребий брошен" была сказана по-гречески
(&#945;&#957;&#949;&#961;&#961;&#953;&#966;&#952;&#969;
&#954;&#965;&#946;&#959;&#962; [анеррифто кибос]), а не по-латыни (Alea
Jacta Est). В фильме "Пираты Карибского моря" Билл Тернер проигрывает
Дэви Джонсу в кости.

Источник:
   1. http://en.wikipedia.org/wiki/Alea_iacta_est
   2. Фильм "Пираты Карибского моря".

Автор:
Гога Тариелашвили (Тбилиси)

Вопрос 10:
Зимой на спутнике Нептуна - Тритоне - температура настолько падает, что
ИКС замерзает в ИГРЕКЕ. На Земле содержание ИКСА в ИГРЕКЕ равно 78%.
Назовите ИКС и ИГРЕК.

Ответ:
Азот и атмосфера. Незачет: Азот и воздух (воздуха на Тритоне нет, только
атмосфера).

Источник:
http://en.wikipedia.org/wiki/Atmosphere_of_Triton

Автор:
Вахтанг Элердашвили (Тбилиси)

Вопрос 11:
В настоящее время в Интернете множество статей посвящено экономическому
кризису. К удивлению автора вопроса, заголовки многих посвященных этой
теме статей или были идентичны, или содержали одни и те же слова. Вот, к
примеру, подзаголовки опубликованной на сайте трибуна.ру статьи: "Плата
за стабильность", "Ньюсмейкер Греф и новая реальность", "Конец
кредитному раю?", "Утекающие миллиарды", "Рынок щупает дно", "Без
паники!". Назовите заголовок этой статьи, который представляет собой
фразу из популярной в России песни, которая первоначально была создана
на французском языке.

Ответ:
"А в остальном, прекрасная маркиза...".

Источник:
http://tribuna.ru/articles/2008/10/08/article4912/

Автор:
Темури Данелия (Тбилиси)

Вопрос 12:
Вот цитата из стихотворения Евгения Евтушенко, в котором мы сделали два
пропуска:
   Порою больно мне и горько это всё
   И силы нет сопротивляться вздору,
   И втягивает жизнь под колесо,
   Как [пропуск 1] втянул когда-то [пропуск 2].
   Заполните оба пропуска.

Ответ:
Шарф, Айседору.

Комментарий:
Речь идет об Айседоре Дункан.

Источник:
Е. Евтушенко. Письмо к Есенину.
http://antology.igrunov.ru/authors/evtush/esenin.html

Автор:
Иван Беляев (Вологда)

Вопрос 13:
24 января 1984 года 22-летний житель Баварии Клаус Фридрих установил
мировой рекорд, действие которого совершилось за 12 мин 57,3 сек, но не
эта продолжительность была рекордной. Подготовка к рекорду шла 31 день
(по 10 часов в день), но и не эта цифра была рекордной. Сам рекорд
исчислялся цифрами 281581 из 320236. Чем же занимался Клаус Фридрих на
протяжении 31 дня?

Ответ:
Складывал костяшки домино.

Комментарий:
Упало 281581 из 320236 костяшек после того, как толкнули первую кость.

Источник:
http://www.evolutsia.com/content/view/722/53/

Автор:
Темури Данелия (Тбилиси)

Вопрос 14:
На некой шхуне ИХ было столько же, сколько у фрегата и двух бригов.
Поэтому команда дала ИМ такие названия. Герои какого писателя
использовали аналогичные названия в целях конспирации?

Ответ:
Г.К. Честертон.

Комментарий:
На шхуне "Томас У. Лоусон" было семь мачт, названных по дням недели.
Честертон - "Человек, который был четвергом".

Источник:
В.П. Крапивин. Фрегат "Звенящий".

Автор:
Людмила Бабич (Казань)

Вопрос 15:
Джозеф Конрад в романе "Негр с "Нарцисса"" писал: "Это изобретение в
своем роде весьма остроумное. Доказательством служит уже хотя бы его
величина - нет другого предмета, столь несоразмерно малого по сравнению
с выполняемой им огромной задачей!". Назовите это одним словом.

Ответ:
Якорь.

Комментарий:
Джозеф Конрад - автор морских приключенческих романов.

Автор:
Людмила Бабич (Казань)

Вопрос 16:
Профессор Хьюберт Файнсворт - самый старый среди основных персонажей
мультсериала "Футурама". Одна из его отличительных черт - то, что он
всегда ходит в тапочках. Эта черта объединяет профессора с персонажем
другого мультсериала. Назовите этого персонажа.

Ответ:
Эйб Симпсон.

Зачет:
Дед из "Симпсонов" и подобные ответы по смыслу.

Источник:
http://ru.wikipedia.org/wiki/Футурама

Автор:
Илья Марченко (Вологда)

Вопрос 17:
[Ведущему: читать нарочито медленно.]
   Основанный в XVII веке шведским королем, этот университет долгое
время пользовался шведским языком в качестве официального. Позднее ему
на смену пришел немецкий. В XX веке из-за Первой мировой войны
университет временно эвакуировали в Воронеж. Какой язык является
официальным в этом университете сейчас?

Ответ:
Эстонский.

Комментарий:
Речь идет об одном из первых российских университетов - Дерптском, или
Тартуском, в городе Тарту, Эстония.

Автор:
Нелли Чижикова (Казань)

Вопрос 18:
Старейший футбольный клуб Шотландии - эдинбургский "Хартс" - основан в
1874 году. Из уважения к истории город предоставил клубу право отразить
этот факт в НЕМ, что клуб активно использует в маркетинговых целях.
Назовите ЕГО двумя словами.

Ответ:
Телефонный номер.

Комментарий:
У "Хартс" есть такая привилегия - пользоваться телефонным номером 1874.
   z-checkdb: Телефонный номер футбольного клуба "Хартс" - 0333 043 1874,
он состоит из одиннадцати цифр, что составляет стандартное для Шотландии
количество, см. http://www.heartsfc.co.uk/contact (Муроджон Хамраев).

Источник:
"Аэрофлот", октябрь 2008 г.

Автор:
???

Вопрос 19:
По одной из версий, его звали Василий, а ЭТО было прозвищем, означавшим
"казачий котел". По другой версии, ЭТО - производное от имени Герман.
Напишите ЭТО.

Ответ:
Ермак.

Источник:
Общие знания.

Автор:
???

Вопрос 20:
Испанский "Кабальеро" обосновался в русском языке как "Кавалер",
немецкий "Риттер" стал "Рыцарем". А вот "Витязь", по мнению историка
Покровского, это трансформировавшийся вариант другого слова. Какого?

Ответ:
Викинг.

Источник:
http://www.africana.ru/Sid/article/krest_torvaldsona.htm

Автор:
???

Вопрос 21:
ЕГО первый рекламный слоган - "Он круглый и длится долго". А вот
знаменитую эмблему - своеобразную ромашку, размещенную не сбоку, а
сверху, - создал сам Сальвадор Дали. О каком продукте идет речь?

Ответ:
Чупа-Чупс.

Комментарий:
Изначально их производила каталонская фирма.

Источник:
http://www.chupachups.ru/library/

Автор:
???

Вопрос 22:
   <раздатка>
   katya2@imperatrica.com - Екатерина II
   kurbsky@anti-ivan.lt - кн. Андрей Курбский
   ioannpii@vatikan.org - Кароль Войтыла
   etat@france.fr
   </раздатка>
   Перед вами вымышленные почтовые адреса известных личностей. Назовите
абсолютно точно человека, которому приписан адрес etat@france.fr.

Ответ:
Людовик XIV.

Зачет:
Король-Солнце. Незачет: Людовик.

Комментарий:
Людовику XIV приписывают фразу "Государство - это я".

Источник:
http://nemenandr.livejournal.com/404080.html

Автор:
Евгений Ярков (Тюмень)

Вопрос 23:
[Ведущему: "майя" прочитать как "мая".]
   Хоть ОН и появился в России 14 декабря 2006 года, статья о НЕМ на
одном сайте называется "С первым майя". Назовите ЕГО.

Ответ:
[Фильм] "Апокалипсис".

Источник:
http://www.film.ru/article.asp?id=4547

Автор:
Дмитрий Кукулин (Тюмень)

Вопрос 24:
Это произведение хорошо передает обстановку периода развала СССР.
Равнодушие общества к судьбе маленького человека, запутанное
международное положение, стремительная инфляция. И, пожалуй, главное:
безудержное стирание белых пятен нашей истории и открытие ужасной правды
о нашем прошлом. А с чем сравнивается человеческое общество в этом
произведении?

Ответ:
Муравейник.

Комментарий:
"А помрет, так помрет..., мы могли бы вести войну против тех, кто против
нас..., наши деньги не лезут в карман..., в нашем прошлом то ад, то
рай..., но от правды я тоже устал".

Источник:
Песня группы "Кино" "Муравейник".

Автор:
???

Вопрос 25:
Вопрос задает Джон Мильтон.
   Несмотря на то, что существование ИГРЕКА не доказано, многие из нас
хотели бы оказаться там. Поиск ИКСОВОГО ИГРЕКА давно уже занимает умы
человечества. Группа "Ария" считает, что найти ИКСОВЫЙ ИГРЕК можно, хотя
он находится довольно далеко, и даже указывает его примерное
местоположение. А вот группа "Кино" полагает, что ИКСОВЫЙ ИГРЕК
находится в другом месте. А где именно?

Ответ:
В наших глазах.

Комментарий:
ИКСОВЫЙ ИГРЕК - потерянный рай. "Далеко, там, где неба кончается край,
ты найдешь потерянный рай". "В наших глазах - потерянный рай". Джон
Мильтон - английский писатель и мыслитель эпохи Английской революции,
автор героической поэмы "Потерянный рай".

Источник:
Упомянутые произведения.

Автор:
???

Вопрос 26:
Статья в студенческой газете, посвященная соревнованиям по настольному
теннису между преподавателями и студентами исторического факультета,
которые не проводились уже много лет, называлась так же, как и
голливудский блокбастер. Только одну и ту же букву в названии фильма
изменили на другую. Напишите название статьи из двух слов.

Ответ:
"Пинг-понг жив".

Комментарий:
Много лет не проводились; аллюзия на вторую часть "Кинг-конга".

Источник:
Газета "ДИФ" истфака КГУ.

Автор:
???

Вопрос 27:
Как известно, римляне, заимствовав в Греции строительные формы, стали
возводить здания по греческим образцам. По мнению Петра Гнедича,
император Август смело мог сказать умирая: "Я получил город [первый
пропуск], а оставляю его [второй пропуск]". Восстановите оба пропуска в
правильном порядке.

Ответ:
"... кирпичным...", "... мраморным".

Источник:
П.П. Гнедич. История искусств. - С. 171.

Автор:
???

Вопрос 28:
После того как Эстебан Мурильо из-за болезни не смог работать, он часами
простаивал перед картиной, изображающей "Снятие с креста", и нередко
впадал в мистическую экзальтацию. Однажды ключарь собора спросил, долго
ли он останется в соборе. На это художник ответил: "Пока...". Закончите,
хотя бы приблизительно, ответ художника.

Ответ:
".... совсем не снимут с креста его тело".

Зачет:
По смыслу.

Источник:
П.П. Гнедич. История искусств. - С. 470.

Автор:
???

Вопрос 29:
Прослушайте стихотворные строчки Сергея Мавроди, который стал
классическим примером в истории финансовых пирамид, но который вряд ли
внесет свой вклад в классическое искусство.
   Боже, боже, боже, боже мой,
   Ну за что?
   Почему?
   Как же, как же, как же, как же так?
   Я уже
   Не пойму.
   Где же, где же, где же, где же вы,
   Те надежды,
   Мечты?
   Вместе, вместе, вместе, вместе быть!
   Лишь вдвоем -
   Я и ты!
   К кому обращался автор произведения?

Ответ:
К Элизе.

Зачет:
К Терезе.

Комментарий:
Этот текст положен на музыку фортепианного произведения Бетховена,
известного всем именно под таким названием.
   Существует версия, согласно которой оно было посвящено Терезе
Малфатти, но при наборе утерянной ныне рукописи в типографии не смогли
разобрать отвратительный почерк композитора. Поэтому ответ "К Терезе"
тоже следует принимать.

Источник:
http://sergeymavrodi.narod.ru/Bethoven_Elize.html

Автор:
???

Вопрос 30:
Мало кто знает, что ЭТО - ягода. Вот уже сотни лет количество любителей
ЭТОГО увеличивается с каждым днем. ЭТО стало известно в Европе лишь в
XVI веке, а изначально было популярно среди проживающих на Ближнем
Востоке. Порой врачи прописывали ЭТО больным как лекарство, а иногда и
вовсе запрещали. Бетховен, большой любитель ЭТОГО, был таким
скрупулезным, что всегда пересчитывал все 60 ягод, когда готовил ЭТО.
Назовите ЭТО.

Ответ:
Кофе.

Комментарий:
Да, кофе - это ягода.

Источник:
http://www.fandv.ru/cofee

Автор:
Команда "ЛОМ" (Калининград)

Тур:
6 тур

Вопрос 1:
На одном из островов в Тихом океане, Бугенвиле, используют ЕГО,
состоящего из 11 элементов. Пять из них относятся к ПЕРВЫМ, а другие
шесть - ко ВТОРЫМ. Мы же привыкли к десяти ПЕРВЫМ и к двадцати одной
ВТОРОЙ. Мы не просим вас назвать ПЕРВЫЕ и ВТОРЫЕ. Назовите ЕГО.

Ответ:
Алфавит.

Комментарий:
На острове Бугенвиле самый короткий алфавит - всего 11 букв, из них 5
гласных (а, э, и, о, у) и 6 согласных (б, г, к, п, р, т).

Источник:
http://www.fandv.ru/alphabet

Автор:
Команда "ЛОМ" (Калининград)

Вопрос 2:
   <раздатка>
   A-a-a, amnes in [...] amplissimi,
   A-a-a, montes in [...] altissimi
   </раздатка>
   Перед вами часть припева популярной песни, переведенной на латынь.
Какой топоним мы дважды пропустили в этом тексте?

Ответ:
Африка.

Зачет:
Africa.

Комментарий:
"В Африке реки вот такой ширины, в Африке горы вот такой вышины". Это
песенка из фильма про Красную Шапочку.

Источник:
http://www.binetti.ru/artes/poesia/carmina_russica.shtml

Автор:
Евгений Ярков (Тюмень)

Вопрос 3:
Существует множество народных поверий о НИХ. Лучшим средством борьбы с
НИМИ является растение, название которого переводится с кельтского языка
как "лук" и "копье". ИХ и один вид животного, обитающий в водной среде,
называют одним и тем же существительным. Назовите ИХ.

Ответ:
Вампиры.

Комментарий:
По множеству поверий, в том числе и славянским, средство борьбы с
вампирами - чеснок, являвшийся обязательным продуктом в рационе любого
кельта. Вампиров и пиявок часто называют кровопийцами.

Источник:
   1. Брэм Стокер. Дракула.
   2. http://ru.wikipedia.org/wiki/Дракула
   3. http://ru.wikipedia.org/wiki/Вампиры
   4. http://www.geohistory.ru/lifestyle/chesnok-dlya-draculi/

Автор:
Команда "ЛОМ" (Калининград)

Вопрос 4:
[Ведущему: непременно "Трэффорд", а не "Траффорд".]
   Бомбежки промышленных районов северной Англии в 1940-1941 годах
достигали невероятных объемов. Так, в ходе налетов на "Трэффорд Парк" -
крупнейшую промзону Европы того времени - количество немецких
бомбардировщиков, одновременно совершавших боевой вылет, иногда доходило
до 270. Масштаб разрушений был под стать: кроме многочисленных
предприятий, производивших военную технику (заводов "Виккерс" и "Форд"),
пострадали и множество гражданских зданий в располагавшихся поблизости
городах: Кафедральный собор, Королевский театр, мюзик-холл и Сэлфордский
госпиталь получили серьезные повреждения. Кроме того, жертвой авиаударов
стало еще одно вполне мирное сооружение, имевшее несчастье располагаться
в том же районе. Назовите "несчастливца".

Ответ:
"Old Trafford".

Зачет:
"Олд Траффорд", "Олд Трэффорд" и т.д.

Комментарий:
В "Трэффорд Парке" находится здоровенная по площади штуковина - домашняя
арена "Манчестер Юнайтед", носящая имя этой самой промзоны. Она и была
задета немецкими снарядами 11 марта 1941 года.

Источник:
   1. http://ru.wikipedia.org/wiki/Олд_Траффорд
   2. http://en.wikipedia.org/wiki/Trafford_Park
   3. http://en.wikipedia.org/wiki/Manchester_Blitz

Автор:
???

Вопрос 5:
Большая политика очень часто вмешивается в большой спорт. Иногда даже
нарушаются главные законы Олимпиады. Именно поэтому Хатуне Лориг,
которая не стала даже призером соревнований, доверили нести флаг США на
церемонии закрытия Олимпийских Игр в Пекине. К этому факту можно также
приплести и национальный вопрос, поднятый в прошлом году одним известным
политиком-неудачником. А почему именно Хатуне Лориг доверили знамя?

Ответ:
Уроженка Грузии.

Зачет:
По смыслу.

Комментарий:
Хатуна является уроженкой Грузии и выступала на Олимпийских играх 1992,
1996 и 2000 годов, причем первую Олимпиаду она провела под знаменами
Объединенной команды (де-факто сборной СНГ), где даже получила бронзовую
медаль, а игры в Атланте и Сиднее провела в сборной Грузии, но уже
безмедально. В 2005 году она приняла гражданство США и на Олимпиаде 2008
года выступила в соответствующей сборной. Джон Маккейн, проведший долгое
время во вьетнамском плену, сказал в августе 2008 года: "Сегодня мы все
грузины".

Источник:
http://www.alexrus.info/post83384205/

Автор:
???

Вопрос 6:
В Корее ОН известен с XIV века. ОН начинался в 10 часов вечера и
заканчивался в 4 утра. Наказание за ЕГО нарушение - от 10 до 40 ударов
палками. ОН был отменен в 1895 году, вновь введен в 1945 году и
действовал вплоть до конца XX века. Назовите ЕГО двумя словами, если его
ввели в двух российских областях в 2008 году.

Ответ:
Комендантский час.

Автор:
Олег Яшкин (Калининград)

Вопрос 7:
Анекдот.
   Случай в театре.
   Один режиссер: Вы, Мария Ивановна, будете играть героиню романа Л.Н.
Толстого.
   Второй режиссер: Нет уж, извольте. Мария Ивановна исполнит роль в
"великой трагедии" У. Шекспира.
   Мария Ивановна: Знаете, господа, я девушка свободная и сама имею
право выбирать, быть мне [ПРОПУСК] или [ПРОПУСК].
   Заполните пропуски двумя причастиями страдательного вида.

Ответ:
"... задавленной...", "... задушенной".

Комментарий:
Роман Толстого - "Анна Каренина", трагедия Шекспира - "Отелло".

Автор:
Евгений Долгирев

Вопрос 8:
Первый российский генерал-губернатор, по одной из версий, использовал ИХ
при исполнении своих обязанностей. ОНИ - два слова. С первым из них вы
можете встретиться на отдыхе, а второе - название одного
астрономического объекта. Назовите ИХ.

Ответ:
Песочные часы.

Автор:
???

Вопрос 9:
3776 метров, 10 этапов, 12 часов. Неотъемлемой частью описанного
процесса является посох с НИМИ. ОНИ на посохе выполняют утилитарную
функцию. Назовите то, с чем упоминались ОНИ в известном русском романсе.

Ответ:
Тройка.

Источник:
Вс. Овчинников. Ветка сакуры. - М.: Молодая гвардия, 1971.

Автор:
???

Вопрос 10:
Прозвище каждого из этих двух людей состояло из двух слов. Первое слово
у обоих - прилагательное, обозначающее один из основных цветов. Вторые
слова их прозвищ в какой-то мере характеризуют особенность их
профессиональных амплуа. Первый - один из символов Португалии, хотя
родился в Африке. Недавно ему при жизни установили монумент. Второй
весной 1943 года пошел работать на завод учеником слесаря, но снискал
себе славу на совершенно ином поприще. В жизни они были большими
друзьями, хоть и часто огорчали друг друга. Мы не просим вас назвать их
прозвища. О ком идет речь?

Ответ:
Эйсебио и [Лев] Яшин.

Комментарий:
Черная Пантера и Черный Паук - прозвища, соответственно, Эйсебио и Льва
Яшина. За искусство обращения с мячом и за мастерство ловить мячи и
парировать коварные удары.

Источник:
   1. http://www.sport-express.ru/art.shtml?86149
   2. http://www.footballnotes.ru

Автор:
Евгений Потапов (Киров)

Вопрос 11:
После невразумительной и скучной игры против Азербайджана в присутствии
80 тысяч болельщиков, ожидавших увидеть яркую и искрометную игру, на
сборную России обрушился шквал критики. В интервью полузащитник сборной
России Константин Зырянов указал на то, что сборная добилась-таки
приемлемого результата при блеклой игре, и перефразировал известное
высказывание римского императора Веспасиана. Напишите это измененное
высказывание.

Ответ:
Три очка не пахнут.

Зачет:
Очки не пахнут и т.п. по смыслу.

Комментарий:
Победа есть победа, за нее дают три очка, не важно, как она добыта.

Источник:
http://www.championat.ru/football/article-31039.html

Автор:
Евгений Потапов (Киров)

Вопрос 12:
Как известно, римские императоры отличались своеобразным чувством юмора.
Циничный Нерон как-то в беседе заявил: "Любимая пища богов - это грибы".
А какой римский бог, по его мнению, служил бесспорным доказательством
этого?

Ответ:
[Император] Клавдий.

Зачет:
Предшественник Нерона.

Комментарий:
Как известно, предшественник Нерона император Клавдий был отравлен
грибами. По обычаю, после смерти Клавдия объявили богом.

Источник:
Светоний. Нерон. 33.1. По изданию: Светоний Гай Транквилл. Жизнь
двенадцати цезарей. / Перев. с лат. и примеч. М.Л. Гаспарова. - М.:
Ладомир, 1999. - 944 с. - С. 210.

Автор:
Денис Микшис (Тюмень)

Вопрос 13:
Внимание, в вопросе есть замена.
   Японская пословица гласит: "И ягненок может проговориться". Словом
"ягненок" мы заменили неодушевленный предмет, являющийся частью названия
города на реке Обь и названием вулкана на Камчатке. У Александра
Сокурова есть фильм с одноименным названием, а у Осипа Мандельштама -
стихотворный сборник. Назовите слово, которое мы заменили.

Ответ:
Камень.

Источник:
   1. http://www.anime.dvdspecial.ru/Japan/sayings.shtml
   2. http://ru.wikipedia.org/wiki/Камень

Автор:
Команда "Гладиолус" (Киров)

Вопрос 14:
В 2007 году журнал "F1 Racing" написал о ситуации в команде "McLaren",
где между Фернандо Алонсо и Льюисом Хэмилтоном разгорелась нешуточная
конкуренция. В заголовке статьи, являющемся известной фразой из
кинофильма, кроме двоих, фигурировало и одно животное. Назовите кличку
этого животного.

Ответ:
Боливар.

Комментарий:
Статья называлась "Боливар не выдержит двоих". Фраза - из отечественного
фильма "Деловые люди" по мотивам новелл О. Генри.

Источник:
   1. http://www.f1news.ru/memuar/2007/mclaren.shtml
   2. http://ru.wikipedia.org/wiki/Деловые_люди

Автор:
Алексей Усанов (Киров)

Вопрос 15:
Слово "лешассе'а" действительно переводится как "ДЕЛАТЬ ЭТО", но совсем
с другой стороны, нежели изображает известная статуя. Так что на самом
деле поступок героя куда более груб. Назовите этого героя.

Ответ:
Самсон.

Комментарий:
Этот глагол встречается в Пятикнижии в значении "разорвать сзади на две
части". Таким образом, выходит, что Самсон порвал льву совсем другое
место.

Источник:
http://ru.wikipedia.org/wiki/Самсон

Автор:
Николай Коврижных (Киров)

Вопрос 16:
В 2001 году эстонский политик Каарел Таранд внес предложение сменить ИКС
на ИГРЕК. Его сторонники утверждают, что ИКС придает Эстонии образ
постсоветской и восточноевропейской страны, тогда как ИГРЕК должен
символизировать тесную связь с северными странами. Назовите ИКС.

Ответ:
Триколор.

Зачет:
Флаг, флаг Эстонии и т.п. по смыслу.

Комментарий:
Они предлагают изменить триколор на скандинавский крестовый флаг с теми
же самыми цветами.

Источник:
http://ru.wikipedia.org/wiki/Флаг_Эстонии

Автор:
Николай Коврижных (Киров)

Вопрос 17:
(pic: 20090826.jpg)
   Перед вами прибор, в который должна вставляться ОНА. А один
программист однажды увидел, как из-под окна вылезла и вонзилась когтями
в землю исполинская ОНА, провела в траве глубокие борозды и снова
скрылась. Назовите ЕЕ двумя словами.

Ответ:
Куриная нога.

Зачет:
По смыслу.

Комментарий:
Этим столовым прибором держали куриную ногу. Программист Александр
Привалов из "Понедельник начинается в субботу" был внутри Избушки на
куриных ногах.

Источник:
   1. http://2opena.livejournal.com/131294.html
   2. http://lib.ru/STRUGACKIE/ponedelx.txt

Автор:
Евгений Ярков (Тюмень)

Вопрос 18:
Легенда связывает ЕГО происхождение с гвоздем и веревкой, которые герцог
Альба обещал "в награду" каждому голландскому гёзу. По другой версии,
своим появлением ОН обязан необходимости постоянно записывать
распоряжения вышестоящего руководителя. Из поговорки мы знаем, что ОН
является непременным атрибутом лиц, занимающих определенную должность.
Назовите эту должность.

Ответ:
Адъютант.

Комментарий:
Согласно армейской поговорке, "нет адъютанта без аксельбанта".
Аксельбант - украшение в виде плетеного шнурка с наконечниками в виде
грифелей. Эти наконечники, по одной версии, являлись грифелями для
записи распоряжений командующего. По другой версии, принц Вильгельм
Оранский наградил своих адъютантов такими украшениями в знак презрения к
угрозе Альбы повесить всех гёзов.

Источник:
   1. http://gorod.crimea.edu/librari/rus21812end/str_28.htm
   2. http://www.waronline.org/IDF/Articles/alget/index.html
   3. http://www.maraby.net/quote/95391/

Автор:
Денис Микшис (Тюмень)

Вопрос 19:
День ЭТОГО отмечается 31 августа. Дату выбрали, переведя в цифры...
Какое слово?

Ответ:
Blog.

Зачет:
Блог.

Комментарий:
Буквы слова "Blog" напоминают соответствующие цифры числа "3108", то
есть 31 августа.

Источник:
http://ru.wikipedia.org/wiki/День_блога

Автор:
Николай Коврижных (Киров)

Вопрос 20:
ИКС - имя героини в заголовке русского классического литературного
произведения, ИГРЕК - известный соковый брэнд, ЗЕТ - то, что, согласно
известному выражению, точно не способствует появлению умных мыслей. ИКС
ИГРЕКЗЕТ - так звали главного персонажа отечественного фильма 1962 года.
Назовите этот фильм.

Ответ:
"Королева бензоколонки".

Комментарий:
ИКС - Людмила ("Руслан и Людмила" А.С. Пушкина), ИГРЕК - сок "Добрый",
ЗЕТ - вечер ("утро вечера мудренее"). ИКС ИГРЕКЗЕТ - Людмила
Добрыйвечер.

Источник:
http://ru.wikipedia.org/wiki/Королева_бензоколонки

Автор:
Алексей Усанов (Киров)

Вопрос 21:
[Ведущему: прочитать "Умный запятая добрый запятая смелый".]
   Он родился в 1917 году, а умер в 2001 году. В титрах к некоторым
фильмам его фамилия написана через "Ы", хотя нам привычнее "И". Статья в
газете "Аргументы и факты", рассказывающая о нем, называлась "Умный,
добрый, смелый...". Закончите название коротким словом.

Ответ:
Трус.

Комментарий:
Речь идет о Георгии Вицине.

Источник:
   1. http://www.peoples.ru/art/cinema/actor/vitsin/index.html
   2. http://www.peoples.ru/art/cinema/actor/vitsin/history5.html

Автор:
Дмитрий Кукулин (Тюмень)

Вопрос 22:
Это имя было выбрано по той причине, что оно ассоциируется у жителей
России с высшими достижениями, а сам ОН - является символом нации. А
теперь назовите двумя словами предмет массой 19 кг, сделанный из серебра
925-й пробы.

Ответ:
Кубок Гагарина.

Источник:
http://ru.wikipedia.org/wiki/Кубок_Гагарина

Автор:
???

Вопрос 23:
Журналиста ОРТ Романа Перевезенцева, работника компании "Грознефтегаз"
Ильяса Магомадова и подполковника Олега Стрюкова можно было назвать ИМИ.
А вы назовите имя и фамилию человека, который в 1996 году прославился в
качестве НЕГО, а в 2002 году, так сказать, остался ИМ навсегда.

Ответ:
Сергей Бодров.

Комментарий:
Перечислены люди, которых похищали в Чечне. Сергей Бодров, прославившись
в 1996 году как "Кавказский пленник", остался им навсегда, когда погиб в
Кармадонском ущелье из-за лавины.

Источник:
   1. http://www.videoguide.ru/card_film.asp?idFilm19323
   2. http://www.webtelek.com/news.php?url/vojna/2002/05/07/arrest/
   3. http://www.rnews.ru/view/society/194920/
   4. http://www.dni.ru/news/russia/2002/5/20/9883.html

Автор:
Дмитрий Кукулин (Тюмень)

Вопрос 24:
В книге "Русская свадьба" авторы пишут, что русский свадебный обряд для
девушки - это метафора смерти в ее роду и нового рождения в роду мужа.
Например, внос в дом на руках невесты женихом - это попытка обмануть
домового. За кого таким образом пытаются заставить домового принять
невесту?

Ответ:
За новорождённого.

Зачет:
По смыслу.

Комментарий:
Только новорождённый в дом не входил, а в доме оказался.

Источник:
   1. Д.М. Балашов, Ю.И. Марченко, Н.И. Калмыкова. Русская свадьба.
   2. http://mi3ch.livejournal.com/1483558.html

Автор:
???

Вопрос 25:
В книге Бориса Акунина "Квест" один из персонажей - американский
миллиардер. Фамилия его лишь на одну букву отличается от породы собак.
Напишите эту фамилию.

Ответ:
Ротвеллер.

Комментарий:
Акунин скрестил фамилии двух наиболее известных миллиардеров - банкира
Ротшильда и нефтяного магната Рокфеллера. Порода - ротвейлер.

Источник:
Борис Акунин. Квест.

Автор:
???

Вопрос 26:
Издательство "Флюид" готовит к изданию в России две ЕГО пьесы. Главный
редактор издательства говорит, что "содержание этих пьес рассказать
невозможно, это абсолютно сюрреалистические сюжеты, шизуха со странными
действующими лицами". Период сюрреализма в другом, основном направлении
ЕГО творчества наступает, отчасти, под влиянием поэтов-сюрреалистов в
1925 году. Назовите ЕГО.

Ответ:
[Пабло] Пикассо.

Комментарий:
Описание пьес очень напоминает содержание картин Пикассо.

Источник:
   1. http://www.gzt.ru/culture/2009/01/26/160755.html
   2. http://ru.wikipedia.org/wiki/Пикассо,_Пабло

Автор:
???

Вопрос 27:
В доме своего отца Авраама ОН постоянно руководил воскресными
музыкальными вечерами, на которых собиралась элита Берлина. При этом он
мог следить за впечатлением слушателей и зачастую по их совету
избавлялся от ЭТОГО. Так вышло, что сейчас одно из ЕГО произведений
тесно связано с ЭТИМ. Назовите это произведение.

Ответ:
Свадебный марш.

Зачет:
Марш Мендельсона.

Комментарий:
В вопросе куча подсказок: Авраам, музыка, Берлин, "вышло" (выйти замуж),
"тесно связано" (связать узами брака), ну и брак. Он - Якоб Людвиг
Феликс Мендельсон (Бартольди).

Источник:
Н.Л. Волковский. Историческая портретная галерея. - Статья "Мендельсон".

Автор:
???

Вопрос 28:
(pic: 20090827.jpg)
   Перед вами приспособление, в которое мужчины начала XX века
вкладывали ЕГО. ОНО является названием советского произведения. Назовите
ЕГО двумя словами.

Ответ:
Лезвие бритвы.

Комментарий:
Этот прибор использовался для заточки бритвенных лезвий. "Лезвие бритвы"
- фантастическое произведение Ивана Ефремова.

Источник:
   1. http://2opena.livejournal.com/134188.html
   2. И. Ефремов. Лезвие бритвы.

Автор:
Евгений Ярков (Тюмень)

Вопрос 29:
БЛИЗНЕЦЫ являются единственными неодушевленными в своем роде. По одной
из версий название "БЛИЗНЕЦЫ" связано с одним сентябрьским днем, который
считался точкой смерти Природы. Какое слово мы заменили на "БЛИЗНЕЦЫ"?

Ответ:
Весы.

Комментарий:
Весы - единственный знак Зодиака, представляющий неодушевленный предмет.
Название связано с днем осеннего равноденствия. Замена, думаю, ясна.

Источник:
Л.С. Баешко, А.Н. Гордиенко, А.Н. Гордиенко. Энциклопедия символов. -
Статья "Весы".

Автор:
???

Вопрос 30:
В фильме 1995 года действие происходит в 2053 году. Герой смотрит
современный фильм "Король Лир" и спрашивает у своей знакомой, что за
старик играет в нем главную роль. В ответ она называет реального актера,
прославившегося благодаря фильму "СКОЛЬКО ГДЕ". Автор вопроса не знает,
славянские ли корни у этого актера. Тем не менее, скажите, что мы
заменили словами "СКОЛЬКО ГДЕ".

Ответ:
Один дома.

Зачет:
Home alone.

Комментарий:
Фильм снят в 1995-м - значит, искать актеров нужно где-то раньше. Раз в
2053-м он старик - значит, в те годы должен быть совсем молодым. Отсюда
и ответ. Старик - Маколей Калкин (намек на славянское происхождение
фамилии в вопросе), знаменитейший фильм - "(сколько?) Один (где?) дома".

Источник:
Фильм "Гаррисон Бержерон".

Автор:
???

Тур:
7 тур

Вопрос 1:
Согласно труду Плутарха, ОН пришелся на праздник доброй Богини, когда
молодой юноша Клодий, переодевшись женщиной, пробрался в храм накануне
полночных таинств. Другой ОН характеризуется обильной игрой светотени.
Назовите его тремя словами.

Ответ:
Последний день Помпеи.

Комментарий:
В первом случае речь идет о жене Цезаря Помпее, которая должна быть выше
подозрений. А другой "Последний день Помпеи" - картина Карла Брюллова.
   z-checkdb: После описанного случая Цезарь развелся с женой, но она
еще долго прожила и даже снова вышла замуж (Антон Губанов).

Источник:
   1. Плутарх. Сравнительные жизнеописания.
   2. http://ru.wikipedia.org/wiki/Последний_день_Помпеи

Автор:
Евгений Магадеев (Уфа)

Вопрос 2:
Статья о депрессии в одном психологическом журнале называлась "Проза
жизни, или Лекарство от скуки". В одном из слов этого названия мы
пропустили одну букву. Напишите это слово в исходном варианте.

Ответ:
Прозак.

Комментарий:
Прозак - популярный антидепрессант.

Источник:
http://medportal.ru/encyclopaedia/psychology/deepdejection/6/

Автор:
Александр Мальцев (Уфа)

Вопрос 3:
Началом работы Уфимского водопровода считается 26 июня 1901 года. Однако
уже в апреле 1901 года начался подвоз запаса дров на территорию
водопроводной станции. Назовите профессию человека, первым начавшего
работать на Уфимском водопроводе.

Ответ:
Сторож.

Зачет:
Охранник и т.п. по смыслу.

Комментарий:
Дрова необходимо было охранять. Собственно, сторож - первая профессия,
упомянутая в Библии.

Источник:
Г.А. Бурдыгин. Сто лет Уфимскому водопроводу.

Автор:
Владислав Виноградов (Уфа)

Вопрос 4:
Перед матчем на первенство мира по шахматам между Ботвинником и Талем
Василий Панов остроумно заметил, что, несмотря на сложность предматчевых
прогнозов, одно можно предсказать совершенно точно: победит... Закончите
цитату одним словом.

Ответ:
Михаил.

Комментарий:
И Ботвинник, и Таль - Михаилы.

Источник:
В. Панов. 40 лет за шахматной доской.

Автор:
Максим Мерзляков (Воронеж)

Вопрос 5:
Внимание, речь в вопросе идет о реально существующем острове!
   Остров Джерба, который находится в Средиземном море, в 2 км от
побережья Туниса, славится своей богатой флорой. На острове
распространена Nymphaea caerulea, цветы которой обладают психотропными
свойствами. Назовите второе, хорошо нам знакомое, название растения
Nymphaea caerulea.

Ответ:
Лотос.

Комментарий:
Ученые считают, что описанный в "Одиссее" мифический остров, на котором
обитали лотофаги, существовал на самом деле и это - остров Джерба. По
"Одиссее", люди забывали прошлое, если ели цветы лотоса.

Источник:
http://www.vokrugsveta.ru/quiz/515/

Автор:
Ираклий Назгаидзе (Тбилиси)

Вопрос 6:
В XVIII веке ботаник Христиан Буксбаум открыл мох буксбаумию. Поскольку
называть вид в свою честь считалось некрасивым, ботаник дал ему имя в
честь ИКСА. Известен случай, когда другой ИКС был представлен в виде
листа. Какого растения?

Ответ:
Клевера.

Комментарий:
Хитрый Христиан Буксбаум назвал мох в честь своего отца. Святой Патрик в
виде трех листьев клевера представил Святую Троицу, в т.ч. и Бога-отца.

Источник:
Л.В. Бардунов. Древнейшие на суше. - Новосибирск: Наука, 1984.

Автор:
Надия Шафигуллина (Казань)

Вопрос 7:
Из-за его популярности в оркестровых концертных программах и
телевизионных заставках, музыканты называют этот вальс "измочаленным".
Какое слово мы заменили в этом вопросе?

Ответ:
Изметеленный.

Комментарий:
Литературно-музыкальный цикл "Метель" (который включает и знаменитый
вальс) Свиридов написал к фильму "Метель".

Источник:
Музыкантские байки.

Автор:
Людмила Бабич (Казань)

Вопрос 8:
По словам Андрея Медведева, ОН не для зайца. Существует московский и
ленинградский способы ЕГО определения. Назовите ЕГО двумя словами.

Ответ:
Счастливый билет.

Комментарий:
"Счастливость" билета можно определить несколькими методами. Московский
- если на автобусном билете напечатано шестизначное число и сумма первых
трех цифр равна сумме последних трех, то этот билет считается
счастливым. Ленинградский, или питерский (менее распространенный), -
если сумма четных цифр билета равна сумме нечетных цифр билета, то билет
считается счастливым.

Источник:
http://ru.wikipedia.org/wiki/Счастливый_билет

Автор:
Зульфия Бурганова (Казань)

Вопрос 9:
Прослушайте отрывок из стихотворения.
   Жучка за мышку, бабка за жучку...
   И никогда не закончится это.
   Внучка за бабку, дедка за внучку...
   В четвертой строке этого стихотворения упоминается слово итальянского
происхождения. Напишите это слово.

Ответ:
Вендетта.

Комментарий:
"Что за жестокий обычай - вендетта".

Источник:
Стихотворение Г. Левитина.

Автор:
Анна Терентьева (Казань)

Вопрос 10:
Владимир Маяковский в одном из своих лозунгов для журнала "Даешь" писал
(внимание, стихотворная форма с пропусками!):
   Профессорская братия вроде Ольденбургов
   князьям служить и сегодня рада.
   То, что годилось для царских ИКСОВ,
   мы вырвем с корнем из красных ИГРЕКОВ.
   ИКСЫ и ИГРЕКИ - имена собственные во множественном числе. Назовите их
в правильном порядке.

Ответ:
Петербурги, Ленинграды.

Комментарий:
Никаких комментариев, всё и так понятно.

Источник:
http://mayakovsky-reklama.narod.ru/slogany6.htm

Автор:
???

Вопрос 11:
В начале XXI века один из этих двоих в своей торжественной речи шутил,
что питает любовь к низким температурам еще с суровой военной зимы 1942
года. Назовите обоих.

Ответ:
[Виталий] Гинзбург и [Алексей] Абрикосов.

Комментарий:
Получили Нобелевскую премию по физике 2003 года за вклад в развитие
теории сверхпроводимости и сверхтекучести.

Источник:
http://ru.wikipedia.org/wiki/Гинзбург,_Виталий_Лазаревич

Автор:
Николай Коврижных (Киров)

Вопрос 12:
[Ведущему: отточие паузой не выделять.]
   Прослушайте цитату. "Слово ... "Коминтерн" напоминает всего лишь о
крепко спаянной организации и жесткой системе доктрин. "Коминтерн" - это
слово, которое можно произнести, почти не размышляя, в то время как
"Коммунистический Интернационал" заставляет пусть на миг, но
задуматься". О принципах какого языка рассуждал в приложении к своему
роману автор этой цитаты?

Ответ:
Новояз.

Зачет:
Newspeak.

Комментарий:
Приведена цитата Джорджа Оруэлла (Принципы новояза. Приложение к
"1984").

Автор:
???

Вопрос 13:
Бросив беглый взгляд на название одного ижевского магазина, автор
вопроса решил, что там продают цифровую технику с хорошей оптикой. И
только прочитав название второй раз, он понял, что там больше шансов
купить не "Шарп", а "Чаппи". Напишите первую буку названия магазина.

Ответ:
Z.

Комментарий:
Магазин назывался "ZOOМИР".

Автор:
???

Вопрос 14:
Лирическая героиня Агнии Барто, устав от белых сугробов, использовала
ЭТО. ЭТО ЖЕ, только в двойном количестве, использовали жители и гости
города в центре одной страны. Какой именно страны?

Ответ:
Изумрудной.

Зачет:
Оз.

Комментарий:
Стихотворение Агнии Барто:
   Я знаю, что надо придумать, чтоб не было больше зимы,
   Чтоб вместо высоких сугробов вокруг зеленели холмы.
   Смотрю я в стекляшку зелёного цвета,
   И сразу зима превращается в лето.
   А жители Изумрудного города носили зеленые очки (в каждых - по две
стекляшки). Страна называлась так же, как и город: "Изгороди и дома были
выкрашены в красивый ярко-зеленый цвет, и люди носили зеленую одежду. -
Это значит, что началась Изумрудная страна, - сказал Железный Дровосек".

Автор:
Евгений Ткаченко (Тюмень)

Вопрос 15:
По версии журналистов "Новых известий", христианскому святому Кассиану
Римлянину просто не повезло: на Руси 29 февраля вспоминали злое божество
славянского пантеона, и образы Кассиана и этого божества смешались. Что
нужно было раздавить в ночь с 29 февраля на 1 марта, мы спрашивать не
будет. А имя упомянутого божества назовите.

Ответ:
Кащей.

Зачет:
Кощей.

Комментарий:
Вспоминали Кащея (который еще и звучит похоже на Кассиана). Смерть его
скрывается в курином яйце. А заодно и "смерть" зимы.

Автор:
Евгений Ткаченко (Тюмень)

Вопрос 16:
Весной 2008 года на похоронах легенды шотландского "Селтика" Томми
Бернса рядом с могилой была замечена футболка с надписью, явно сделанной
ребенком: "Святой Томми, ты сейчас уже на небесах. Пожалуйста, передай
привет остальным святым. Особенно [слово пропущено]". Заполните пропуск
вовсе не именем святого.

Ответ:
Санкт-Петербургу.

Зачет:
Петербургу.

Комментарий:
Фанаты "Селтика" враждуют с фанатами "Глазго Рейнджерс". Питерский
"Зенит" в мае 2008 года в финале кубка УЕФА обыграл "Глазго Рейнджерс".

Автор:
Евгений Ткаченко (Тюмень)

Вопрос 17:
[Ведущему: в первом случае читать "ИКСА, запятая, ИГРЕКА, запятая,
ЗЕТА".]
   Кстати, о Петербурге. Екатерина Андреева назвала Санкт-Петербург
городом ИКСА, ИГРЕКА, ЗЕТА. ИКС, ИГРЕК и ЗЕТ по отдельности несмотря на
неполноту информации у многих прочно ассоциируются с тремя конкретными
личностями. В то же время ИКС ИГРЕК ЗЕТ могут обозначать и одного
человека. Найдите ИГРЕК.

Ответ:
Ильич.

Комментарий:
Город Петра, Ильича, Чайковского.

Автор:
Артем Кушнир (Тюмень)

Вопрос 18:
По одной легенде, шведы, увидев подходившую к острову русскую яхту в
сопровождении галиота, поспешно скрылись. Они даже не успели забрать
котел с едой. Какой город находится сейчас на этом острове?

Ответ:
Кронштадт.

Комментарий:
От слова "котел" якобы происходит название острова Котлин, где
впоследствии был основан Кронштадт.

Автор:
Евгений Ткаченко (Тюмень)

Вопрос 19:
(pic: 20090828.gif)
   Ответьте двумя словами, название какой статьи в Электронной
энциклопедии Кирилла и Мефодия мы от вас закрыли.

Ответ:
"Лед Зеппелин".

Автор:
Евгений Ткаченко (Тюмень)

Вопрос 20:
Какой крылатой фразой создатели телепрограммы "Главный герой" назвали
совместный проект группы "Уматурман" и Патрисии Каас?

Ответ:
Смесь французского с нижегородским.

Комментарий:
Нижегородская группа "Уматурман" в 2008 году исполнила песню вместе с
француженкой Патрисией Каас.

Автор:
Евгений Ткаченко (Тюмень)

Вопрос 21:
Вскоре после Октябрьской революции народные острословы говорили, что
Романова Александра своим поведением уничтожила трон императора Николая.
Кому на самом деле была посвящена эта фраза?

Ответ:
[Григорию] Распутину.

Комментарий:
Первые буквы слов данной фразы образуют фамилию Распутин.

Автор:
Евгений Ткаченко (Тюмень)

Вопрос 22:
Изначально это была песня о любви, написанная на идиш. Первые русские
варианты начинались так: "Старушка не спеша дорогу перешла" и "Красавица
моя красива, как свинья". В 1940 году Павел Гандельман написал свои
слова на ту же музыку, ставшие наиболее популярными. В каком городе
происходит действие в его варианте?

Ответ:
Кейптаун.

Комментарий:
"В кейптаунском порту...".

Автор:
Николай Васильев (Тюмень)

Вопрос 23:
По одной из версий, ИХ привезли к нам монголо-татары. У многих народов
есть ИХ аналоги. Например, на западе России ИХ разновидность с почти
прозрачной оболочкой называется "колдуны". Другие ОНИ отличаются
оранжевой "оболочкой" и живым "содержимым". Назовите город - родину этой
"разновидности".

Ответ:
Екатеринбург.

Зачет:
Свердловск.

Комментарий:
ОНИ - пельмени, а конкретная разновидность - команда КВН "Уральские
пельмени". Их "форменная одежда" - оранжевые рубашки.

Источник:
   1. http://ru.wikipedia.org/wiki/Пельмени
   2. http://ru.wikipedia.org/wiki/Колдуны

Автор:
Владислав Тайшин (Ханты-Мансийск)

Вопрос 24:
Продолжим тему. "Уральские пельмени" даже после ухода из "большого" КВН
продолжают творческую деятельность и создают свои программы: "Назад в
булоШную", "Звездные вопли", "Смех в большом городе"... Еще одно
название напоминает о сериале, герои которого чем-то близки команде. В
то же время название изменено в связи с "пельменной" спецификой.
Воспроизведите полученное название.

Ответ:
"Счастливы в тесте".

Комментарий:
Сериал "Счастливы вместе". Герои его действительно близки "Пельменям",
поскольку проживают в том же Екатеринбурге.

Источник:
   1. http://www.pelmeny.com/index.php?chapter=5&page=25
   2. http://www.ofbox.net/index.php?newsid=276
   3. http://www.lean-m.ru/projects/happytogether

Автор:
Владислав Тайшин (Ханты-Мансийск)

Вопрос 25:
Прослушайте диалог из одного чата:
   - Ты олицетворение птицы-феникса: сколько тебя ни бань - все равно
возвращаешься!
   - Олицетворение чего?
   - Эх, молодежь!..
   Далее автор называет одного мультперсонажа, тоже чем-то близкого
команде "Уральские пельмени". Назовите этого персонажа.

Ответ:
Кенни.

Зачет:
Кенни Маккормик.

Комментарий:
Его сколько ни убивай - вернется. Его с "Пельменями" связывает оранжевый
цвет одежды.

Источник:
http://bash.org.ru/quote/395191

Автор:
Максим Самков (Ханты-Мансийск)

Вопрос 26:
Первая цитата: "Обычная пища ПЕРВОГО - тундровые грызуны, птенцы и яйца
во множестве гнездящихся в тундре птиц, а также падаль и выбросы моря".
Вторая цитата: "ВТОРЫЕ в московской Руси назначались из "дворян добрых и
знатных людей", для производства межевания и для составления ... книг.
Никаких специальных знаний от них не требовалось; между ними
встречались, хотя, по всей вероятности, и не часто, люди совсем
неграмотные". Для полноты картины добавим, что ПЕРВЫЙ и ВТОРОЙ
отличаются одной буквой. Назовите обоих.

Ответ:
Песец, писец.

Источник:
   1. Энциклопедия Кирилла и Мефодия - 2004.
   2. http://dic.academic.ru/dic.nsf/brokgauz_efron/80028/

Автор:
Антон Катермин (Нижневартовск)

Вопрос 27:
19 сентября в одном БАКе обнаружилась утечка почти 6 тонн жидкого гелия.
Одна из основных характеристик этого БАКа составляет 26659 метров.
Назовите ближайший к этому БАКу крупный город.

Ответ:
Женева.

Комментарий:
БАК - это большой адронный коллайдер. 26659 м - длина основного кольца
ускорителя.

Источник:
   1. http://www.vesti.ru/doc.html?id=211271&cid=1
   2. http://www.forum.ximicat.com/viewtopic.php?p=22714

Автор:
Антон Катермин (Нижневартовск)

Вопрос 28:
В народной медицине он применяется, в частности, при лечении язвы,
туберкулеза, дизентерии, диабета и других болезней. А в разных
бестолковых словарях ему соответствуют значения "сотрудник ГАИ" и
"повышение цен". Назовите его.

Ответ:
Подорожник.

Источник:
   1. http://www.narmed.ru/travnik/othark/podorojnik/
   2. http://blogs.mail.ru/mail/nastya204/5CC74813B6F0E851.html

Автор:
Елена Богомолова (Нижневартовск)

Вопрос 29:
В 2008 году в составе олимпийской гандбольной сборной было 11 "ТАКИХ
ИХ". В 1938 году в экранизации ТАКОГО "ЕГО" главную роль исполнил Михаил
Жаров. Назовите ЕГО.

Ответ:
Медведь.

Комментарий:
"Чеховские Медведи" - гандбольный клуб из Чехова. В 1938 году появилась
экранизация пьесы Чехова "Медведь".

Источник:
   1. http://www.2008pekin.ru/pekinrussia2008_3_35.html
   2. http://www.ch-medvedi.ru/?sel=kom&gl=1#1
   3. http://ru.wikipedia.org/wiki/Медведь_(фильм,_1938)

Автор:
Антон Катермин (Нижневартовск)

Вопрос 30:
В одном из рассказов Бернара Вербера на Париж упал метеорит, обладающий
жутким запахом. Чтобы избавиться от зловония, люди покрывали метеорит
бетоном, гипсом, стеклом... На самом деле он был специально помещен на
Землю инопланетянином. Назовите его профессию.

Ответ:
Ювелир.

Зачет:
Продавец ювелирных изделий, ловец жемчуга.

Комментарий:
Так инопланетянин создавал жемчужины.

Источник:
Б. Вербер. Запах.

Автор:
Максим Мезрин (Нижневартовск)

Тур:
8 тур

Вопрос 1:
Внимание, в вопросе словами "ИКС", "ИГРЕК" и "ЗЕТ" заменены другие
слова.
   Согласно известному афоризму Сократа, ИКС - это лучшая приправа к
любой еде. Норвежский писатель ИГРЕК сделал "ИКС" названием одного из
своих произведений, а еще ИГРЕК часто упоминается вместе с ЗЕТОМ,
который можно использовать как средство от ИКСА. Назовите ИКС, ИГРЕК и
ЗЕТ.

Ответ:
Голод, Кнут, пряник.

Комментарий:
По Сократу лучшая приправа к любой еде - голод. Повесть "Голод"
принадлежит перу Кнута Гамсуна.

Источник:
   1. БЭКМ-2007, словарь афоризмов, ст. "Сократ".
   2. БЭКМ-2007, ст. "Гамсун".

Автор:
???

Вопрос 2:
Прослушайте отрывок из стихотворения Мережковского "Чужбина-Родина", в
котором мы пропустили три слова:
   Эти бледные березы,
   И дождя ночные слезы,
   И унылые поля...
   О, проклятая, святая,
   О, чужая и родная
   [три слова пропущено] земля!
   Пропущенные три слова на слух неотличимы от названия растения.
Назовите это растение.

Ответ:
Мать-и-мачеха.

Зачет:
Мать и мачеха.

Комментарий:
Чужая и родная мать и мачеха земля.

Источник:
   1. Русская поэзия начала XX века: Антология. - М.: Детская
литература, 2003. - С. 35.
   2. http://slovari.yandex.ru/dict/brokminor/article/27/27336.html

Автор:
Сергей Веселков (Вологда)

Вопрос 3:
Внимание, бескрылка!
   ПАРИС КРИЧИТ ТРОЯНЦАМ, НАПУГАННЫМ НЕУЯЗВИМОСТЬЮ АХИЛЛА.
   Битва Париса взъярила.
   Вскликнул он, мчась на подмогу:
   "Стрелами бейте Ахилла!
   [...]!".
   Закончите бескрылку строкой из песни.

Ответ:
Смело, товарищи, в ногу.

Комментарий:
Строка из известной революционной песни.

Источник:
http://www.a-pesni.golosa.info/starrev/smelotovarischi.htm

Автор:
Евгений Ярков (Тюмень), Алексей Усанов (Киров)

Вопрос 4:
Согласно русскому фольклору, бабий ум подобен ЭТОМУ: и косо, и криво, и
на два конца. А Николай Вашкевич считает, что ЭТО получило свое название
от своеобразного произнесения арабами слова "семерка". Назовите ЭТО.

Ответ:
Коромысло.

Комментарий:
Применяем арабский способ чтения, т.е. справа налево, концентрируя
внимание на согласных, как в арабском, получаем ЛСМРК, что значит: аль
(артикль) + СМРК ("семерка").

Источник:
В. Даль. Толковый словарь русского языка.

Автор:
Людмила Бабич (Казань)

Вопрос 5:
В "Независимой газете" ИХ около 55, в "Пушкинском музее" - около 58, а в
"Советской конституции" почти 120. Своим названием ОНИ обязаны стручку
рожкового дерева. Назовите ИХ.

Ответ:
Караты.

Комментарий:
Упомянуты некоторые названия именных алмазов Алмазного фонда России.

Источник:
   1. http://www.alrosa.ru/diamonds/named/
   2. http://ru.wikipedia.org/wiki/Карат

Автор:
???

Вопрос 6:
В одном стихотворении, в числе прочих, упомянут смешной толстяк,
подпрыгивающий, как только у него спрашивают, как дела, а также некий
нерешительный персонаж, который долгое время не может определиться, Петя
он или Вова. Назовите прилагательное, характеризующее этого персонажа.

Ответ:
Безымянный.

Комментарий:
Речь идет о пальцах.
   Вот большой, смешной толстяк,
   Любит хвастать просто так.
   "Как дела?" - спроси его.
   Он подпрыгнет, крикнет: "Во!".
   Безымянный до утра
   Выбирает имена:
   Может, Петя? Или Вова?
   Или Алла Пугачева?

Источник:
http://yarastu.narod.ru/

Автор:
???

Вопрос 7:
На юмористическом рисунке Бориса Спеца присутствуют девушка на поле боя
и юноша-милитарист. При этом юноша и девушка являются тезками
спортсменов, признанных в 2008 году лучшими россиянами в биатлоне.
Назовите имена этих юноши и девушки.

Ответ:
Максим и Катюша.

Зачет:
Максим и Катя.

Комментарий:
Максим и Катюша (военная техника) должны хорошо стрелять. "Стреляющие
лыжники" - Максим Чудов и Екатерина Юрьева.

Источник:
Рисунок Бориса Спеца.

Автор:
Надия Шафигуллина, Валерия Зарипова (Казань)

Вопрос 8:
Пятеричники - существа из произведений Станислава Лема - живут на
горячей планете Антилене и мерзнут уже при 600 градусах Цельсия. Поэтому
они даже слышать не хотят о ПЕРВОМ, в то время как описания ВТОРОГО их
весьма интересуют. А что, согласно популярной комедии, находится между
ПЕРВЫМ и ВТОРЫМ?

Ответ:
Чистилище.

Комментарий:
ПЕРВЫЙ - рай, ВТОРОЙ - ад. В "Божественной комедии" было и чистилище.

Источник:
   1. http://www.warrax.net/Satan/Dark_literature/lem22.html
   2. Данте. Божественная комедия.

Автор:
???

Вопрос 9:
Комментируя лидерство в чемпионате Голландии клуба-середняка,
еженедельник "Футбол" писал: "Еще год назад увидеть на первом месте
"[ПРОПУСК]" можно было лишь в [ПРОПУСК]". Восстановите любой из
пропусков.

Ответ:
Бреду.

Комментарий:
Клуб - "Бреда". "Бреду" в бреду.

Источник:
В вопросе.

Автор:
???

Вопрос 10:
Прослушайте четверостишие Романа Адрианова, в котором словами "ПЕРВАЯ" и
"ВТОРАЯ" мы заменили другие два слова.
   Людей пугая многомерностью,
   Слова старательно прилизаны,
   И странно мне, что ПЕРВАЯ с ВТОРОЮ
   Одними буквами написаны.
   ПЕРВАЯ и ВТОРАЯ присущи человеку и в разных ситуациях являются
синонимами или антонимами друг другу. Какие слова мы заменили на
"ПЕРВАЯ" и "ВТОРАЯ"?

Ответ:
Верность и ревность.

Зачет:
В любом порядке.

Источник:
http://zhurnal.lib.ru/a/adrianow_r_o/sestratalanta.shtml

Автор:
Екатерина Новицкая, Светлана Зацепилина (Калининград)

Вопрос 11:
Константина Драгаша, Иоанна де Бриенна, Франца II, Хайле Селассие и
некоторых других иногда называют так же, как и оскароносную историческую
драму. А как именно?

Ответ:
Последний император.

Комментарий:
Перечислены последние императоры разных империй. Фильм Бернардо
Бертолуччи "Последний император" снят о трехлетнем китайском императоре
Пу И.

Источник:
   1. http://en.wikipedia.org/wiki/Constantine_XI_Palaiologos
   2. http://en.wikipedia.org/wiki/John_of_Brienne
   3. http://en.wikipedia.org/wiki/Francis_II,_Holy_Roman_Emperor
   4. http://en.wikipedia.org/wiki/Haile_Selassie_I
   5. http://en.wikipedia.org/wiki/Puyi

Автор:
Темури Данелия (Тбилиси)

Вопрос 12:
В память о НЕЙ на карте русской Арктики осталось ЛИШЬ ТРИ СЛОВА: остров
Софии, мыс Софии. Назовите ЕЕ фамилию.

Ответ:
Колчак.

Комментарий:
Это супруга А.В. Колчака, София, а вовсе не Анна Тимирева. Аллюзия на
песню из фильма "Адмирал", где "Лишь три слова: "Останься в живых,
Адмирал"". И тоже за три слова выдают четыре. :-)

Автор:
Анна Терентьева (Казань)

Вопрос 13:
Как известно, Игнобелевская премия вручается в начале октября за
достижения, которые невозможно воспроизвести или же нет смысла этого
делать. В Игнобелевском комитете существуют также шуточные должности.
Например, "хранителем метлы" является исследователь в области квантовой
физики Рой Глаубер, в чьи обязанности входит уборка зала от самолетиков.
В 2005 году он не смог присутствовать на церемонии вручения. Назовите
город, в котором он находился в это время.

Ответ:
Стокгольм.

Комментарий:
Игнобелевские премии вручаются в начале октября, то есть в то время,
когда называются лауреаты настоящей Нобелевской премии. В 2005 году Рой
Глаубер получил половину Нобелевской премии по физике. Другая половина
присуждена Джону Холлу и Теодору Хэншу.

Источник:
http://ru.wikipedia.org/wiki/Глаубер,_Рой

Автор:
???

Вопрос 14:
Со времен Древнего Египта миру были известны такие металлы, как золото,
серебро, ртуть, медь, железо, олово и свинец. В XV веке к ним могла быть
причислена и сурьма, однако многие алхимики упорно отказывались
признавать ее за металл. А всё потому, что им не был известен ОН.
Назовите ЕГО четырьмя буквами.

Ответ:
Уран.

Комментарий:
Древние египтяне связывали семь известных им небесных тел с металлами:
золото - Солнце, серебро - Луна, ртуть - Меркурий, медь - Венера, железо
- Марс, олово - Юпитер, свинец - Сатурн. А для сурьмы Уран еще не был
обнаружен. Гершель открыл Уран только в XVIII веке.

Источник:
Б. Некрасов. Курс общей химии.

Автор:
Вахтанг Элердашвили (Тбилиси)

Вопрос 15:
12 сентября 2007 года на Новосибирской ГЭС возник пожар, который
усилиями большой группы пожарных был успешно локализован. Тем не менее,
последствия пожара были очевидны. В связи с этим появилась статья,
заголовок которой совпадал с названием произведения 1972 года,
созданного группой англичан. Назовите это произведение.

Ответ:
"Smoke on the Water".

Комментарий:
В результате пожара была загрязнена река Обь, а воздух весь пропитался
дымом. Речь идет о композиции "Smoke on the Water" группы "Deep Purple".

Источник:
   1. http://news.ngs.ru/more/31033.php
   2. http://en.wikipedia.org/wiki/Smoke_on_the_Water

Автор:
Ираклий Назгаидзе (Тбилиси)

Вопрос 16:
Во время юбилейного концерта Николая Добронравова ведущая концерта
сказала: "Николай Добронравов о любви написал много песен, а о НЕЙ
только одну. Но это, наверное, тот самый случай, когда...". Мы надеемся,
что вы сможете закончить фразу тремя словами.

Ответ:
"... песни довольно одной".

Комментарий:
ОНА - надежда. Пропущены слова из песни на стихи Добронравова "Надежда".

Источник:
Юбилейный концерт Николая Добронравова, Первый канал, 22.11.2008 г.

Автор:
Владимир Сорокин (Вологда)

Вопрос 17:
"Что вам принести? Завтрак, обед или ужин?". Такой вопрос принято
задавать в заведениях определенного типа. Назовите этот тип заведений.

Ответ:
Казино.

Комментарий:
В казино отсутствуют часы и другие предметы, напоминающие игроку о
течении времени.

Источник:
А. Вознесенский. Улисс улиц.

Автор:
Евгений Ярков (Тюмень)

Вопрос 18:
Внимание, в вопросе словами "ИКС" и "ИГРЕК" заменены другие два слова.
   Во время матча чемпионата России по баскетболу тренер одной из команд
взял тайм-аут, в течение которого методично объяснял подопечным, как
нужно играть. Комментатор канала "Спорт" по этому поводу сказал: "Тренер
сделал ИКС, чтобы дать ИГРЕК". Какие отличающиеся только первой буквой
слова мы заменили на "ИКС" и "ИГРЕК"?

Ответ:
Остановка, установка.

Комментарий:
Сделал остановку, чтобы дать установку на остаток матча.

Источник:
Трансляция матча "Урал-Грейт" - "Химки" на телеканале "Спорт",
08.03.2008 г.

Автор:
???

Вопрос 19:
Во времена Петра I была учреждена медаль "За пьянство". Один из
параметров этой медали равен шести целым восьми десятым. Что это за
параметр?

Ответ:
Вес.

Зачет:
Масса.

Комментарий:
Эта медаль считается самой тяжелой и вешалась "злоупотребляющему" на шею
так, чтобы он ее не мог снять.

Источник:
http://ru.wikipedia.org/wiki/За_пьянство_(медаль)

Автор:
Евгений Ярков (Тюмень)

Вопрос 20:
В фильме Анджея Вайды советский солдат одну из ЕГО частей делает
портянкой, а другую вешает на стену. Назовите ЕГО.

Ответ:
Флаг Польши.

Комментарий:
Флаг Польши состоит из белой и красной полосы. Белая становится
портянкой, а красная - знаком коммунистов.

Источник:
Фильм "Катынь".

Автор:
Иван Беляев (Вологда)

Вопрос 21:
(pic: 20090829.jpg)
   Перед вами два изображения, обнаруженные в Вашингтоне на одном из
предметов. Какие два английских слова, отличающихся только двумя
буквами, мы скрыли от вас на этом изображении?

Ответ:
Push, pull.

Зачет:
В любом порядке.

Комментарий:
На одной картинке скрыто слово "PULL", а на другой - "PUSH". В США эти
надписи есть практически на всех дверях в общественных местах.
Разумеется, картинки были с разных сторон двери, тем не менее, это -
один объект.

Источник:
Личный опыт автора вопроса.

Автор:
Георгий Бакрадзе (Тбилиси)

Вопрос 22:
ЭТИМ СЛОВОМ принято называть человека с больными нервами, с ярко
выраженными признаками вырождения. А Терри Пратчетт в одном из своих
романов назвал ЭТИМ СЛОВОМ человека, у которого не хватало 22 зубов.
Напишите ЭТО СЛОВО.

Ответ:
Декадент.

Комментарий:
10 зубов - декадент.
   z-checkdb: С 1925 года значение слова, принятое в Советском Союзе,
изменилось, сейчас это просто последователь соответствующего направления
в искусстве (Антон Губанов).

Источник:
   1. http://www.feb-web.ru/feb/slt/abc/lt1/lt1-1862.htm
   2. http://people.cs.uchicago.edu/~dinoj/pratchet.html

Автор:
Вахтанг Элердашвили (Тбилиси)

Вопрос 23:
Внимание, в вопросе есть замены.
   В телепередаче, посвященной Зурабу Церетели, его роскошный московский
особняк был назван: "ПРОКЛЯТЫЙ СТАРЫЙ дом на ПРОКЛЯТОЙ СТАРОЙ улице".
Какие два слова мы заменили на "ПРОКЛЯТЫЙ СТАРЫЙ"?

Ответ:
Большой грузинский.

Комментарий:
Он и расположен на Большой Грузинской улице.

Источник:
Истории в деталях, СТС.

Автор:
???

Вопрос 24:
Джованни Баттиста Риччоли в 1651 году опубликовал книгу, где приведено
77 аргументов против учения Коперника, которую он назвал новым ИМ. ОН -
это арабизированная форма названия "Великое построение". Назовите ЕГО.

Ответ:
Альмагест.

Комментарий:
"Альмагест" ("Великое построение") - знаменитая книга Птолемея, где
провозглашается геоцентрическая система.

Источник:
Энциклопедия для детей. - Т. 16. Физика. - Ч. 1. - С. 117.

Автор:
???

Вопрос 25:
По одной из версий, эта американская актриса выбрала себе псевдоним в
память о предках, сложив его из английских букв, составляющих слово
"Германия". Назовите этот псевдоним.

Ответ:
Мэг Райан.

Комментарий:
Meg Ryan, Germany.

Источник:
http://www.russian-club.net/article_1802.html

Автор:
Николай Коврижных (Киров)

Вопрос 26:
Внимание, в вопросе есть замены.
   В рассказе Уильяма Тенна персонаж говорит недальновидной девушке, что
если она будет плохо кормить своего мужа, то тот решит, что у нее голова
не думает. Какие три слова мы заменили на "голова не думает"?

Ответ:
Котелок не варит.

Зачет:
Не варит котелок.

Комментарий:
Двусмысленная шутка. Надеемся, вы еще полны сил и котелки у вас варят
как надо.

Источник:
Уильям Тенн. Шутник.

Автор:
???

Вопрос 27:
В одной из пародий на телепередачу "Поле чудес" в супер-игре ведущий
традиционно выкладывает на барабан таблички с названиями призов. Среди
них "Лада-Калина", "УАЗ-патриот", "Лада-Приора". Назовите надпись на
последней табличке.

Ответ:
Автомобиль.

Комментарий:
Очень много шуток сложено про знаменитое восклицание Леонида Аркадьевича
"... и автомобиииль!", ибо это самый шикарный приз на упомянутой
передаче. Здесь же шутят над российским автопромом в придачу.

Источник:
Один из выпусков передачи "Большая разница" на ОРТ.

Автор:
???

Вопрос 28:
   <раздатка>
   Total depravity - всеобъемлющая греховность
   Unconditional election - безусловное избрание
   Limited atonement - ограниченное искупление
   Irresistible grace - неотразимая благодать
   Perseverance of the saints - стойкость святых.
   </раздатка>
   В символ веры кальвинизма входят пять пунктов, которые вы можете
видеть перед собой. Назовите этот символ одним словом.

Ответ:
Тюльпан.

Зачет:
Tulip.

Комментарий:
Аббревиатура, по первым буквам названий принципов.

Источник:
Л.С. Баешко, А.Н. Гордиенко, А.Н. Гордиенко. Энциклопедия символов. -
Статья "Тюльпан".

Автор:
???

Вопрос 29:
При изготовлении базиса зубных протезов, который служит для крепления
зубов, используют розовую пластмассу, если протез для "ПЕРВЫХ", и
фиолетовую, если для "ВТОРЫХ". Слова "ПЕРВЫЙ" и "ВТОРОЕ" присутствуют в
названии книги 1971 года наряду с именем собственным и частью тела.
Напишите это название.

Ответ:
"Белый Бим Черное ухо".

Комментарий:
Базис имитирует десну. У "белых" (белокожих) людей десны розовые, а у
"черных" (афроамериканцев, например) - фиолетовые.

Источник:
   1. Занятие на кафедре факультетской ортопедической стоматологии
МГМСУ.
   2. http://ru.wikipedia.org/wiki/Белый_Бим_Чёрное_ухо_(книга)

Автор:
???

Вопрос 30:
По одной из версий, у этого героя был реальный прототип - итальянец по
имени Маурицио. Он командовал венецианскими войсками на Кипре и потерял
там свою жену при крайне подозрительных обстоятельствах. А какую фамилию
носил этот итальянец?

Ответ:
Отелло.

Комментарий:
"Мауро" по-итальянски также означает "мавр", это и привело к ошибке
Шекспира, присвоившего герою такую национальность.

Источник:
http://ru.wikipedia.org/wiki/Отелло

Автор:
Сергей Скорик (Луганск)

Тур:
9 тур

Вопрос 1:
ОНА является названием национальной премии в российском футболе,
вручаемой лучшему молодому футболисту Премьер-лиги. У автора вопроса она
связана с несколькими ровно выведенными буквами "А". Назовите ЕЕ двумя
словами, начинающимися на одну букву.

Ответ:
Первая пятерка.

Источник:
   1. http://ru.wikipedia.org/wiki/Первая_пятёрка
   2. ЛОАВ.

Автор:
Николай Коврижных (Киров)

Вопрос 2:
Василий Ливанов, снимавшийся в "Этюде в багровых тонах", в одном
интервью пояснил, что при озвучивании по просьбе чиновников пришлось
заменить одно слово словом "Восток". Напишите это слово.

Ответ:
Афганистан.

Комментарий:
По тексту рассказа "Этюд в багровых тонах" Холмс при первой встрече с
Ватсоном угадывает, что тот недавно вернулся из Афганистана. Но при
озвучивании фильма из опасений политической цензуры "Афганистан" был
заменен на "Восток". Тем не менее, в кадре по движению губ явственно
видно, что Василий Ливанов произносит слово "Афганистан".

Источник:
   1. Интервью Василия Ливанова в программе "Новости" на Первом канале,
эфир от 05.03.2009 г.
   2. http://ru.wikipedia.org/wiki/Гражданская_война_в_Афганистане

Автор:
Константин Ключевский (Луганск)

Вопрос 3:
Есть несколько версий происхождения цветов "Ньюкасл Юнайтед". По одной
версии, цвета были выбраны в честь священника отца Долматиуса Хоутмана,
по другой - они повторяли форму роялистов времен Английской гражданской
войны. По третьей версии - на цвета повлияли "соседи", строившие свои
"дома" рядом с футбольным стадионом. Назовите этих соседей.

Ответ:
Сороки.

Комментарий:
Футболистов "Ньюкасл Юнайтед" называют сороками.

Источник:
"Великие клубы Англии", 2008, N 9 (33).

Автор:
Александр Толстобров (Саранск)

Вопрос 4:
Внимание, в вопросе есть замены.
   По мнению Дмитрия Быкова, Ильф и Петров сочинили новую "Одиссею", где
роль медузы, по созвучию, играет мадам Грицацуева, и нет сомнений, что
Бендер, останься он с ней, быстро превратился бы в камень. Какие слова
мы заменили на Медузу и камень?

Ответ:
Цирцея, свинья.

Зачет:
Кирка, свинья.

Источник:
http://www.rulife.ru/index.php?mode=article&artID=1211

Автор:
Александр Толстобров (Саранск)

Вопрос 5:
Первоначально он в своем завещании, составленном 14 марта 1893 года,
выразил желание направить средства от своих патентов на строительство
крематориев в крупных городах, чем, по его мнению, должен был заниматься
стокгольмский Каролинский институт. Однако папа римский признал кремацию
неподобающей формой погребения, и он изменил завещание. Назовите его.

Ответ:
[Альфред] Нобель.

Источник:
http://www.rian.ru/spravka/20071210/91630895.html

Автор:
Александр Толстобров (Саранск)

Вопрос 6:
Один чувашский народный поэт в 20-е годы писал следующие строчки:
   "День и ночь шумит селенье.
   Говорят, приехал [ПРОПУСК],
   Говорят, он из КУКУЙСКА,
   Ну, до нас оттуда близко!".
   Догадавшись, чью фамилию мы заменили словом "пропуск", ответьте,
какой город мы обозвали КУКУЙСКОМ.

Ответ:
Симбирск.

Комментарий:
Говорят, приехал Ленин, говорят, он из Симбирска...

Источник:
И.Б. Брайнин. К вождю за советом. - М., 1986. - С. 226.

Автор:
???

Вопрос 7:
В дохристианской Руси детей не называли "взрослыми" именами до возраста
10-13 лет, а давали имена, образованные от числительных. В числе таких
имен фамилия знаменитого россиянина, с недавнего времени - президента.
Назовите этого человека.

Ответ:
[Владислав] Третьяк.

Комментарий:
Далее приводились примеры: Первак, Вторак, Третьяк. Владислав Третьяк -
легендарный советский хоккейный вратарь, с 2006 года президент Федерации
хоккея России.

Источник:
   1. http://ru.wikipedia.org/wiki/Третьяк
   2. http://ru.wikipedia.org/wiki/Третьяк,_Владислав_Александрович

Автор:
???

Вопрос 8:
В один из дней начала лета прошлого года на главной странице сайта
Яндекс появился заголовок "ОН в НИХ". Под заголовком помещалось
множество адресов других сайтов. А вот энциклопедия Кирилла и Мефодия
сообщает, что ОН был в НИХ с 1820 по 1826 год. Назовите и ЕГО, и ИХ.

Ответ:
Пушкин, ссылки.

Источник:
БЭКМ-2007, ст. "Пушкин Александр Сергеевич".

Автор:
???

Вопрос 9:
В телепрограмме "Смак" Роман Карцев рассказал такую историю. На съемках
фильма "Волны Черного моря" его дублер при прыжке получил травму -
серьезные переломы. Через много лет Карцев встретил этого человека в
Калифорнии, тот показал Карцеву протез и при этом сказал: "Я благодарен
вам, моя жизнь сложилась очень удачно, но [два слова пропущено] в кино
больше не будет". Заполните пропуск.

Ответ:
"... ноги моей...".

Источник:
Телепрограмма "Смак", Первый канал, 22.11.2008 г.

Автор:
Юрий Денисов (Вологда)

Вопрос 10:
Согласно одному источнику, ЭТО стало косвенной причиной смены дислокации
одной пары. Согласно другому источнику, ЭТО стало косвенной причиной
десятилетней войны. Назовите фамилии двух тезок, которым ЭТО косвенным
образом принесло миллионы долларов.

Ответ:
Джобс и Возняк.

Комментарий:
Яблоко. Адам и Ева изгнаны из рая. Троянская война - из-за яблока
Париса.

Автор:
Ираклий Назгаидзе (Тбилиси)

Вопрос 11:
Глава ФБР (Федерального Бюро Расследований) Джон Эдгар Гувер очень любил
делать пометки на полях адресованных ему служебных записок. Если верить
Википедии, однажды Гувер, читая одну из таких записок, увидел, что на
листочке оставлены очень узкие поля, и приписал: "Обратите внимание на
поля". Сотрудники не поняли, что именно имел в виду Гувер и, зная его
чудаковатый нрав, не осмелились спросить, но активизировали оперативную
работу в отношении двух государств. Назовите эти два государства.

Ответ:
Канада и Мексика.

Комментарий:
По-английски надпись "Обратите внимание на поля" звучит как "Watch the
borders". Border также переводится как "граница", поэтому было решено
обратить больше внимания на контролирование границы с соседними
странами.

Источник:
http://en.wikipedia.org/wiki/J._Edgar_Hoover

Автор:
Сергей Капанадзе (Тбилиси)

Вопрос 12:
Внимание, мнемоническое правило: "Кто и шутя и скоро пожелаетъ Пи узнать
число, ужъ знаетъ". Текстовый редактор Word подчеркнул в нем три слова.
Напишите эти три слова абсолютно точно.

Ответ:
Пожелаетъ, ужъ, знаетъ.

Комментарий:
Это дореволюционный способ, записанный по старым правилам орфографии,
когда в конце слова писался твердый знак.

Источник:
Н. Латыпов. Основы интеллектуального тренинга.

Автор:
Людмила Бабич (Казань)

Вопрос 13:
Тулин, Петров, Иванов, Силин, Фрей, Перючёв... В этом списке не хватает
еще многих фамилий. Назовите хотя бы одну - самую известную.

Ответ:
Ленин. Незачет: Ульянов.

Комментарий:
Это псевдонимы Владимира Ильича Ульянова.

Автор:
Евгений Долгирев

Вопрос 14:
Как известно, остряки любят давать прозвища многим людям. Так, например,
известному итальянцу, начинавшему спортивную карьеру в церковной команде
"Дон Орион", они дали прозвище "кудрявый". Но известно и другое, более
подходящее ему, прозвище, совпадающее с именем героя серии романов и
кинофильмов XX века. Мы не просим вас написать прозвище. Назовите этого
человека.

Ответ:
[Пьерлуиджи] Коллина.

Комментарий:
Всем известный арбитр Коллина - лысый, и острословы быстро придумали ему
колкое прозвище - Фантомас.

Источник:
   1. http://ru.wikipedia.org/wiki/Коллина,_Пьерлуиджи
   2. http://www.blog.shevi.name/?p=288

Автор:
Евгений Потапов (Киров)

Вопрос 15:
Прослушайте текст:
   Тяжелая техника издавала звуки низких частот на равнинной местности.
Юноши призывного возраста приближались к финальной стадии тактических
вооруженных действий.
   Что в это время происходило с их юным лидером?

Ответ:
Несли с пробитой головой.

Источник:
Песня "На поле танки грохотали".

Автор:
Команда "Гладиолус" (Киров)

Вопрос 16:
В сообщении о неприятности, произошедшей с официальным сайтом фильма
"Хакеры", встречается часть названия совсем другого фильма - советского.
Назовите режиссера этого советского фильма.

Ответ:
Эльдар Рязанов.

Комментарий:
"Ирония судьбы...".

Автор:
Команда "СМЕНА"

Вопрос 17:
Блиц.
   19 марта 2008 года на сайте Voffka.com было проведено несерьезное
исследование. Автор попытался предположить, о чем сняты известные
фильмы, если судить о них только по названию. Так, по его мнению,
"Вспомнить всё" - про вчерашний праздник, а "Фантастическая четверка" -
про тюнингованные "Жигули" четвертой модели. Вникнув в подобную логику,
постарайтесь ответить:
   1. Как называется фильм про русскую рулетку и "догони меня, кирпич"?
   2. Как называется фильм о том, как цыгане построили Шаттл?
   3. Как называется фильм про алкоголичку?

Ответ:
   1. "Жестокие игры".
   2. "Табор уходит в небо".
   3. "Пила".

Источник:
http://voffka.com/archives/2008/03/19/042558.html

Автор:
Игорь Якимчук (Тюмень)

Вопрос 18:
На сайте "Спорт сегодня", среди прочего, есть множество конференций,
ведущие которых отвечают на вопросы пользователей. Какому виду спорта
посвящена конференция Шевченко Алексея и Белова Алексея?

Ответ:
Хоккею [с шайбой].

Источник:
http://www.sports.ru/conference/shaiba/

Автор:
Игорь Якимчук (Тюмень)

Вопрос 19:
Линейку вкладов одного из российских банков составляют вклады с
названиями "Карамель", "Зефир", "Грильяж" и "Шоколад". Ее название
совпадает с названием фильма 1960 года об обществе, переживающем
экономическое чудо после лет нищеты. Назовите режиссера этого фильма.

Ответ:
[Федерико] Феллини.

Комментарий:
"Сладкая жизнь".

Источник:
   1. http://www.globexbank.ru/main.mhtml?Part=336&PubID=1238
   2. http://ru.wikipedia.org/wiki/Феллини,_Федерико

Автор:
Игорь Якимчук (Тюмень)

Вопрос 20:
Согласно русской народной примете, если это событие произойдет в
октябре, зима будет бесснежной. В названии произведения отечественного
фантаста это событие не произошло. Назовите писателя, в названии
произведения которого это событие произошло.

Ответ:
[Рэй] Брэдбери.

Источник:
   1. Е. Лукин. И гром не грянул (2001).
   2. Р. Брэдбери. И грянул гром (1952).

Автор:
Мария Рафалович (Тюмень)

Вопрос 21:
Известный своей творческой плодовитостью Стивен Кинг сказал, что
ненавидит их, и потому всегда в настроении что-либо написать. Их
обозначение в одной из популярных компьютерных программ состоит из 11
символов. Изобразите шестой символ.

Ответ:
: (двоеточие)

Комментарий:
Речь идет о пустых страницах.

Источник:
   1. "Esquire", июль-август 2008 г.
   2. about:blank в Internet Explorer.

Автор:
Юрий Гнатюк (Сургут)

Вопрос 22:
Некоторые плакаты утверждают, что Z главнее всех X. Известный анекдот
утверждает, что Z - это X в предмете одежды, изобретателем которого
считается Коко Шанель. Один мультперсонаж пытался доказать, что он Z,
предъявив другой предмет одежды. А какой предмет он использовал
непосредственно после этого, попытавшись предстать в другом образе?

Ответ:
Таз.

Зачет:
Тазик.

Комментарий:
Плакаты, посвященные пешеходным переходам, говорят, что зебра главнее
всех лошадей. В анекдоте зебру приняли за лошадь в пижаме, изобретателем
коей считается Коко Шанель. Волк из "Ну, погоди!", пытаясь зайти в
концертный зал с черного хода, сначала выдавал себя за "зёбру", показав
тельняшку, а затем, прикрывшись тазом, - за черепаху.

Автор:
Алексей Кучин (Сургут)

Вопрос 23:
Прослушайте стихотворение.
   По дороге шли бандиты,
   Смотрят - черный кот сердитый
   Им дорогу пересек:
   - Кто послал тебя, зверек?
   - ИКС, - сквозь зубы он изрек.
   Какое слово мы заменили на "ИКС"?

Ответ:
МУР.

Зачет:
Мур.

Автор:
Екатерина Сосенко (Тюмень)

Вопрос 24:
В 2008 году "Зенит" выиграл кубок УЕФА в матче с шотландцами.
Посвященная этому событию статья в газете "Советский спорт" получила
заголовок, отличающийся от известной идиомы добавлением двух букв и
подчеркивающий, что "Зенит" попал в нужную точку. Воспроизведите
последнее слово этого заголовка.

Ответ:
Глазго.

Комментарий:
"Не в бровь, а в Глазго".

Источник:
"Советский спорт" от 15.05.2008 г.

Автор:
Екатерина Сосенко (Тюмень)

Вопрос 25:
Внимание, в вопросе есть замена.
   В рецензии на альбом Земфиры "Спасибо" автор не скупится на хвалебные
эпитеты, в том числе, изменив в известной идиоме одну букву, называет
пластинку ПОП ЗЕМЛИ. Вспомните, к какому музыкальному стилю относится
творчество певицы, и напишите, какие два слова мы заменила на "ПОП
ЗЕМЛИ".

Ответ:
Рок изобилия.

Комментарий:
Обыгрывается известная идиома "рог изобилия".

Источник:
"Афиша", 2007, N 19.

Автор:
Юрий Гнатюк (Сургут)

Вопрос 26:
В мифологическом словаре 1996 года выпуска утверждается, что раньше в
низшей мифологии великорусов и латгальцев этим словом называли
маленького зловредного домового духа, которого часто представляли в
образе пузатой крысы "с лицом вроде человеческого". В одной легенде он
"требует выкуп, угрожая потушить огонь". Не исключено, конечно, что
авторы этого словаря просто пошутили. Какая фамилия произошла, по их
мнению, от этого слова?

Ответ:
Чубайс.

Комментарий:
Чубась, чубысь - рыжий бесенок. Но словарь издан в 1996 году, а более
ранних упоминаний об этом существе мы не нашли...

Источник:
Мифологический словарь. / Под ред. Д.С. Лихачева, Б.А. Рыбакова и др. -
М.: Наука, 1996.

Автор:
Александр Воробьев (Тюмень)

Вопрос 27:
Сокращенное название этого вида страха называется "гексафобия". Он
повлиял, например, на изменение, связанное с электропоездом Осташков -
Москва. А еще многие были недовольны выбором даты премьеры некоего
фильма. Как называется этот фильм?

Ответ:
"Омен".

Зачет:
"Знамение", "Предзнаменование", "Omen".

Комментарий:
Полное название - гексакосиойгексеконтагексафобия. Номер поезда 666
пришлось изменить: страшно! Премьера фильма "Омен" состоялась 6.06.06.

Источник:
   1. http://ru.wikipedia.org/wiki/Число_зверя
   2. Газета "Антенна-Телесемь", 2007, N 43 (348).

Автор:
Александр Кудеенко (Тюмень)

Вопрос 28:
По одной из версий, ОНА символизирует дружбу трех человек: Готлиба,
Вильгельма и Эмиля. Изобразите ЕЕ.

Ответ:
[Эмблема Mercedes.]

Комментарий:
Готлиб Даймлер и Вильгельм Майбах стояли у истоков современного
автомобилестроения, а Эмиль Еллинек одержал немало спортивных побед на
автомобилях фирмы "Daimler" и дал им прославленное ныне имя - Mercedes.

Источник:
А.А. Краснов. Самые красивые и знаменитые автомобили мира. - М.: Мир
энциклопедий "Аванта+", Астрель, 2007.

Автор:
Александр Воробьев (Тюмень)

Вопрос 29:
В 1863 году Николаю предложили написать образ ПЕРВОГО Александра, взяв
при этом лицо ДРУГОГО Александра. Примерно через 80 лет для образа
ПЕРВОГО Александра взяли лицо ДРУГОГО Николая. Назовите фамилию того
Александра, которого мы назвали ДРУГИМ.

Ответ:
Романов.

Комментарий:
Довольно часто иконы изображались с лицами действующих монархов. Вот и
Николая Ге попросили изобразить образ Александра Невского с лицом
Александра Второго. А для ордена Александра Невского было взято лицо
сыгравшего его в фильме Николая Черкасова.

Источник:
Мастера исторической живописи. / Авторы-составители Г.В. Дятлева, К.А.
Ляхова. - М.: Вече, 2001.

Автор:
Алина Ли (Тюмень)

Вопрос 30:
[Ведущему: Stoboy читать как "стОбой" (с ударением на первый слог).]
   Цитата из романа Набокова "Дар": "У этой крупной хищной немки было
странное имя; мнимое подобие [ДАЛЕЕ НАМИ ПРОПУЩЕНО ДВА СЛОВА] придавало
ему звук сентиментального заверения: ее звали Clara Stoboy". Напишите
два пропущенных слова, соблюдая падеж.

Ответ:
Творительного падежа.

Источник:
В.В. Набоков. Дар: Романы. - Свердловск: Средне-Уральское кн. изд-во,
1989.

Автор:
Алина Ли (Тюмень)

Тур:
10 тур

Вопрос 1:
Внимание, в вопросе есть замены.
   В XVII веке английский священник Томас Холл основал движение, которое
объявило ЧИСТКУ ЗУБОВ "дьявольской работой". При дворе короля Людовика
XVI ЧИСТИЛИ ЗУБЫ исключительно мужчины. А недавно американская компания
"Hard Candy" стала добавлять в некий товар кофеин. Специалисты компании
утверждают, что это помогает совместить два утренних ритуала - ПОЧИСТИТЬ
ЗУБЫ и выпить чашечку кофе. Что мы заменили словами "ЧИСТИТЬ ЗУБЫ"?

Ответ:
Красить губы.

Источник:
   1. http://www.ill.ru/news.art.shtml?c_article=523
   2. http://www.fg.ru/new_main/newshow/show.php?news=685
   3. http://www.yes.spata.ru/index.php?a=483
   4. http://www.vorle.ru/forum/forum_read.php?id=69975&f=12

Автор:
Александр Кудеенко (Тюмень)

Вопрос 2:
Александр Герцен полагал, что русское правительство устраивает к лучшему
не Y, а X. Поль Валери полагал, что Y уже не то, что было раньше. А
сериалу "Путешественник" на одном российском канале дали название с
упоминанием X, противопоставив его названию знаменитой трилогии.
Напишите в правильном порядке то, что мы заменили на X и Y.

Ответ:
Прошлое, будущее.

Комментарий:
Сериал назвали "Вперед в прошлое".

Источник:
   1. http://www.lib-aforizm.info/
   2. http://tv3channel.build2.ru/viewtopic.php?pid=551

Автор:
Александр Кудеенко (Тюмень)

Вопрос 3:
В рамках перехода на двухуровневую систему образования все профессии
были поделены на ответственные и особо ответственные. Причем X отнесли к
первым, а Y - ко вторым. Однако героиня культового советского фильма
небезосновательно утверждала, что профессия X не менее ответственна, чем
Y. Назовите обе профессии.

Ответ:
Врач, учитель.

Зачет:
В любом порядке.

Комментарий:
Надя Шевелёва и Женя Лукашин в фильме "Ирония судьбы, или С легким
паром!" оказались представителями именно этих профессий.

Источник:
Фильм "Ирония судьбы, или С легким паром!".

Автор:
Алина Ли (Тюмень)

Вопрос 4:
[Ведущему: внятно прочитать "которого" после "тезка".]
   Песня, написанная в 1929 году Хосе Фернандесом Диасом на стихи Хосе
Марти, является символом борьбы за свободу и едва ли не неофициальным
гимном одной страны. Парадоксально, что посвящена она юной жительнице
города, "тезка" которого уже семь лет как ассоциируется вовсе не со
свободой, а скорее наоборот. Назовите этот город.

Ответ:
Гуантанамо.

Комментарий:
"Гуантанамера" (исп. Guantanamera - девушка из Гуантанамо) - одна из
самых известных кубинских патриотических песен. Место содержания
международных террористов в тюрьме Гуантанамо (в 15 км от одноименного
города) появилось в январе 2002 года, когда туда были доставлены из
Афганистана первые 20 человек, обвиняемых "в участии в боевых действиях
на стороне исламских экстремистов" - талибов.

Автор:
Софья Бардасова (Тюмень)

Вопрос 5:
Когда одно из крупнейших итальянских издательств решило СДЕЛАТЬ ЭТО, оно
выпустило плакаты с обложками мировых шедевров литературы, в заглавии
которых стали фигурировать числа 1388,8 [1388 целых 8 десятых]; 2,1 [2
целых 1 десятая] и 70. Назовите абсолютно точно, что эти плакаты
рекламировали.

Ответ:
30-процентную скидку.

Комментарий:
"1984", Три мушкетера", "Сто лет одиночества".

Источник:
Журнал "Что читать", 2008, N 1. - С. 13.

Автор:
Алина Ли (Тюмень)

Вопрос 6:
Кожан - 19,9, лев - 13,5, кошка - 12,1, слон - 3,6, лошадь - 2,9, жираф
- 1,9. Мы только что огласили средние показатели некоего параметра для
разных животных. Как называется состояние, когда этот показатель у
человека на протяжении длительного времени равен 0?

Ответ:
Бессонница.

Комментарий:
Это среднесуточные показатели продолжительности сна.

Источник:
"Ридерз дайджест" - "Иммунитет: как укрепить оборону".

Автор:
Денис Моргунов (Сургут)

Вопрос 7:
Несколько лет назад в стартовом составе клуба НБА "Лос-Анджелес Лэйкерз"
выходили четыре суперзвезды: Гарри Пэйтон, Коби Брайант, Карл Мэлоун и
Шакил О'Нил. По этому поводу американские спортивные журналисты шутили,
что пятым в такой компании главный тренер может выпускать еще одну
суперзвезду, главного джокера этой голливудской команды. Хотя он за
"Лэйкерз" в официальном матче вряд ли сыграет. Назовите его фамилию.

Ответ:
Николсон.

Комментарий:
Многие фанаты упомянутого баскетбольного клуба - голливудские звезды, а
Николсон (сыгравший Джокера в фильме "Бэтмен") - пожалуй, самый
преданный из них - его можно увидеть практически на каждом домашнем
матче "Лэйкерз".

Источник:
Один из выпусков программы "NBA Live" на канале 7 ТВ.

Автор:
Денис Моргунов (Сургут)

Вопрос 8:
В журнале "Men's Health" есть юмористическая рубрика "P.S. Чему нас учит
эта картина?". Одна из картин описывается так: "Находясь на солнце,
обязательно надевай головной убор. Многие знают, что такое солнечный
удар, но не все - насколько он коварен. Человек под палящим солнцем
запросто может потерять сознание, упасть и удариться головой. А другой
человек, тоже, на беду, без головного убора, кинется помогать первому и
также окажется сражен солнечным ударом. А за ним третий, четвертый,
пятый...". Напишите фамилию автора описываемой картины.

Ответ:
Верещагин.

Комментарий:
Картина "Апофеоз войны".

Источник:
Журнал "Men's Health", июль 2008 г. - С. 194 (в малоформатном журнале).

Автор:
Денис Моргунов (Сургут)

Вопрос 9:
Внимание, в вопросе есть замены.
   Один из героев Станислава Лема, занимающийся прогнозами на основе
лингвистических возможностей языка, изрекает: "Например, "МОРдоваться":
представив, что это слово восходит к "МОРдовии", вы поймете, почему так
много футурологов-пессимистов!". Догадавшись, о какой "Мордовии" шла
речь, напишите, какое слово мы заменили на "мордоваться".

Ответ:
Утопиться.

Комментарий:
"Утопию" написал Томас МОР. :-)

Источник:
Станислав Лем. Футурологический дневник.

Автор:
Айрат Шакиров (Сургут)

Вопрос 10:
До 70-х годов прошлого столетия эта "игра интеллектуалов" по понятным
причинам не пользовалась интересом телезрителей. А Владимир Синицын
иронично заметил, что причина неизвестности игры в Советском Союзе
спрятана за парадоксальностью самой ее сути для "страны победившего
социализма". Назовите эту игру.

Ответ:
Снукер.

Комментарий:
Снукер - разновидность бильярдной игры, при которой на столе
располагаются биток (белого цвета) и 21 прицельный шар (15 красных и 6
"цветных", забиваемых поочередно). Широкую популярность стала
приобретать с началом телетрансляций "в цвете". Одним из элементов игры
является "снукер" (с англ. - "маска"), при котором биток "прячется" от
прицельных шаров. Комментатор канала "Евроспорт-Россия" Владимир
Синицын: "Что это за игра, в которой один белый бьет столько красных?!".

Источник:
   1. http://ru.wikipedia.org/wiki/Снукер
   2. Канал "Евроспорт-Россия". Матч О'Салливан - Магуайр. 24.01.2008 г.

Автор:
Айрат Шакиров (Сургут)

Вопрос 11:
Внимание, в вопросе есть замены.
   Комментатор Роман Скворцов как-то прокомментировал ситуацию на
баскетбольной площадке примерно так: "Холден пасует под кольцо
Андерсену. Андерсен ложным броском поднимает в воздух одного, затем еще
двоих. Ну, прямо ИКС ДЕЛЬТЫ какой-то!". Один ИКС ДЕЛЬТЫ отмечается 28
октября, другой - 7 декабря. Какие два слова мы заменили на "ИКС
ДЕЛЬТЫ"?

Ответ:
День авиации.

Комментарий:
Когда игроки в баскетболе пытаются накрыть бросок противника, они
выпрыгивают вверх. Если такие попытки предпринимают последовательно
несколько игроков, комментатор Роман Скворцов в шутку описывает их как
"день авиации".

Источник:
В вопросе.

Автор:
???

Вопрос 12:
Однажды автор вопроса стал участником диалога, в котором собеседником
были последовательно в уменьшительной форме упомянуты представительница
молодежной субкультуры, женское имя и компьютерная программа. Чему был
посвящен диалог?

Ответ:
Выбор одежды.

Зачет:
Одежда; покупка одежды.

Комментарий:
Доброжелательная тетенька-продавец продемонстрировала восхищенному
автору вопроса футболки с размерами M, L и XL, предварив их священной
мантрой "ЭМОчка, Элечка, Excelка".

Источник:
Личный опыт автора вопроса.

Автор:
Айрат Шакиров (Сургут)

Вопрос 13:
[Ведущему: прочитать по слогам название токсина.]
   Как известно, яд и лекарство отличаются всего лишь дозой. Одним из
сильнейших токсинов на Земле является токсин палочки ботулизма. Причем к
ботулизму не вырабатывается иммунитет. При отравлении
БО-ТУ-ЛО-ТОК-СИ-НОМ развивается мышечный паралич. Какой продукт
изготавливают на основе этого вещества и достаточно успешно применяют
сегодня?

Ответ:
Ботокс.

Зачет:
Ну, если кто-то ответит Лантокс или Диспорт, засчитывать. :-)

Комментарий:
Введение крайне малых доз ботулотоксина в виде инъекций в область
мимических морщин вызывает локальный мышечный паралич, из-за чего
морщины разглаживаются. Название "БОТОКС" образовано от "БОтулоТОКСин".
Отсутствие иммунитета позволяет исключить как вариант ответ "прививки".

Источник:
http://www.botox.ru

Автор:
Денис Моргунов (Сургут)

Вопрос 14:
Пьер Фошар считается изобретателем привинченного к полу кресла, системы
маленьких зеркал, а также ЕГО. ОН в афоризме Эмиля Кроткого перекинут
через пропасть, отделяющую от молодости. Назовите ЕГО двумя словами.

Ответ:
Зубной мост.

Комментарий:
Дантист Пьер Фошар по сути является изобретателем стоматологического
кабинета, а также зубного моста в его современном виде.

Источник:
   1. http://esquire.ru/articles/04/5minutes/
   2. http://lib.ru/ANEKDOTY/KROTKIJ/nenapisannoe.txt

Автор:
Айрат Шакиров (Сургут)

Вопрос 15:
[Ведущему: часть вопроса до "ОНА" прочитать быстро.]
   В "продолжении" известного-преизвестного произведения ОНА, уже будучи
королевой, строго покарала ЕГО за преступление. Назовите ЕЕ и ЕГО.

Ответ:
Клара, Карл.

Комментарий:
Королева Клара карала Карла за кражу кораллов.

Источник:
http://www.koshki-mishki.ru/category-155-43.html

Автор:
Айрат Шакиров (Сургут)

Вопрос 16:
Многие из нас наверняка видели ЕЕ рождение, а вот ЕЕ триумф и туалет -
наверное, не многие. В некой последовательности ОНА занимает вторую
позицию. Назовите ее соседей, расположившихся на первом и третьем местах
в этой последовательности.

Ответ:
Меркурий, Земля.

Комментарий:
Речь идет о картинах "Рождение Венеры", "Триумф Венеры", "Туалет
Венеры". Планета Венера - вторая от Солнца, Меркурий и Земля - первая и
третья соответственно.

Автор:
Денис Моргунов (Сургут)

Вопрос 17:
Название одной сети специализированных магазинов совпадает с названием
международной организации, на эмблеме которой в числе прочего изображен
меч. Однако расшифровываются они по-разному. Мы не будем спрашивать, для
кого предназначены эта сеть магазинов. А как расшифровывается ее
название?

Ответ:
Интересное положение.

Комментарий:
Сокращенно - "Интерпол".

Источник:
Журнал "9 месяцев", ноябрь 2008 г.

Автор:
Денис Моргунов (Сургут)

Вопрос 18:
Одна из читательниц журнала "Мой ребенок" сообщила мужу о своей
беременности во время просмотра спектакля в театре. Реакцию супруга она
прокомментировала следующим образом: "На сцене играли точно не
[ПРОПУСК], но [ВТОРОЙ ПРОПУСК] была обеспечена". Заполните второй
пропуск двумя словами.

Ответ:
Немая сцена.

Комментарий:
Играли точно не "Ревизора".

Источник:
Журнал "Мой ребенок", декабрь 2005 г.

Автор:
Денис Моргунов (Сургут)

Вопрос 19:
В марте 1975 года боксер Чак Вепнер едва не победил самого Мохаммеда
Али. В 2003 году Вепнер вновь заявил о себе, выступив против знаменитого
итальянца, но уже вне ринга. Ведь во многом благодаря Чаку этот
итальянец и стал знаменит. Назовите фамилию этого итальянца.

Ответ:
Сталлоне.

Комментарий:
Именно этот бой сподвиг Сильвестра Сталлоне на написание сценария фильма
"Рокки". Но Слай Вепнеру так за это и не заплатил до 2003 года.
Прототипом Рокки Бальбоа, кстати, стал именно Чак Вепнер, а не Рокки
Марчиано.

Источник:
http://lenta.ru/sport/2004/10/01/rocky/

Автор:
Аркадий Либерман (Тюмень)

Вопрос 20:
Внимание, в вопросе есть замены.
   Действие фильма происходит в Москве в 1925 году, в дни международного
футбольного турнира. Съезжаются все звезды: Торре, Маршалл, Ейтс и
другие. Город сотрясает футбольная горячка, и главный герой даже
пропускает собственную свадьбу. Лишь вмешательство Пеле спасает пару от
размолвки. Мы не спрашиваем, что мы заменили футболом. Ответьте, какую
фамилию мы заменили на "Пеле".

Ответ:
Капабланка.

Комментарий:
Речь идет о фильме "Шахматная горячка", действие которого происходит во
время Первого Московского международного шахматного турнира. Как
известно, в роли самого себя там снялся Капабланка. Фамилии же других
шахматистов не заменены, что, надеемся, помогло вам ответить на этот
вопрос.

Источник:
http://ru.wikipedia.org/wiki/Шахматная_горячка

Автор:
Аркадий Либерман (Тюмень)

Вопрос 21:
[Ведущему: никак не выделять "Е" в фамилии "Лесток" и тем более не
сообщать, что слово написано с заглавной буквы.]
   В середине XVIII века по воле рока в центре скандала оказался Лесток,
имеющий непосредственное отношение к медицине. В результате некто Иван
Иванович был приговорен к смертной казни, но отделался ссылкой. В каком
фильме этого Ивана Ивановича сыграл Владислав Стржельчик?

Ответ:
"Гардемарины, вперед!".

Комментарий:
Иоганн Лесток - придворный медик, который помимо своих непосредственных
обязанностей еще немного плел интриги. "По воле рока" - намек на самую
известную песню из фильма.

Источник:
http://ru.wikipedia.org/wiki/Лесток,_Иоганн_Германн

Автор:
Аркадий Либерман (Тюмень)

Вопрос 22:
Самый известный в Америке ОН - 6 июня 1944 года, когда началась операция
по высадке войск в Нормандии. В России ОН появился на экранах в прошлом
году. Назовите ЕГО.

Ответ:
День Д.

Комментарий:
В США так традиционно называется день начала военной операции. А в
России фильм с таким названием снял Михаил Пореченков.

Источник:
   1. http://ru.wikipedia.org/wiki/День_Д
   2. http://ru.wikipedia.org/wiki/День_Д_(фильм)

Автор:
???

Вопрос 23:
В пользу того, что это падальщик, говорят большие обонятельные
рецепторы, мощные зубы для дробления кости и вероятная низкая скорость
передвижения. В пользу того, что это хищник, говорят бинокулярное зрение
и следы укусов на других животных. Запишите его название пятью
символами.

Ответ:
T-Rex.

Источник:
http://ru.wikipedia.org/wiki/Тираннозавр

Автор:
Аркадий Либерман (Тюмень)

Вопрос 24:
Тур продолжается, господа!
   Первым ЕГО сыграл Игорь Горбачев. А актер, проживший, по его
признанию, 30 лет ИМ и С НИМ и даже открывший ресторан ЕГО имени, был
лишь четвертым. Всего же известно девять ЕГО экранных воплощений. Какой
актер воплотил этот образ последним?

Ответ:
[Олег] Меньшиков.

Источник:
http://ru.wikipedia.org/wiki/Двенадцать_стульев_(роман)

Автор:
???

Вопрос 25:
Закончите одним словом фрагмент стихотворения-аллюзии Иосифа Бродского
"На смерть Жукова". Рифмовка - "ababab".
   Маршал! поглотит алчная Лета
   эти слова и твои прахорЯ.
   Все же, прими их - жалкая лепта
   родину спасшему, вслух говоря.
   Бей, барабан, и военная флейта,
   громко свисти на манер ...

Ответ:
Снегиря.

Комментарий:
Данное стихотворение - аллюзия на известное стихотворение "Снегирь" -
посвящение Гаврилы Державина на смерть Суворова.

Источник:
Проект Леонида Парфенова "Российская империя", том 2, серия 4.

Автор:
Аркадий Либерман (Тюмень)

Вопрос 26:
Блиц.
   1. X и Y - это то, что совместил в названии француз, написавший о
Жюльене. Найдите X и Y.
   2. Z и Y - это то, что совместил в названии русский, написавший о
непобежденном чемпионе мира по шахматам. Найдите Z и Y.
   3. X и Z - это то, что совместили в названии владельцы сети
алкомаркетов. Найдите X и Z.

Ответ:
   1. Красное и черное.
   2. Белые и черные.
   3. Красное и белое.

Комментарий:
X, Y, Z - красный, черный и белый цвета соответственно. Роман Стендаля -
"Красное и черное" (главный герой - Жюльен Сорель), роман Александра
Котова - "Белые и черные" (главный герой - Александр Алехин), сеть
алкомаркетов - "Красное и белое".

Автор:
Аркадий Либерман (Тюмень)

Вопрос 27:
С одной стороны, ЭТО аэроплан "Флайер-1", или маяк в Портленде, или
персик, или американский бизон, или скрипка, труба и гитара, или
мемориал "Маунт-Рашмор"... С другой стороны, ЭТО - уроженец Вирджинии,
который стал первым из сорока четырех. Назовите псевдоним известного
музыканта, который получится при удвоении ЭТОГО.

Ответ:
50 cent.

Зачет:
Фифти-сент, Фифти.

Комментарий:
На аверсах американских монет в 25 центов изображен первый президент США
Джордж Вашингтон, а реверс украшают различные символы различных штатов
Америки. 50 Cent, он же Fifty, - известный чернокожий рэпер.

Источник:
   1. http://ru.wikipedia.org/wiki/Двадцатипятицентовики_пятидесяти_штатов
   2. http://ru.wikipedia.org/wiki/50_Cent

Автор:
Анна Кутузова (Тюмень)

Вопрос 28:
Этот долговязый джентльмен с пышными бакенбардами, бородкой клинышком, в
брюках в полоску, сюртуке и цилиндре появился на свет благодаря Томасу
Насту. Мы не просим вас назвать его имя. Напишите аббревиатуру, которая
в нем содержится.

Ответ:
USA. Незачет: США.

Комментарий:
В вопросе говорится про дядюшку Сэма, который считается олицетворением
Соединенных Штатов. В его имени - Uncle Sam - содержится аббревиатура
USA.

Источник:
"Вокруг света", 2007, N 9.

Автор:
Анна Кутузова (Тюмень)

Вопрос 29:
[Ведущему: названия читать раздельно и внятно, под запись.]
   У арабов их 28, а у нас обычно выделяют всего 5 - "палец", "краткий с
обеих сторон", "отраженный назад", "плясовой" и еще один, названный,
предположительно, в честь музыкального инструмента. Назовите того, кто
согласно классику, так и не смог разобраться в двух из них.

Ответ:
[Евгений] Онегин.

Комментарий:
Речь идет о стихотворных размерах. "Не мог он ямба от хорея, как мы ни
бились, отличить".

Источник:
   1. http://basik.ru/interesting_facts/literature/
   2. http://ru.wikipedia.org/wiki/Стихотворный_размер
   3. А.С. Пушкин. Евгений Онегин.

Автор:
Дмитрий Журавский (Тюмень)

Вопрос 30:
Внимание, найденное автором вопроса на одном из форумов хокку в
классическом стиле!
   Водка и вода
   Между собой похожи
   ...
   Закончите это хокку незаконченным устойчивым выражением.

Ответ:
Как две капли во...

Зачет:
Как две капли вод...

Источник:
http://www.maccentre.ru/board/viewtopic.php?t=51986&postdays=0&postorder=asc&start=0

Автор:
Дмитрий Журавский (Тюмень)

Тур:
11 тур

Вопрос 1:
В США ОН введен восемнадцатой поправкой к Конституции, а отменен
двадцать первой поправкой. В России ОН действовал с 1914 по 1923 год. 16
мая 1985 года Президиум Верховного Совета СССР издал указ, прозванный в
народе "ПолуОН". Назовите ЕГО двумя словами.

Ответ:
Сухой закон.

Источник:
http://ru.wikipedia.org/wiki/Сухой_закон

Автор:
Анна Рубцова (Вологда)

Вопрос 2:
Редкий автор вопросов ЧГК обходится без них, а вот Уинстон Черчилль без
них прекрасно обходился, научившись этому, по собственному признанию, у
американского дипломата Самнера Уэллса. Воспроизведите заявление
Черчилля, сделанное после встречи с Гарри Трумэном 12 февраля 1946 года.

Ответ:
Без комментариев.

Зачет:
На русском или английском языке в единственном или множественном числе.

Комментарий:
Без комментариев.

Источник:
   1. http://www.gramota.ru/spravka/phrases/?alpha=%C1
   2. http://en.wikipedia.org/wiki/No_comment

Автор:
Дмитрий Журавский, Аркадий Каюгин (Тюмень)

Вопрос 3:
Прослушайте странный рассказ: "Ломоносов получил новое вещество.
Салтыков-Щедрин проявил мягкотелость, и первым применил его в
промышленности Карамзин. Достоевский стушевался, увидев летчика
Хлебникова". По нашему мнению, в рассказе не хватает упоминания
устройства, сконструированного американским инженером Уэксли для
Всемирной выставки в Нью-Йорке в 1927 году, а также уроженца города
Хронова. Назовите фамилию этого человека.

Ответ:
Чапек.

Комментарий:
В рассказе представлены авторы и придуманные им неологизмы, ставшие
частью общей лексики. На всякий случай напоминаем, что слово "робот"
придумал не Карел Чапек (он и родился в другом городе), а его брат
Йозеф.

Источник:
   1. http://basik.ru/interesting_facts/about_the_words/
   2. http://ru.wikipedia.org/wiki/Неологизм
   3. http://ru.wikipedia.org/wiki/Чапек,_Йозеф
   4. http://ru.wikipedia.org/wiki/Робот

Автор:
Аркадий Каюгин (Тюмень)

Вопрос 4:
Внимание, в вопросе словами "ржавые", "воронёные" и "лопаты" заменены
другие слова.
   "РЖАВЫЕ ЛОПАТЫ" - это организация деревенской самообороны в Китае,
возникшая в 1910-х годах. Герой фильма "Трасса 60" считает, что люди не
видят РЖАВЫЕ ЛОПАТЫ, потому что с детства привыкли к ВОРОНЁНЫМ. Какие
однотипные предметы герой фильма использовал для иллюстрации своих слов?

Ответ:
Игральные карты.

Комментарий:
Речь в вопросе идет о красных пиках.

Источник:
   1. БСЭ.
   2. Фильм "Трасса 60".

Автор:
Аркадий Каюгин (Тюмень)

Вопрос 5:
ПЕРВЫЙ - крутой сыщик из американского сериала. ВТОРОЙ - символ
уничтожения и смерти, но в то же время - предвестник новой жизни.
Перевод фамилии ПЕРВОГО вместе со ВТОРЫМ - это то, что одна парочка
долгое время демонстрировала перед Выставкой. Назовите фамилию человека
родом из Риги, которому эта парочка обязана своим появлением на свет.

Ответ:
Мухина.

Комментарий:
Первый - Хаммер (даже не принципиально, какой именно - Майк или Слэдж, -
они оба крутые сыщики, хоть и из разных сериалов), т.е. молот. Второй -
серп. Серп и молот с 1937 года в руках Рабочего и Колхозницы, долгое
время стоявших у одно из входов ВДНХ.

Источник:
   1. http://www.kinopoisk.ru/level/1/film/230598/
   2. http://www.kinopoisk.ru/level/1/film/231082/
   3. http://slovari.yandex.ru/dict/encsym/article/SYM/sym-0634.htm
   4. http://ru.wikipedia.org/wiki/Мухина,_Вера_Игнатьевна

Автор:
Алексей Ламбин (Тобольск)

Вопрос 6:
Сейчас, когда уже не действует приказ Народного комиссариата просвещения
РСФСР от 24 декабря 1942 года, мы можем смело написать, что Великий
князь Михаил Александрович, брат императора Николая II, во время своего
ареста в Перми жил в "королевских номерах". Поэтому, возможно, пришлось
бы пояснять, заодно рассказывая о владельце гостиницы. А тогда, после
приказа, с НЕЙ было бы проще. Запишите ЕЕ.

Ответ:
Ё.

Комментарий:
В 1942 году Народный комиссариат просвещения РСФСР употребление буквы
"ё" сделал обязательным в печатных текстах, однако эта норма
продержалась лишь до начала 1950-х годов. Великий князь Михаил
Александрович был арестован в гостинице, владельцем которой являлся
человек с фамилией Королёв, и номера были "королёвские".

Источник:
Я. Боргер. В защиту буквы Ё. "Дар веков сберечь". - Тюмень: Мандрика,
2008. - С. 71.

Автор:
Игорь Балюнов (Тобольск)

Вопрос 7:
Продолжим тему. На сайте bash.org.ru приводилась такая просьба:
   - Всем привет. Мне нужна песенка очень известная. Топните левой
ногой, затем правой, хлопните в ладоши...
   А какая песня была нужна этому человеку?

Ответ:
"We Will Rock You".

Комментарий:
Вопрос, конечно, простой, но автору очень хотелось посмотреть, как он
будет браться командами. Этим вопросом мы продолжили тему "королевских"
вопросов.

Источник:
http://bash.org.ru/quote/393767

Автор:
Алексей Ламбин (Тобольск)

Вопрос 8:
[Ведущему: в слове "Веве" ударение на втором слоге.]
   2 марта 1978 года с кладбища швейцарского городка Веве двое
безработных похитили тело, потребовав за его возврат 600 тысяч франков.
Переговоры длились около месяца, и преступникам пришлось временно
перезахоронить покойного. Позже владелец поля, где прятали тело, воздвиг
на этом месте дубовый крест, на котором была вырезана тросточка. Чье
тело было похищено?

Ответ:
[Чарли] Чаплина.

Источник:
И. Джохадзе. Криминальная хроника человечества (I-XXI вв.): Истории о
политических и уголовных преступлениях. - М.: Октопус, 2007.

Автор:
Алексей Ламбин (Тобольск)

Вопрос 9:
В одном из эпизодов фильма "Жизнь Дэвида Гейла" начальник тюрьмы
проводит экскурсию по своему заведению. Обратив внимание на японский
садик, он посоветовал не следовать одной широко распространенной
традиции. Что предписывает эта традиция?

Ответ:
Бросать монеты в воду.

Зачет:
По смыслу.

Комментарий:
В садике помимо прочего был и пруд. А кому хочется снова попасть в
тюрьму?

Источник:
В вопросе.

Автор:
Игорь Балюнов (Тобольск)

Вопрос 10:
Эта боевая машина получила часть своего названия - 00 - благодаря
использованию календаря, согласно которому год выпуска - 2600. Какое-то
время эта машина превосходила вражеские, но уже через три года морально
устарела. Однако она применялась до 2605 года (по тому же
летоисчислению), но в последний год этой техникой управляли только...
Кто?

Ответ:
Камикадзе.

Комментарий:
Боевая машина - это самый известный японский истребитель времен Второй
мировой войны A6M Zero.

Источник:
Ф. Функен. Вторая мировая война 1939-1945; США - Япония - Китай - Армии
великих держав в 1943-1945 - Сражающаяся Франция - Дания - Нидерланды -
Балканские страны. - М., 2002.

Автор:
Игорь Балюнов (Тобольск)

Вопрос 11:
Внимание, в вопросе есть замены.
   Обычными концертными костюмами этого советского музыкального
коллектива являлась СПОРТИВНАЯ ОДЕЖДА. Из-за этого артисты не смогли
совершить гастрольное турне по городам США: американское правительство
мотивировало свой отказ тем, что нельзя допускать в страну такое
количество иностранных СПОРТСМЕНОВ. Догадавшись, какие слова мы заменили
словами "спортивная одежда" и "спортсмены", назовите фамилию создателя
этого коллектива.

Ответ:
Александров.

Комментарий:
Словами "спортивная одежда" и "спортсмены" мы заменили слова "военная
форма" и "военные". А коллектив - Дважды краснознаменный ансамбль песни
и пляски Российской армии (в прошлом - Краснознаменный ансамбль
красноармейской песни и пляски СССР).

Источник:
Кто есть кто. Биографический справочник. / Гл. ред. Г.П. Шалаева. - М.,
2007.

Автор:
Игорь Балюнов (Тобольск)

Вопрос 12:
Советским хозяйкам это блюдо нравилось по причине непритязательности и
малых затрат. По одной из версий, впервые его приготовила супруга
Генриха III. По другой версии, его придумал влюбленный повар и посвятил
даме своего сердца. А как звали эту даму?

Ответ:
Шарлотта.

Комментарий:
Шарлотка, по разным версиям, названа в честь Шарлотты - только
неизвестно, какой именно.

Источник:
   1. http://www.kulina.ru/articles/rec/blyuda_iz_testa/zarlotki/
   2. http://ru.wikipedia.org/wiki/Шарлотка

Автор:
Алена Балюнова (Тобольск)

Вопрос 13:
---

Ответ:
---

Вопрос 14:
Эпиграмма Аккермана посвящена барышне определенного телосложения:
   Ты меня спросила: "Что же я такое?
   Для какой я цели Богом создана?".
   Ты для людоеда - сочное жаркое,
   А для астронома - [два слова пропущено].
   Мы не просим вас назвать особенности этой барышни. Назовите два
слова, которые были пропущены.

Ответ:
"... полная луна".

Источник:
Эпиграмма Толстой барышне Ф. Аккермана. // Русские эпиграммы XVIII-XX
вв. - Смоленск, 2006. - С. 336.

Автор:
Алена Балюнова (Тобольск)

Вопрос 15:
Внимание, в вопросе есть замена.
   Чарльз Фолкс описывает так одну сравнительно древнюю процедуру: "В
бочонок укладывали чешую, заливали ее смесью уксуса и песка, а затем
принимались трясти и перекатывать бочонок". Какое слово мы заменили
словом "чешуя"?

Ответ:
Кольчуга.

Комментарий:
Таким способом производилась чистка кольчуги от ржавчины.

Источник:
Чарльз Фолкс. Средневековые доспехи. Мастера оружейного дела. - М.,
2005.

Автор:
Игорь Балюнов (Тобольск)

Вопрос 16:
Вишванатан Ананд, разбивший соперника, сказал: "Во время игры за меня
болели многие шахматисты, а вот за него - никто из его коллег". Также он
признался, что всегда жалел героя некоего фильма, но сейчас этого
чувства у него не осталось. Если верить сериалу, снимаемому как
продолжение фильма, один из "коллег" соперника Ананда мог стать
прародителем этого киногероя. Какую фразу произнес этот киногерой
непосредственно перед тем, как буквально разбил своего врага?

Ответ:
"Аста ла виста, бэби".

Зачет:
В любой допустимой транскрипции или на языке оригинала. :-) Можно без
"бэби".

Комментарий:
Ананд победил шахматный компьютер "Гениус", участвовавший, кстати, в
турнире на общих основаниях. Жалел Виши Терминатора (по всей
вероятности, из фильма "Терминатор-2: Судный день"). А по сериалу
"Хроники Сары Коннор" прародителем терминаторов и всей Скайнет мог стать
шахматный компьютер. А "твердый" Т-800 буквально разбил замерзшего в
жидком азоте Т-1000, выстрелив в него. Разбил, правда, ненадолго...

Источник:
   1. В.Н. Пак. 100 выдающихся шахматистов XX века. - М., 2004. - С.
427.
   2. Х/ф "Терминатор-2".
   3. Т/с "Хроники Сары Коннор".

Автор:
Алена Балюнова (Тобольск)

Вопрос 17:
Согласно правилам внутреннего распорядка воспитательных колоний, ЭТО
предоставляется осужденному сразу после прибытия в исправительное
учреждение. А герой фильма "Король вечеринок" говорил, что ЭТО - всегда
интервью. Назовите ЭТО двумя словами.

Ответ:
Первое свидание.

Комментарий:
На первом свидании надо попытаться узнать о собеседнике побольше - как в
интервью.

Источник:
   1. http://www.prison.org/law/pvk061006.shtml
   2. Фильм "Король вечеринок" (2002), реж. Уолт Беккер.

Автор:
Дмитрий Кукулин (Тюмень)

Вопрос 18:
(pic: 20090830.jpg)
   Перед вами ОНА главы по корпоративным коммуникациям компании "LEGO
Group" Шарлотты Симонсен. Статья о правилах ЕЕ изготовления
заканчивается пожеланием "Пусть ваша ОНА станет козырной!". Назовите ЕЕ
двумя словами.

Ответ:
Визитная карточка.

Комментарий:
Вот такие оригинальные визитки получили топ-менеджеры "Лего". Лицо
человечка делают похожим на владельца визитки, а на спине размещают
телефоны и электронный адрес.

Источник:
   1. http://www.reklamaster.com/articles/id/13889/index.html
   2. http://www.levitas.ru/BD/bd55.htm

Автор:
Дмитрий Кукулин (Тюмень)

Вопрос 19:
Евгений Гришковец вспоминает, что однажды во время прохождения службы
ему удалось уговорить друзей-матросов посетить театр только после того,
как он сказал им, что спектакль будет про представителей смежной
специальности. Напишите название этого спектакля.

Ответ:
"На дне".

Комментарий:
Пришлось сказать, что он про подводников.

Источник:
Е. Гришковец. Год жизни. - М.: Астрель, 2008. - С. 186.

Автор:
Дмитрий Кукулин (Тюмень)

Вопрос 20:
Во время пребывания в Англии Великого посольства русские некоторое время
жили в доме этого адмирала. Позднее тот воспользовался данным фактом,
чтобы поправить свое финансовое положение и получить компенсацию за
убытки, якобы понесенные от русских. А герои известного произведения в
доме, названном его именем, тоже желая получить компенсацию за свои
убытки, получили ЕЕ. Назовите ЕЕ тремя словами.

Ответ:
Карта острова сокровищ.

Зачет:
Любой ответ из трех слов, четко указывающий на карту из "Острова
сокровищ".

Комментарий:
Речь идет об адмирале Джоне Бенбоу и произведении Стивенсона "Остров
сокровищ". Хозяева трактира "Адмирал Бенбоу" Джим Хокинс и его мать,
желая получить деньги за проживание Билли Бонса, прихватили из его
сундука еще и карту.

Источник:
   1. http://ru.wikipedia.org/wiki/Бенбоу,_Джон
   2. Р.Л. Стивенсон. Остров сокровищ. - М.: Современник, 1992. - С. 25.

Автор:
Дмитрий Кукулин (Тюмень)

Вопрос 21:
Персонаж одного фильма, убегая по туннелю от смертельной опасности,
запнулся, упал, СДЕЛАЛ ЭТО и в итоге погиб. А футбольный клуб "Зенит"
СДЕЛАЛ ЭТО в первом же туре российской Премьер-лиги 2008 года. Какие два
слова мы заменили на "СДЕЛАЛ ЭТО"?

Ответ:
Потерял очки.

Источник:
   1. Фильм "Мумия" (1999), реж. Стивен Соммерс.
   2. http://football.sport-express.ru/russia/premier/2008/calendar/

Автор:
Дмитрий Кукулин (Тюмень)

Вопрос 22:
По одной из версий, смена "вертикального" способа на "горизонтальный"
произошла при Людовике XIV, любившем наблюдать за ЭТИМ и старавшемся,
чтобы ему было лучше видно. Википедия дает ЭТОМУ определение, в котором
есть слово "изгнание". Назовите ЭТО.

Ответ:
Роды.

Зачет:
Рождение детей и т.п.

Комментарий:
Роды - естественный физиологический процесс, завершающий беременность и
заключающийся в изгнании плода из матки через канал шейки матки и
влагалище, называемые в этом случае родовыми путями.

Источник:
   1. http://www.9months.ru/press/2_02/32/
   2. http://www.materinstvo.ru/art/1360/
   3. http://ru.wikipedia.org/wiki/Роды_у_человека

Автор:
Любовь Черкашина (Тюмень)

Вопрос 23:
"Вы никак не можете запомнить, в какой последовательности идут байты,
килобайты, мегабайты и т.д.? Вы все время путаете, что больше - терабайт
или гигабайт? Правильный ответ на этот вопрос до сих пор знают далеко не
все". Данный текст использован для описания оригинального подарка,
который мы нашли на сайте одного из интернет-магазинов. Изготовители
обещают, что теперь правильную последовательность названий выучат даже
малые дети. Подарок называется "КиберОНИ". Какое слово мы заменили на
"ОНИ"?

Ответ:
Матрешки.

Комментарий:
В набор входят шесть вложенных одна в другую матрешек: Терабайт,
Гигабайт, Мегабайт, Килобайт, Байт и самая маленькая Бит.

Источник:
http://www.ac-studio.ru/e-shop/_id/662/

Автор:
Дмитрий Кукулин (Тюмень)

Вопрос 24:
В полнометражном мультфильме "Футурама" робот Бендер отправляется в
прошлое, чтобы уничтожить Филиппа Фрая. 8 ноября 2000 года во Флориде
неточным выстрелом Бендер уничтожает корзину с бумагами. Какая фамилия
была написана на этой корзине?

Ответ:
Гор.

Комментарий:
Уничтожив часть избирательных бюллетеней, Бендер тем самым повлиял на
итоги выборов.

Источник:
"Футурама. Большой Куш Бендера".

Автор:
Евгений Ярков (Тюмень)

Вопрос 25:
Спасенный Джеймс Бартли вспоминал светло-розовый крупноребристый купол и
похожие на каучук стены. Описывая этот случай, писатель Джулиан Барнс
сказал, что Бартли - это ОН нашего времени. Назовите его.

Ответ:
Иона.

Комментарий:
Бартли, словно библейского Иону, проглотил кит.

Источник:
Д. Барнс. История мира в 10,5 главах.

Автор:
Евгений Ярков (Тюмень)

Вопрос 26:
Закончите двумя словами стихотворение Александра Петровича-Сырова:
   Играя глазками, весной,
   Меня к любви склоняла дама.
   А я переживал за свой
   Поношенный ...

Ответ:
"... костюм Адама".

Комментарий:
Костюм Адама - отсутствие одежды.

Источник:
http://www.ironicpoetry.ru/autors/15-p/petsirov/igraya_glazkami.html

Автор:
Евгений Ярков (Тюмень)

Вопрос 27:
На территории Западной Европы имела хождение поговорка "Ускользнуть от
Сциллы". А какое устаревшее слово упоминается в русском аналоге этой
поговорки?

Ответ:
Полымя.

Комментарий:
"Ускользнуть от Сциллы" означало попасть к более опасной Харибде.

Источник:
Р. Сабатини. Белларион.

Автор:
Евгений Ярков (Тюмень)

Вопрос 28:
Греческий поэт III века сравнил ИХ с облаками, таЯщими в своем чреве рев
бури. В отечественном произведении один из НИХ также летел по небу.
Какой инсектицид упоминается в этом произведении?

Ответ:
Карбофос.

Комментарий:
Речь идет о слонах. В первом случае - это слоны Ганнибала. Во втором -
полосатый слон из мультфильма про "колобков".

Источник:
   1. А. Ладинский. Слоны Ганнибала.
   2. М/ф "Следствие ведут Колобки. Дело о похищении слона".

Автор:
Евгений Ярков (Тюмень)

Вопрос 29:
В газете "Коммерсантъ" был опубликован алфавитный список налогов,
взимаемых с граждан в разные времена в разных странах. Какое имя
упоминается в рассказе о налоге, расположенном между налогом на трудовую
деятельность и налогом на урожай?

Ответ:
Веспасиан.

Зачет:
Тит Флавий Веспасиан.

Комментарий:
Налог на туалеты.

Источник:
http://www.ippnou.ru/article.php?idarticle=003685

Автор:
Евгений Ярков (Тюмень)

Вопрос 30:
Некоторые из НИХ в XII-XIII веках называли себя на немецкий манер -
волчатами - детьми покровительницы Рима. Назовите тех, кто им
противостоял.

Ответ:
Гибеллины.

Комментарий:
Гвельфы (от нем. Whelf - "волчонок", то есть выкормыш волчицы,
покровительницы Рима) - сторонники папы и итальянской независимости.
Ожесточенное противостояние гвельфов с гибеллинами разделяло всю страну
на протяжении XII-XV вв.

Источник:
   1. http://en.wikipedia.org/wiki/Guelphs_and_Ghibellines
   2. Р. Сабатини. Белларион.

Автор:
Евгений Ярков (Тюмень)

Тур:
12 тур

Вопрос 1:
В одной детской игре необходимо провести мячик между трех ведерок и
забросить в четвертое. Как называется эта игра?

Ответ:
"Колобок".

Комментарий:
Первые предметы обозначают зайца, медведя и волка, от которых Колобок
ушел, а последний предмет - лису.

Источник:
http://ec-dejavu.ru/k/Kolobok.html

Автор:
Евгений Ярков (Тюмень)

Вопрос 2:
В средневековье ОНИ предназначались для того, чтобы комфортно
передвигаться по грязным улицам. Юмористы связывают ИХ с несчастной
женщиной. Что это за женщина?

Ответ:
[Анна] Каренина.

Комментарий:
ОНИ - туфли на платформе.

Источник:
   1. http://www.aands.org/raisedheels/Pictorial/extant.html
   2. http://otvet.mail.ru/question/12703477/

Автор:
Евгений Ярков (Тюмень)

Вопрос 3:
Если вожак собачьей упряжки умел ДЕЛАТЬ ЭТО, за него платили повышенную
цену, так как такое умение могло спасти хозяина в случае падения.
Известен случай, когда некая женщина погибла, СДЕЛАВ ЭТО. Назовите эту
женщину.

Ответ:
Жена Лота. Незачет: Эвридика (там обернулся Орфей).

Комментарий:
СДЕЛАТЬ ЭТО - обернуться. Хороший вожак умел оборачиваться и
возвращаться за упавшим хозяином. Жена Лота превратилась в соляной
столп.

Источник:
   1. В. Пикуль. Богатство.
   2. Ветхий завет.

Автор:
Евгений Ярков (Тюмень)

Вопрос 4:
Закончите тремя древними словами юмористическое двустишие,
опубликованное в Живом Журнале:
   Мы делили апельсин -
   ...

Ответ:
Мене, текел, упарсин.

Комментарий:
Исчислено, взвешено, поделено.

Источник:
http://bablohar-p.livejournal.com

Автор:
Евгений Ярков (Тюмень)

Вопрос 5:
Рядовой Виктор Малышев был подвергнут эксперименту, проводившемуся в
поддержку космической программы. Когда через год эксперимент закончился,
рядовой Малышев вскочил с кровати и сломал себе обе ноги. Тем
удивительнее истории об этом человеке. Назовите этого человека.

Ответ:
Илья Муромец.

Комментарий:
В космосе мышцы атрофируются и вымывается кальций из костей. Малышев
лежал бездвижно целый год. Илья Муромец лежал на печи 33 года, а потом
вскочил и начал свои богатырские похождения.

Источник:
   1. А. Ломачинский. Курьезы военной медицины.
   2. Русские былины.

Автор:
Евгений Ярков (Тюмень)

Вопрос 6:
В произведении Василия Звягинцева отряд Вадима Ляхова, капитана в
возрасте около 35 лет, попадает в параллельный мир, где наталкивается на
многочисленные группы живых мертвецов. Один из этих мертвецов, в отличие
от остальных, идет на контакт и соглашается сопровождать отряд Ляхова. С
каким римским поэтом его сравнивает Ляхов?

Ответ:
Вергилий.

Зачет:
Публий Вергилий Марон.

Комментарий:
Земную жизнь пройдя до половины (35 лет), Данте очутился в сумрачном
лесу. Там он находит Вергилия, готового сопровождать Данте в мир иной.

Источник:
   1. В. Звягинцев. Время живых.
   2. Данте. Божественная комедия.

Автор:
Евгений Ярков (Тюмень)

Вопрос 7:
В 1648 году Флорен Перье поднялся на гору Пюи-де-Дом с неким металлом и
опровергнул утверждение Аристотеля о том, что природа боится пустоты. По
чьей просьбе он это сделал?

Ответ:
[Блез] Паскаль.

Комментарий:
Паскаль сам подняться не смог и попросил своего зятя Флорена Перье,
который и доказал существование атмосферного давления с помощью
запаянных резервуаров и ртути.

Источник:
http://www.karabah88.ru/press2008/08/0830.html

Автор:
Евгений Ярков (Тюмень)

Вопрос 8:
В 1903 году Соединенные Штаты Америки поддержали колумбийских
сепаратистов. Назовите двумя словами то, что стало принадлежать США в
результате этой акции.

Ответ:
Панамский канал.

Комментарий:
Панама тогда принадлежала Колумбии. Америка воспользовалась
разногласиями в государстве и получила себе лакомый кусочек.

Источник:
Вл. Гаков. XX век. Хроника человечества.

Автор:
Евгений Ярков (Тюмень)

Вопрос 9:
В некоторых концлагерях тюремные бирки изготавливались таким образом,
чтобы их можно было переломить в случае смерти заключенного. Герой
произведения Курта Воннегута сравнил такие бирки с ЭТИМ. ЭТО получило
свое название от английского звукоподражательного глагола. Назовите ЭТО.

Ответ:
Крекер.

Комментарий:
Бирка была вся в дырках и походила на крекер. "Крекер" - от англ. to
crack - растрескиваться.

Источник:
   1. http://lib.ru/INOFANT/WONNEGUT/bojnya.txt
   2. http://byaki.net/2007/04/04/zachem_krekeru_dyrki.html

Автор:
Евгений Ярков (Тюмень)

Вопрос 10:
В произведении Сергея Панарина ЭТИМ СЛОВОМ называли дипломированного
доктора. ЭТИМ ЖЕ СЛОВОМ принято называть самых длинных позвоночных в
истории Земли. Назовите это слово.

Ответ:
Диплодок.

Комментарий:
Дипломированный доктор - сокращенно "диплодок". Звероящер диплодок мог
достигать более 28 метров в длину.

Источник:
   1. Сергей Панарин. Всё как у людей.
   2. http://www.zooeco.com/int-dinizavri0-5.html

Автор:
Евгений Ярков (Тюмень)

Вопрос 11:
Армии древнего мира активно пользовались пращами, поэтому при подготовке
поля битвы выделяли специальное время. Аналогичное время упоминается в
библейской книге. Назовите эту книгу.

Ответ:
Экклезиаст.

Комментарий:
Армии выделяли время собирать камни, чтобы лишить вражеских пращников
подручных снарядов. "Время собирать камни и время разбрасывать камни" -
известная цитата из Экклезиаста. "Всему свое время, и время всякой вещи
под небом: время рождаться и время умирать; время разбрасывать камни, и
время собирать камни; время войне, и время миру".

Автор:
Евгений Ярков (Тюмень)

Вопрос 12:
После того как подул легкий ветерок, ОН воскликнул:
   - Машите, машите руками! Если б у вас их было больше, чем у великана
Бриарея, и тогда пришлось бы вам поплатиться!
   Назовите ЕГО.

Ответ:
Дон Кихот.

Комментарий:
Ветер подул, ветряные мельницы пришли в движения. Бриарей - один из
сторуких гекатонхейров. Дон Кихот - известный борец с мельницами.

Источник:
М. Сервантес. Дон Кихот Ламанчский.

Автор:
Евгений Ярков (Тюмень)

Вопрос 13:
В одной шуточной подборке различным произведениям искусств даются новые
названия. Так, "Последний день Помпеи" Брюллова назван "Первым запуском
Большого адронного коллайдера", "Ночь на Днепре" Куинджи стала "Аварией
на ДнепроГЭСе". А какую скульптуру назвали "Прокладкой силового кабеля"?

Ответ:
"Лаокоон".

Комментарий:
Там длинная змея и суровые мужики.

Источник:
http://p-i-f.livejournal.com/98209.html

Автор:
Евгений Ярков (Тюмень)

Вопрос 14:
В "Книге Натаниэля" Данте попадает в место, где Бог хранит свой
инструментарий: вилы, котлы и прочее. Данте обращает внимание на три
затертые буквы. Назовите эти три буквы.

Ответ:
С.К.Л.

Комментарий:
Данте попадает в АД. Его смущает надпись "АД", на которой явно
проступают следы от букв СКЛ, таким образом, Ад - это Склад.

Источник:
Книга Натаниэля.

Автор:
Евгений Ярков (Тюмень)

Вопрос 15:
В произведении Мигеля Сервантеса некое изделие называется кораллами для
"Богородицы". Аналог этого изделия в произведении русского автора
помогал составлять головоломные фразы. А из чего он был изготовлен?

Ответ:
Из нефрита.

Комментарий:
Речь идет о четках. Четки у Акунина помогали усиленно думать и состояли
из 25 нефритовых костяшек.

Источник:
   1. М. Сервантес. Дон Кихот Ламанчский.
   2. Б. Акунин. Нефритовые четки.

Автор:
Евгений Ярков (Тюмень)

Вопрос 16:
Когда упоминали ЕГО фамилию, говорили: "Граждане, обождите радоваться.
Брежнева, Андропова, Черненко еще вспомните". Назовите ЕГО.

Ответ:
[Михаил Сергеевич] Горбачев.

Комментарий:
По первым буквам.

Источник:
http://discoworld.narod.ru/age80/u1-20.html

Автор:
Евгений Ярков (Тюмень)

Вопрос 17:
С Майклом Джексоном это произошло во время выступления в Мэдисон Сквер
Гарден. Аналогичное событие произошло с известным коллежским асессором,
который полагал, что это событие вызвано порчей колдовок-бабок, нанятых
штаб-офицершей Подточиной. Назовите фамилию этого коллежского асессора.

Ответ:
Ковалев.

Комментарий:
Джексон лишился носа, нечаянно задев его рукой. А 25 марта коллежский
асессор (майор) Ковалев проснулся без носа в произведении Гоголя "Нос".

Источник:
   1. http://www.chrab.chel.su/archive/31-10-01/5/A120321.DOC.html
   2. Н.В. Гоголь. Нос.

Автор:
???

Вопрос 18:
Существует версия, по которой ЭТО СЛОВО произошло от того, что самого
последнего ребенка в семье называли просто "другой". Именно поэтому
знаменитого персонажа называли ТАК. Многим из вас наверняка известно,
что если вы - последний оставшийся, то вас назовут ТАК. Как?

Ответ:
Дурак.

Комментарий:
Ребенка в семье называли "другой", "следующий" или "другак" (так же, как
и первак, вторак...), а потом сократили до "дурак". Поэтому младший сын
в сказках - Иван-Дурак. В известной карточной игре дурак тот, кто
последний остался с картами на руках.

Источник:
   1. http://ru.wikipedia.org/wiki/Дурак
   2. http://ru.wikipedia.org/wiki/Дурак_(карточная_игра)

Автор:
???

Вопрос 19:
Согласно легенде, флаг Каталонии возник так: король франков Карл Лысый
использовал желтый щит и то, чего в тот момент было предостаточно. Мы не
спрашиваем вас, что это. Назовите людей, как правило, меняющих ЭТО на
деньги.

Ответ:
Доноры.

Комментарий:
Произошло это во время битвы с маврами в 878 году.

Источник:
Журнал "GEO", 2006, N 11. - С. 74.

Автор:
Евгений Ткаченко (Тюмень)

Вопрос 20:
В песне Александра Васильева "ЭратО" ОНА делает свое дело, а герой поет.
Автору вопроса неизвестно, связано ли это как-то с ЕЕ именем, но песня
поется резко, отрывисто, временами едва не переходя на крик. Какое слово
мы заменили на "ЭратО".

Ответ:
Терпсихора.

Комментарий:
Муза танца. "Терпсихора танцует, а я пою". Описанная манера исполнения -
это намек на корень "псих". ЭратО - тоже муза (любовной поэзии).

Источник:
   1. http://www.pesni.ru/song/3842/
   2. http://ru.wikipedia.org/wiki/Терпсихора
   3. http://ru.wikipedia.org/wiki/Эрато

Автор:
???

Вопрос 21:
Избушка Бабы-Яги могла бы ДЕЛАТЬ ЭТО, но обычно не делала. По мнению
журнала "Вокруг света", группа датских дизайнеров и инженеров изобрела
дом, который ДЕЛАЕТ ЭТО. А кто описал животное, которое ДЕЛАЛО ЭТО?

Ответ:
[Редьярд] Киплинг.

Комментарий:
ДЕЛАТЬ ЭТО - ходить сам(а) по себе.

Источник:
"Вокруг света", 2009, N 1. - С. 30.

Автор:
Наталия Пачкова (Алчевск)

Вопрос 22:
Этот рыцарь родился около 1473 года. Слава отважного и благородного
воина окружала его всю жизнь. О его бесстрашии свидетельствует тот факт,
что он в одиночку защищал мост через реку Гарильяно от 200 испанских
солдат. А следствием отсутствия к нему претензий и упреков со стороны
монарха было жалование ему привилегии собрать именную сотню воинов.
Могила героя находится в Гренобле. Назовите самое известное сооружение,
названное в его честь.

Ответ:
[Форт] Боярд.

Комментарий:
У Баярда было прозвище "рыцарь без страха и упрека".
   z-checkdb: Пьер Террайль де Баярд (Bayard) и форт Боярд (Fort Boyard)
никак не связаны между собой, они даже пишутся по-разному.

Источник:
   1. http://www.banzay.kz/?pg=view&vip=15&razd=13&id=28
   2. http://ru.wikipedia.org/wiki/Баярд,_Пьер_Террайль_де

Автор:
Денис Микшис, Евгений Ткаченко, Евгений Ярков (Тюмень)

Вопрос 23:
По мнению героини Павича, ОНА пахнет тремя вчерашними розами. Художник
угадал это, поэтому смог добиться шедевральной точности изображения. А
кто написал рассказ о мальчике, спасшем ЕЕ?

Ответ:
[Рэй] Брэдбери.

Комментарий:
ОНА - улыбка Моны Лизы. В рассказе Рэя Брэдбери "Улыбка" (входившем в
школьную программу) мальчик спасает кусок картины с улыбкой.

Источник:
   1. М. Павич. Невидимое зеркало. История для девочек. - М., 2004. - С.
59-60.
   2. Р. Брэдбери. Улыбка.
http://www.umniki.ru/any.php?func=show_article&article_id=261&lst=3

Автор:
Наталия Пачкова (Алчевск)

Вопрос 24:
Заполните пропуск в гарике Игоря Губермана двумя словами:
   "Красив, умен, слегка сутул,
   набит мировоззрением,
   вчера [ПРОПУСК] я заглянул
   и вышел с омерзением".

Ответ:
"... в себя...".

Источник:
http://lib.ru/GUBERMAN/gariki1.txt

Автор:
Наталия Пачкова (Алчевск)

Вопрос 25:
Недавно одна из российских компаний вела переговоры с некой бельгийской
деревообрабатывающей фирмой. Бельгийцы, желая произвести впечатление на
российских партнеров, упирали на солидность своей организации и давние
традиции в бизнесе.
   - Мы очень давно в этом деле, - говорит представитель фирмы и
совершенно серьезно добавляет со значением. - Наша фирма поставляла
доски еще для [ПРОПУСК] Людовика XVI.
   Аргумент возымел действие. Договор был заключен. Как-никак, поставщик
королевского двора... Заполните пропуск одним словом.

Ответ:
"... эшафота...".

Источник:
http://askerov.livejournal.com/94758.html

Автор:
Евгений Сибиряк (Луганск)

Вопрос 26:
ИХ прототипами считают так называемые латинские квадраты, исследованные
математиком Леонардом Эйлером в XVIII столетии. В своем современном виде
ОНИ впервые появились в прессе в 1979 году. Сайт "Вокруг света" пишет,
что многие молодые люди узнали об ИХ существовании благодаря развитию
мобильной связи. Мы не спрашиваем вас, о чем идет речь. Скажите, с какой
страны началось повальное увлечение ИМИ.

Ответ:
Япония.

Комментарий:
Речь идет о судоку.

Источник:
http://www.vokrugsveta.ru/quiz/484/

Автор:
Максим Курилов (Луганск)

Вопрос 27:
Закончите одним словом шутливое трехстишие Александра Танского в стиле
японской поэзии:
   Жена лицо намазывает маской.
   Я на нее смотрю, щетину мыля.
   Куда до нас японцам с их ...

Ответ:
"... кабуки".

Комментарий:
Исполнители кабуки используют сложный грим.

Источник:
   1. http://www.stihi.ru/poems/2002/08/22-239.html
   2. http://ru.wikipedia.org/wiki/Кабуки

Автор:
Сергей Скорик (Луганск)

Вопрос 28:
Его жизнь и мировоззрение перевернула книга "Над пропастью во ржи", с
главным героем которой он себя отождествлял, а также статья в журнале
"Эсквайр", разрушившая идеал его кумира, который, представьте себе,
перестал однажды просить дать этому миру еще один шанс и жил в Нью-Йорке
как обычный богач. Мы не спрашиваем вас, кто он. Назовите его кумира.

Ответ:
[Джон] Леннон.

Комментарий:
Речь идет о Марке Чепмене, убийце Джона Леннона.

Источник:
   1. http://ru.wikipedia.org/wiki/Чепмен,_Марк_Дэвид
   2. http://bookz.ru/authors/al_bert-goldman/jzl09/1-jzl09.html

Автор:
Евгений Ткаченко (Тюмень)

Вопрос 29:
Герой фильма "Скафандр и бабочка" перенес инсульт, в результате которого
был практически полностью парализован. Однако он БЫЛ УСЛЫШАН и с помощью
единственного работающего органа смог надиктовать книгу. Название
рецензии на сайте "Коммерсанта" одной буквой отличается от крылатого
выражения, исходящего из Библии. Воспроизведите название рецензии.

Ответ:
"Глаз вопиющего".

Источник:
   1. http://www.kommersant.ru/doc-y.aspx?DocsID=854432
   2. Энциклопедический словарь крылатых слов и выражений. - М.:
Локид-Пресс, 2005. - С. 181.

Автор:
Антон Стыка (Луганск)

Вопрос 30:
Когда сериал "Твин Пикс" показывался в Германии, серии получили названия
с намеками на известные цитаты. Например, "Покойся с болью" - намек на
"Покойся с миром", "Последний вечер" - намек на "Последняя вечеря".
Вспомните о создателе и напишите двумя словами перевод выражения,
аллюзией на которое является название серии "Капризный закон".

Ответ:
Суд Линча.

Комментарий:
"Lynch-Justiz" [Линч-юстИц]. Создатель сериала - Дэвид Линч.

Источник:
http://twinpeaks.cinema.ru/TwinPeaks/faq_tv_m.htm

Автор:
Антон Стыка (Луганск)

Тур:
13 тур

Вопрос 1:
Прослушайте цитату из журнала "КиноДайджест", в которой заменен один из
эпитетов: "Семейная драма Сэма Мендеса "Жизнь сначала" - несладкий
коктейль из старого доброго "кино кухонной раковины", чеховских мотивов,
Теннесси Уильямса и "божественного" дуэта Ди Каприо - Уинслет". А теперь
потрудитесь восстановить оригинальный эпитет.

Ответ:
Титанического.

Комментарий:
В оригинале: "... титанического дуэта Ди Каприо - Уинслет...". Самый
известный экранный дуэт Ди Каприо и Уинслет - в фильме "Титаник". Титаны
- боги в греческой мифологии.

Источник:
Журнал "КиноДайджест", 2009, N 1-2.

Автор:
Сергей Андрюхин (Луганск)

Вопрос 2:
Фалмут Кирни, рожденный в 1830 году в лоне протестантской церкви,
переживший голод и эмигрировавший в США, послужил поводом для написания
книги об ирландских корнях ЭТОГО ЧЕЛОВЕКА. Как, по мнению автора книги,
должна выглядеть фамилия ЭТОГО ЧЕЛОВЕКА?

Ответ:
O'Bama.

Зачет:
О'Бама.

Источник:
http://www.blog-books.ru/articles/barak-obama-na-samom-dele-barak-obama-817.html

Автор:
Антон Стыка (Луганск)

Вопрос 3:
У Бернара Вербера своеобразная манера сопоставлять данные. Писатель
считает, что ЭТОТ ХРАНИТЕЛЬ многолик. Древние египтяне называли его
Анубис, индуисты называли богом мертвых Яма, греки - Хароном, римляне -
Меркурием, а христианам он знаком именно под ЭТИМ именем. Назовите ЕГО.

Ответ:
Святой Петр.

Комментарий:
В христианстве святой Петр "хранит" ключи от небесного царства.

Источник:
Б. Вербер. Империя ангелов.

Автор:
Алла Васильева (Луганск)

Вопрос 4:
Один из персонажей Бернара Вербера получил задание написать сочинение на
свободную тему. Его история называется "Почти папа" и рассказывает о том
времени, когда на конклаве среди кардиналов, претендующих на папский
сан, будет компьютер, в котором заложены все основные принципы
христианства. Такой папа будет иметь массу достоинств - пожизненное
избрание гарантирует практически вечное правление, а если в него
выстрелит какой-нибудь псих, то можно ограничиться ремонтом. К тому же
такой робот-гуманоид постоянно реформирует сам себя. Выбранный робот
получает традиционное христианское имя Пий. Назовите достаточно точно
его порядковый номер.

Ответ:
3,1415926...

Зачет:
3,14.

Источник:
Б. Вербер. Империя ангелов.

Автор:
Алла Васильева (Луганск)

Вопрос 5:
Мать шептала колыбельную, листья в саду опадали, вечерний киносеанс,
негромкая музыка, с утра - контора, перебросился словами, завел
грузовик, загудела улица, сирены, мегаполис, и только ночью - гроза.
Ответьте двумя словами, что описал автор в данном вопросе.

Ответ:
Шкала звука.

Комментарий:
По увеличению децибел.

Источник:
Справочник необходимых знаний. - Пермь, 1995. - С. 18.

Автор:
???

Вопрос 6:
В Древней Руси первый был законом, второй - прощением, третий -
законопреступлением, четвертый - нечестием. О чем идет речь?

Ответ:
О браке.

Комментарий:
Разрешалось не более двух браков.

Источник:
Календарь на 2006 год, 2 января.

Автор:
???

Вопрос 7:
В 1940-е годы в Новосибирске произошел такой случай. Через громадную
очередь, расталкивая всех, в зал влез матерый разбитной мужик. Вслед за
ним - 17-летний парнишка: "Стойте, джентльмен, куда вы лезете?". Мужик
повернулся и грозно спросил: "Это я - джентльмен?! Ты как меня посмел
обзывать?". И здесь добавил некую фразу. Если вы догадались, какие два
образа подарила эта сценка, ответьте, какую фразу добавил грубый
джентльмен.

Ответ:
Ну, погоди!

Источник:
"Российская газета", 14.10.2004 г. - С. 4.

Автор:
???

Вопрос 8:
Если вы обращали на это внимание, то согласитесь с моим наблюдением. Он
в современном виде стал коротким, широким и прямым, как бы вздернутым,
тогда как раньше всегда был длинный и изогнутый. К тому же раньше он
располагался ниже и больше отвечал потребностям содержимого. О чем идет
речь?

Ответ:
Чайный носик.

Комментарий:
В современных чайниках нарушается обычное давление воды в чайнике, и
вода закипает быстро, но неравномерно.

Источник:
В.В. Похлебкин. Занимательная кулинария. - М., 1986. - С. 12.

Автор:
???

Вопрос 9:
Во второй половине XIX века в России на свадьбе, когда дружка приезжал
за невестой, он ударял по сторонам косяков дверей лошадиным кнутом.
Какие слова он при этом произносил?

Ответ:
Во имя Отца, и Сына, и Святаго Духа!

Автор:
???

Вопрос 10:
Как известно, лобань могла быть прямоугольная или с округлым верхом.
Правда, чаще всего встречается так называемая волютообразная лобань, то
есть изображающая в симметрии две женских груди, развернутых к небу.
Между ними может быть вставлен трилистник или другая деталь. Встречаются
лобани с голубками, в форме латинской буквы S и другие. Ответьте
максимально точно, частью чего является лобань.

Ответ:
Наличник.

Комментарий:
Низ называется фартук.

Источник:
С.А. Соловьева. Русские. - 2004. - С. 404-409.

Автор:
???

Вопрос 11:
По мнению одного женского журнала, эта деталь гардероба может
подчеркнуть бедра, грудь или талию в зависимости от того, на уровне чего
она будет находиться. Назовите эту деталь.

Ответ:
Женская сумочка.

Источник:
"Маруся", 2008, N 11.

Автор:
???

Вопрос 12:
Драки на футбольных матчах случаются нередко, но та, что произошла в
аргентинском городе Пелварио, вошла в историю футбола как негативный
курьез. Кто подрался на поле?

Ответ:
Судьи.

Источник:
http://www.footballguru.org/facts/

Автор:
???

Вопрос 13:
Прослушайте цитату из публикации неизвестного автора, опубликованную на
сайте www.dotfun.ru:
   "За обеденным столом профессор Преображенский беседует со своим
помощником доктором Борменталем: Еда, Иван Арнольдович, штука очень
хитрая, ее еще надо уметь достать. Когда вместо пенсии у вас в кармане
только [пропуск]. Да-с-с. И еще, если вы беспокоитесь о своем здоровье,
мой добрый совет - во время еды не говорите о политике и - боже вас
сохрани - не смотрите российское телевидение".
   Заполните пропуск тремя словам.

Ответ:
Вошь на аркане. Незачет: Блоха на цепи.

Комментарий:
Поговорка "Шиш в кармане и вошь на аркане".

Источник:
http://www.dotfun.ru/Krizis-s-umorom/prikoly-pro-krizis-roman-po-m-bulgakovu.html

Автор:
Павел Рогозин (Миасс)

Вопрос 14:
В Берлине на улице с символичным названием Карл-Либкнехт-штрассе
открылся один музей. В этом музее можно посмотреть на автомобиль с
пластмассовым кузовом, посидеть на стульях кинотеатра, побывать в
обстановке квартиры в панельном доме для рабочего класса. Музей
специально открыт для тех немцев, которые испытывают особое чувство, для
которого даже появился неологизм. Этот неологизм всего одной буквой
отличается от обозначения другого чувства. Назовите данный неологизм.

Ответ:
Остальгия.

Комментарий:
Музей был открыт для тех, кто ностальгирует по ГДР. Панельные дома
строили именно в ГДР, автомобиль с пластмассовым кузовом - легендарный
"Трабант". "Ост" - по-немецки "восток".

Источник:
"Вокруг света", 2007, N 1.

Автор:
Артем Кушнир (Тюмень)

Вопрос 15:
По версии историка Альфонсо Энсенат де Виллалонга, этот великий человек
был шотландцем и пиратом. И звали его Педро де Сотто, а вовсе не так,
как называет его всё прогрессивное человечество. Педро родился в Генуе,
в семье лавочников шотландского происхождения де Скотто. "У него были
светлые глаза, веснушки, и он был блондином. Так, по крайней мере, его
описывают современники. Ничего общего с общепринятым образом, который
был придуман гораздо позднее". Историк также утверждает, что Педро в
молодости работал на пирата по имени Винченцо и взял его фамилию для
того, чтобы скрыть характер своих истинных отношений с ним. Назовите эту
фамилию.

Ответ:
Колумб.

Комментарий:
Речь идет, конечно же, о том самом мореплавателе Христофоре Колумбе.

Источник:
   1. http://news.turizm.ru/spain/12996.html
   2. http://osvita.org.ua/news/42783_ru.html
   3. http://www.sunhome.ru/joumal/117294

Автор:
Алла Васильева (Луганск)

Вопрос 16:
Внимание, в вопросе есть замена.
   Елена Исинбаева - известная спортсменка, установившая более 20
рекордов. Когда в 2005 году Елена установила еще один рекорд, некая
газета вышла с заголовком "Пять метров над уровнем моря". Какое слово и
на какое мы заменили?

Ответ:
Мира, моря.

Комментарий:
Заголовок звучал так: "Пять метров над уровнем мира".

Источник:
http://2005.novayagazeta.ru/nomer/2005/58n/n58n-s26.shtml

Автор:
???

Вопрос 17:
Мэрилин Монро, Кейт Уинслет в образе Розы Дьюитт Бьюкейтер, "Блондинка в
законе" Риз Уизерспун, Вивьен Ли в образе Скарлетт О'Хара, Одри Хепберн
в неотъемлемом наряде от "Givenchy" [живаншИ], Элизабет Тейлор в образе
Клеопатры, американская певица Бейонсе, героини "Секса в большом городе"
(Миранда, Шарлотта, Кэрри и Саманта) - всех этих женщин объединяет одно
имя, которое стало популярным во второй половине XX века. Какое?

Ответ:
Барби.

Комментарий:
В этих образах выпускалась кукла Барби.

Источник:
http://olenenyok.livejournal.com/1083612.html

Автор:
???

Вопрос 18:
В этом году любимейшему отечественному мультику исполняется 40 лет. На
нем выросло не одно поколение, при этом мультфильм не получил ни одной
награды. Некоторые особо бдительные в главном герое усматривали прообраз
Элвиса Пресли, а в его единомышленниках - битлов. По еще более
крамольной версии, в отношениях главной героини и ее отца усматривали
прямую пародию. А на кого именно?

Ответ:
На Леонида и Галину Брежневых.

Комментарий:
В те годы дочь генсека уже имела скандальную известность из-за своего
эксцентричного поведения и многочисленных романов. Ее первой любовью и
первым мужем был артист цирка Евгений Милаев. Второй любовью Галины
Брежневой стал Игорь Кио, сын знаменитого иллюзиониста Эмиля Ренарда.
Еще у нее был роман со знаменитым танцором Марисом Лиепой, который был
моложе ее на 11 лет. Уже будучи замужем за милиционером по фамилии
Чурбанов, она завязала отношения с цыганским актером, солистом театра
"Ромэн" Борисом Буряце.

Источник:
http://www.rian.ru/culture/20090202/160735330.html

Автор:
Алла Васильева (Луганск)

Вопрос 19:
В этом напитке, довольно своеобразном, присутствует вещество под
названием туйон (alpha-thujone), отрицательно действующее на мозг.
Вещество это блокирует рецепторы, которые в нормальных условиях тоже
играют роль блокаторов, хотя и частично, - они регулируют потоки ионов,
проходящих через клетки мозга, и тем самым не дают им возбуждаться без
причины. Блокировка блокаторов растормаживает клетки, и они
возбуждаются. Именно поэтому напиток в некоторых странах запрещают.
Некоторые исследователи полагают, что на самом деле название этого
напитка происходит от греческого слова, которое означает "непригодный
для питья". Назовите этот напиток.

Ответ:
Абсент.

Источник:
   1. http://www.progulka.ru/znatoki/200201/24.html
   2. http://ru.wikipedia.org/wiki/Абсент

Автор:
Алла Васильева (Луганск)

Вопрос 20:
Выдержка с сайта Артемия Лебедева.
   Применив в одном из своих параграфов некое математическое действие,
Лебедев пишет, что для ТАКОГО сайта важен длинный и незапоминаемый
адрес. Побольше графики, анимации и рекламы. С ТАКИМ карандашом проще
простого: грифель должен быть перпендикулярен стержню. В ТАКОМ мобильном
телефоне для начала хорошо бы предусмотреть нажатие минимум сорока
кнопок для поиска нужной записи в записной книжке. ТАКОЕ ЭТО известно с
1998 года благодаря Эндрю Дэвису. Что мы заменили словами "ТАКОЕ ЭТО"?

Ответ:
Идеальное убийство.

Комментарий:
Словом "ТАКОЙ" заменено слово "идеальный". Эндрю Дэвис - режиссер фильма
"Идеальное убийство".

Источник:
   1. http://www.artlebedev.ru/kovodstvo/sections/150/
   2. http://www.exler.ru/films/25-09-2003.htm

Автор:
Александр Толстобров (Саранск)

Вопрос 21:
По одной из версий, среди НИХ есть: патриарх, волшебник, актер,
"железнодорожник", преступник, пара хулиганов, пара телепатов, педагог и
многие другие. Мы не просим вас назвать ИХ. Назовите город в Самарской
области, чей телефонный код 84650.

Ответ:
Кошки.

Комментарий:
Перечислены персонажи мюзикла "Cats".

Источник:
   1. http://www.catsmusical.ru/index.php?item=526
   2. http://baza.vgd.ru/3/52691/
   3. http://www.telcode.ru/select-num.asp?num=3320

Автор:
Александр Толстобров (Саранск)

Вопрос 22:
После того как в 1957 году Никита Симонян забил два мяча в ворота одной
из команд, хозяин этой команды захотел его купить. Более известен
случай, когда данный клуб хотел купить другого советского футболиста,
даже известна была сумма сделки, но та сделка также не состоялась.
Правда, в 90-е годы клуб пополнил-таки выходец из СССР. Мы не спрашиваем
вас, что это за клуб. Назовите растение, с которым связывают этот клуб,
хотя в современной символике этого растения уже нет.

Ответ:
Фиалка.

Комментарий:
Речь идет о "Фиорентине".

Источник:
   1. http://ru.wikipedia.org/wiki/Фиорентина
   2. http://www.sovsport.ru/gazeta/article-item/315121

Автор:
Александр Толстобров (Саранск)

Вопрос 23:
Закончите анекдот: "Капля никотина убивает лошадь, а капля никотина с
щепоткой травы делает эту лошадь...".

Ответ:
"... Пегасом".

Источник:
http://www.anekdot.ru/id/371727/

Автор:
Александр Толстобров (Саранск)

Вопрос 24:
---

Ответ:
---

Вопрос 25:
---

Ответ:
---

Вопрос 26:
---

Ответ:
---

Вопрос 27:
---

Ответ:
---

Вопрос 28:
---

Ответ:
---

Вопрос 29:
---

Ответ:
---

Вопрос 30:
Автор необычного монумента решил запечатлеть ее в том виде, в каком она
предстала в альманахе "Аониды" в 1797 году. Сам монумент сначала
поместили рядом с памятником Н.М. Карамзину, что было вполне логично, но
недавно монумент переехал в парк Дружбы народов близ вечного огня,
наверное, с намеком, что та, которой посвящен монумент, будет жить
вечно. Кому или чему возведен этот монумент?

Ответ:
Букве "Ё".

Источник:
"Мир имен и названий", март 2007 г. - С. 6.

Автор:
Александр Толстобров (Саранск)

Тур:
14 тур

Вопрос 1:
В полном титуле этого правителя были слова: "Наследник Норвежский,
герцог Шлезвиг-Голштинский, Сторнмарнский, Дитмарсенский и
Ольденбургский". Назовите максимально точно место убийства последнего из
них.

Ответ:
Подвал Ипатьевского дома в Екатеринбурге.

Комментарий:
Император Всероссийский. Речь идет о Николае II.

Источник:
В.И. Куликов. История государственного управления в России. - М.:
Академия, 2003. - С. 346.

Автор:
???

Вопрос 2:
Этимологический словарь Фасмера сообщает, что это слово появилось в
России при Петре I, к нам оно пришло от французов, к ним - от
итальянцев, а те заимствовали его от арабского слова "дом изготовления".
Из них современному человеку более всего известен лондонский, который в
последнее время стал связан с Россией. Назовите это слово.

Ответ:
Арсенал.

Источник:
Этимологический словарь Фасмера.

Автор:
???

Вопрос 3:
Внимание, литовский язык!
   Саусис, васарис, ковас, баландис, гягуже, биржялис, лепа, ругпютис,
ругсеис, спалис, лапкритис, груодис. В какой русской сказке можно было
бы встретить всех этих персонажей?

Ответ:
"Двенадцать месяцев".

Источник:
Русско-литовско-польский разговорник.

Автор:
???

Вопрос 4:
Во время одной из игр в итальянском футбольном чемпионате вратарь Гроссо
сказал судье, чтобы тот купил очки, за что был немедленно оштрафован и
удален с поля. Но после разбирательства дисциплинарной комиссии
выяснились факты, которые не позволили считать это оскорблением судьи.
Назовите тривиальную причину такого решения дисциплинарной комиссии.

Ответ:
Гроссо - окулист, а судья - близорук.

Зачет:
По упоминанию окулиста и болезни глаз.

Источник:
http://www.footballguru.org/facts/

Автор:
???

Вопрос 5:
В рукописи "Законы шахмат" второй половины XV века из Валенсии
говорится, что ОНИ бывают трех видов: полного истребления, обычный и
вынужденный. Всё почти как в жизни. Правда, историки называют ИХ тезок
отголосками языческих заклинаний. Один собеседник автора сравнивал ИХ со
специями. Коллеге автора все время не достает в работе другого ИХ тезки.
Назовите ИХ.

Ответ:
Маты.

Источник:
"64 - Шахматное обозрение", 1998, N 11.

Автор:
???

Вопрос 6:
Русская пословица рассказывает о ненасытности кармана человека ЭТОЙ
ПРОФЕССИИ. Екатерина II считала недопустимым, чтобы каждый ее подданный
выполнял работу человека ЭТОЙ ПРОФЕССИИ в своих интересах. Литературный
герой приписывал людям ЭТОЙ ПРОФЕССИИ неприятие свободной жизни и другие
недостатки. Назовите ЭТУ ПРОФЕССИЮ.

Ответ:
Судьи.

Комментарий:
Народные пословицы гласят: "Судейский карман - что утиный зоб: и корму
не разбирает, и сытости не знает", "Утиного зоба не накормишь,
судейского кармана не наполнишь". Екатерина II в статье 25 "Грамоты на
права, вольности и преимущества благородного российского дворянства" от
21 апреля 1785 года указала, что "несправедливо ... бы было, когда бы
всяк в собственном деле вздумал сделаться судьею". Из монолога Чацкого:
"А судьи кто? За древностию лет // к свободной жизни их вражда
непримирима. // Сужденья черпают из забытых газет // времен Очаковских и
покоренья Крыма..." и т.д.

Источник:
   1. Русские пословицы и поговорки. / Сост. А.М. Жигунов. - М., 1969. -
С. 358-359.
   2. Российское законодательство в 9 томах. - Т. 5. Законодательство
периода расцвета абсолютизма. / Отв. ред. Е.И. Индова. - М.: Юридическая
литература, 1987. - С. 30.
   3. А.С. Грибоедов. Горе от ума.

Автор:
Денис Микшис (Тюмень)

Вопрос 7:
Внимание, в вопросе есть замена.
   Выступая на церемонии открытия памятника, 82-летняя англичанка
отметила, что в качестве материала для него, "возможно, предпочла бы
КАМЕНЬ". "Однако бронза тоже сойдет - она не заржавеет". Мы не просим
назвать англичанку. Назовите материал, который мы заменили на "камень".

Ответ:
Железо.

Комментарий:
Речь идет о "железной леди" Маргарет Тэтчер.

Источник:
http://www.lenta.ru/news/2007/02/22/bronze/

Автор:
Александр Толстобров (Саранск)

Вопрос 8:
Этот человек был для Солженицына одной из главных и самых дорогих фигур.
Неслучайно именно его он посетил в Овсянке во время возвращения в
Россию. Формулировка премии Солженицына, врученной этому человеку,
звучит так: "[ПРОПУСК] - писателю мирового масштаба, бесстрашному
солдату литературы, искавшему свет и добро в изувеченных судьбах природы
и человека". Назовите этого писателя.

Ответ:
[Виктор Петрович] Астафьев.

Источник:
http://www.rg.ru/2009/03/03/astafev.html

Автор:
Александр Толстобров (Саранск)

Вопрос 9:
Анекдот.
   Родители нарекли сына Емелей. Приходят они в детский сад и спрашивают
у воспитателя, не бьют ли его дети. Воспитатель отвечает: "А кто бить-то
будет - Фома, Лука или Фрол?".
   Каким словом мы заменили слово "бить"?

Ответ:
Дразнить.

Зачет:
По смыслу.

Источник:
http://www.mk.ru/blogs/idmk/2009/02/18/mk-daily/395360/

Автор:
Александр Толстобров (Саранск)

Вопрос 10:
По мнению посетителя гостевой книги ВВ, Репин Илья Ефимович - Торпедон.
Он на своей картине "Бурлаки на Волге" изобразил всех до единого...
Кого?

Ответ:
Болельщиков этого клуба.

Зачет:
По смыслу.

Источник:
http://spartak.msk.ru/gb/2009-03-25_2.html

Автор:
Александр Толстобров (Саранск)

Вопрос 11:
Одна из посетительниц сайта bash.org.ru пишет: "Сегодня препод по
конструкциям предложил взять учебник "Расчет кровельных конструкций",
автор - [ПРОПУСК]. Задумалась впервые, а неспроста он там жил...".
Восстановите пропуск.

Ответ:
Карлсон.

Источник:
http://bash.org.ru/quote/403061

Автор:
Александр Толстобров (Саранск)

Вопрос 12:
ИКС - один из идеальных объектов - имеет форму бутылки не потому, что у
дизайнера вдруг стрельнуло, и он решил сделать горлышко узким, а из-за
дороговизны материала, который нужно экономить. Назовите этот материал.

Ответ:
Пробка.

Комментарий:
ИКС - это бутылка. :-)

Источник:
http://www.artlebedev.ru/kovodstvo/sections/140/

Автор:
Александр Толстобров (Саранск)

Вопрос 13:
Предлагается ввести новый знак, состоящий из двух запятых. Графически и
интонационно такой символ гораздо лучше подходит для обозначения паузы.
Известна судьба типографических нововведений - редкие из них
наслаждаются популярностью. Но, в отличие от малоизвестного сегодня
интерробанга, знак паузы может оказаться востребованным и актуальным в
современных текстах. А что собой представляет знак интерробанг?

Ответ:
Вопроцательный знак.

Зачет:
Гибрид вопросительного и восклицательного знаков.

Источник:
http://www.artlebedev.ru/kovodstvo/sections/133/

Автор:
Александр Толстобров (Саранск)

Вопрос 14:
Известный западный политолог Джеффри Хау в 1985 году написал, что ее
одежда, ее внешний вид, ее умение пользоваться косметикой - это,
несомненно, гарантия того, что Советский Союз очень скоро превратится в
развитую западную страну. По мнению авторов программы "Мой серебряный
шар", этой женщиной восхищались на Западе, но ее недолюбливали в России.
Мы не спрашиваем вас, кто эта женщина. Назовите того, с кем она
познакомилась весной 1951 года на танцах в общежитии МГУ на улице
Стромынка.

Ответ:
[Михаил Сергеевич] Горбачев.

Комментарий:
Речь идет о Раисе Максимовне Горбачевой (Титаренко).

Источник:
   1. "Комсомольская жизнь", 1990, N 17. - С. 2.
   2. http://www.rutv.ru/tvp.html?id=41643&cid=33&d=0

Автор:
Александр Толстобров (Саранск)

Вопрос 15:
Участники одного исторического события 1898 года собрались отметить его
в траттории "У велосипедиста". Примерно четверть из собравшихся получила
"таргетту". Назовите то, что через четверть века заменило "таргетту".

Ответ:
"Скудетто".

Комментарий:
Праздновался первый розыгрыш чемпионата Италии.

Источник:
"Великие клубы": "Фиорентина", "Наполи", "Лацио", 2008, N 12. - С. 2-3.

Автор:
Александр Толстобров (Саранск)

Вопрос 16:
В ежедневной сетке вещания канала "Бибигон" передачи повторяются за день
несколько раз. Исключением служит лишь одна передача, которая выходит
ежедневно в одно и то же время. Назовите эту передачу.

Ответ:
"Спокойной ночи, малыши!".

Комментарий:
Глупо было бы показывать эту передачу утром.
   z-checkdb: В России в 2009 году было 11 часовых поясов. Нет ничего
глупого в том, чтобы в 9 часов утра по Москве показывать эту передачу
для детей Камчатки (Антон Губанов).

Источник:
Программа телеканала "Бибигон".

Автор:
Александр Толстобров (Саранск)

Вопрос 17:
Один из посетителей хоккейной гостевой "Невод" после финальной серии
написал: "Приехал сейчас, значит, после выходных, захожу на кухню. Вижу,
что кончилась заварка. Вспоминаю, что покупал в пятницу пачку. Лезу в
шкафчик, и что я там вижу? Чай "[ПРОПУСК]". Спрашивается, чего два дня с
таким интересом 6-й и 7-й матч смотрел? Намек ведь был ясный". Напишите
абсолютно точно название этого чая.

Ответ:
"АкБарс gold".

Зачет:
"Ак барс голд".

Комментарий:
Аллюзия на золото "Ак Барса" в Кубке Гагарина.

Источник:
Ig_71, гостевая "Невод".

Автор:
Александр Толстобров (Саранск)

Вопрос 18:
Согласно одному афоризму, "Бог создал людей разными, полковник Кольт
сделал их равными...". Назовите уроженца Алтайского края, который,
согласно этому афоризму, сделал людей свободными.

Ответ:
Михаил Калашников.

Источник:
   1. http://board.rt.mipt.ru/?read=2731140
   2. http://ru.wikipedia.org/wiki/Калашников,_Михаил_Тимофеевич
   3. http://www.gfi-udm.ru/ru-ru/rechi_i_vstupleniya/pozdravleniya/detail.htm?itemid=692952

Автор:
Александр Толстобров (Саранск)

Вопрос 19:
Жена президента США Мишель Обама намерена разбить огород на южной
лужайке Белого дома. Один из посетителей сайта http://www.vseseriozno.ru
посоветовал завести курей, пристроечку к Белому Дому, там уточек,
индюшек. Опять же хавронью завести. Зато всегда свежее мясо, зелень к
столу. А там и трактор бы купили, телятник отстроили. Зажили бы, как...
Закончите эту мысль.

Ответ:
Белые люди.

Источник:
http://www.vseseriozno.ru/?cid=3437

Автор:
Александр Толстобров (Саранск)

Вопрос 20:
Одна из главных гоголевских тем, никем покамест не отслеженная, - месть,
мстительность. Она заявлена с самого начала - в самом таинственном и
жестоком из русских триллеров, в прозаической поэме 1832 года "Страшная
месть". История развивается от конца к началу, покамест всё описанное в
ней не начинает выглядеть чудовищной местью одного мертвеца другому, а
[ПРОПУСК] под конец объясняется муками огромного, самого страшного
мертвеца, который шевелится под землей и в бессильной злобе грызет свои
кости. Восстановите пропуск.

Ответ:
Землетрясение.

Источник:
http://www.rulife.ru/index.php?mode=article&artID=1211

Автор:
Александр Толстобров (Саранск)

Вопрос 21:
Полет "Яркой звезды" в течение нескольких дней будоражил мировую
общественность. В одной структуре обсуждались возможная реакция по
поводу "Яркой звезды" и меры по отношению к одному азиатскому городу.
Назовите этот город.

Ответ:
Пхеньян.

Комментарий:
Название северокорейской ракеты "Кван мен сон" в переводе с корейского
означает "Яркая звезда".

Источник:
http://www.svobodanews.ru/content/article/1607141.html

Автор:
Александр Толстобров (Саранск)

Вопрос 22:
В вопросе есть замена.
   Николай аж подскакивает. "Замолчи, - рычит на нее, - я заслуженный
мастер спорта". Действительно, и удостоверение есть. Плюс к пенсии шли
бы шестьсот в буквальном смысле заработанных кровью рублей, если бы
Николай однажды утром собрался с мыслями, восстановил паспорт и съездил
в собес. Какие три слова мы заменили на "заслуженный мастер спорта"?

Ответ:
Почетный донор России.

Источник:
http://www.izvestia.ru/special/article3127043/

Автор:
Александр Толстобров (Саранск)

Вопрос 23:
Личное местоимение первого лица самоубийственно настроенные поэты любят
только в единственном числе. Во множественном они его не жалуют. "Мы",
"нас", "нам" у них встречаются заметно реже, чем у жизнелюбов. Профессор
Пеннебейкер просканировал одного отечественного поэта. С компьютером не
поспоришь. Заявлял, стоял на глыбе, а сам всё "я" да "я". Назовите этого
поэта.

Ответ:
[Владимир] Маяковский.

Комментарий:
Маяковский еще в раннем манифесте, вместе с другими футуристами, заявил,
что стоит "на глыбе слова "мы"".

Источник:
http://www.voanews.com/russian/archive/2002-02/a-2002-02-13-1-1.cfm

Автор:
Александр Толстобров (Саранск)

Вопрос 24:
В ответ на решение от 23 октября 1958 года была послана телеграмма:
"Бесконечно благодарен, тронут, горд, удивлен, смущен". Напишите смысл
телеграммы, отправленной тому же адресату несколько позднее.

Ответ:
Отказ от Нобелевской премии.

Зачет:
По смыслу.

Комментарий:
Речь идет о Пастернаке.

Источник:
http://www.rian.ru/analytics/20090116/159383331.html

Автор:
Александр Толстобров (Саранск)

Вопрос 25:
В фантастическом романе 1960 года, написанном Константином Волковым,
герои - советские ученые - прилетают на некую планету с миссией дружбы и
помощи. Там они находят высокоразвитую гуманоидную расу, с которой
устанавливают сотрудничество. Для исследования планеты местные ученые
присылают к ним своего географа, вернее, ареографа. На какую планету
прилетели герои книги?

Ответ:
На Марс.

Источник:
К.С. Волков. Марс пробуждается. - М.: Советская Россия, 1961. - С. 225.

Автор:
Александр Толстобров (Саранск)

Вопрос 26:
Однажды пятидесятник Богдашка Ленивцев во время обхода дворов
Енисейского острога обнаружил, как один человек занимается тяжким
преступлением. Вот как Ленивцев докладывал об этом: "В той избе из
бумашки пьет енисейский служилый человек Ондрюшка Котлов". А что, по
утверждению Ленивцева, пил из бумажки Ондрюшка Котлов?

Ответ:
Табак.

Зачет:
По смыслу.

Комментарий:
Тогда употреблялось словосочетание "пить табак".

Источник:
А.А. Бродников. Борьба с курением в Сибири в XVII веке.
http://www.zaimka.ru/culture/brodnikov1.shtml

Автор:
Николай Гордиевский (Тюмень)

Вопрос 27:
Гасан Гусейнов описывает один из хитрых приемов запоминания имен
античных героев с помощью телефонной книги, например: Имя ИКС, телефон:
не хочет платить долгов. Назовите того, кого мы заменили на "ИКС".

Ответ:
Стрепсиад.

Источник:
http://www.strana.ru/doc.html?id=120822&cid=7

Автор:
Александр Толстобров (Саранск)

Вопрос 28:
Этот роман вышел в конце 1974 года, в трех номерах журнала "Новый мир".
До выхода роман прошел жесткую цензуру. Вот некоторые замечания
цензоров: "Советский генерал не должен задыхаться, давиться кашлем и
лить слезы. Стыдно это читать. Генерала - выбросить!", "Кто дал право
автору позорить нашу Советскую Дуньку? Выбросить!". Назовите это
произведение.

Ответ:
"В августе сорок четвертого...".

Источник:
http://www.mk.ru/blogs/MK/2008/12/03/society/384287/

Автор:
Александр Толстобров (Саранск)

Вопрос 29:
Известный современный философ Жак Деррида родился в 1930 году в одной из
французских колоний и переехал в Париж уже достаточно сформировавшимся
человеком. С течением времени его работы все больше становятся
охваченными чувством, которое сам философ называл, немного изменяя
хорошо известное переживание. Назовите страну, на территории которой
родился Жак Деррида.

Ответ:
Алжир.

Комментарий:
Чувство - ностальжирия. На языке оригинала ее отличает от ностальгии
всего две буквы.

Источник:
П. Стретерн. Деррида за 90 минут.

Автор:
Максим Мерзляков (Воронеж)

Вопрос 30:
Заголовок одной из статей, посвященных триумфальному выступлению
российских спортсменов на Олимпиаде в Турине, был на две первые буквы
длиннее ставшего не так давно известным политического лозунга. Мы не
просим вас назвать этот лозунг. Назовите вид спорта, которому в статье
было уделено наибольшее внимание.

Ответ:
Фигурное катание.

Зачет:
Мужское одиночное фигурное катание.

Комментарий:
Статья называлась "Плющенко - так!".

Источник:
"Комсомольская правда" от 17.02.2006 г.

Автор:
Максим Мерзляков (Воронеж)

Тур:
15 тур

Вопрос 1:
Внимание, в вопросе есть замены.
   Прослушайте фрагмент стихотворения Николая Гумилева:
   Солнце свирепое, солнце грозящее,
   Бога, в пространствах идущего,
   Лицо сумасшедшее,
   Солнце, сожги ВТОРОЕ
   Во имя ТРЕТЬЕГО,
   Но помилуй ПЕРВОЕ!
   В древнескандинавских гадательных практиках ПЕРВОЕ, ВТОРОЕ и ТРЕТЬЕ
обозначались терминами Урд, Верданди и Скульд соответственно. Назовите в
правильном порядке слова, которые мы заменили на "ПЕРВОЕ", "ВТОРОЕ" и
"ТРЕТЬЕ".

Ответ:
Прошедшее, настоящее, грядущее. Незачет: Прошлое, настоящее, будущее.

Источник:
   1. Г.Л. Олди. Бездна голодных глаз. (Эпиграф к одной из глав.)
   2. Э. Торссон. Северная магия.

Автор:
Максим Мерзляков (Воронеж)

Вопрос 2:
Возможно, ответ на следующий вопрос вас шокирует.
   Габриэль Гарсиа Маркес писал о НЕЙ так: "Никто не сравнится с НЕЙ ни
в пении, ни в танце, ни у кого нет такой невинной чувственности, которая
кажется ЕЕ собственным изобретением...". Назовите ЕЕ.

Ответ:
Шакира.

Зачет:
Шакира Исабель Мебарак Риполль.

Комментарий:
Соотечественница-таки.

Источник:
http://www.myjane.ru/articles/text/?id=2773

Автор:
Максим Мерзляков (Воронеж)

Вопрос 3:
Жозе Сарамаго сравнивал эти два слова с двумя веслами - гребля идет на
лад, лишь когда работаешь обоими. Эти два слова в равной пропорции можно
встретить в русском названии некой игры, английский вариант названия
которой - lateral thinking puzzle [лэтерал синкинг паззл]. Назовите эту
игру.

Ответ:
Данетки.

Источник:
Ж. Сарамаго. Евангелие от Иисуса.

Автор:
Максим Мерзляков (Воронеж)

Вопрос 4:
Согласно одной цитате с bash.org.ru, оптимиста от пессимиста можно
отличить не только с помощью половины стакана воды, но и по
затертости... Чего?

Ответ:
Клавиш 0 и 9.

Зачет:
Клавиш ) и (.

Комментарий:
Смайлики.

Источник:
http://bash.org.ru/quote/397384

Автор:
Сергей Крупник (Воронеж)

Вопрос 5:
После того как московский "Спартак" со счетом 0:3 проиграл первый матч
полуфинала Кубка страны, в ответном матче болельщики требовали от
команды чуда. Это подтверждал и баннер, вывешенный на трибунах.
Воспроизведите слово, написанное на баннере с нарушением орфографии,
причем сделайте в нем те же две ошибки.

Ответ:
4уд0.

Комментарий:
"Спартаку" нужна была победа со счетом 4:0.

Источник:
http://www.sovsport.ru/gazeta/default.asp?p=2&date=2007/5/10&id=258713

Автор:
Максим Мерзляков (Воронеж)

Вопрос 6:
После того как биатлонист Уле-Эйнар Бьорндален предложил разрешить
антидопинговой комиссии проводить обыски в домах у спортсменов,
подозреваемых в употреблении допинга, комментатор телеканала "Спорт"
предположил наличие у норвежца русских корней. Назовите имя и фамилию
человека, в родстве с которым комментатор заподозрил Бьорндалена.

Ответ:
Павлик Морозов.

Комментарий:
Сдал своих. :-)

Источник:
Трансляция с Кубка мира по биатлону на телеканале "Спорт".

Автор:
Максим Мерзляков (Воронеж)

Вопрос 7:
В статье, посвященной Чемпионату Европы по футболу - 2008, говорится,
что СДЕЛАТЬ ЭТО могут только обладатели Шенгенской визы. Для того чтобы
легче было СДЕЛАТЬ ЭТО, в последнее время всё чаще используют ультразвук
или инфракрасный свет. Ответьте тремя словами, что мы заменили словами
"СДЕЛАТЬ ЭТО".

Ответ:
Попасть в вену.

Комментарий:
В статье речь шла о билетах в столицу Австрии. Для облегчения поиска вен
при уколе сейчас используются ультразвуковые либо инфракрасные датчики.

Источник:
   1. http://biznews.hup.ru/2008/06/24/болельщикам-все-сложнее-попасть-в-вен/
   2. http://www.mosmedclinic.ru/news/933
   3. http://www.trimm.ru/php/content.php?group=0&id=1752

Автор:
Максим Мерзляков (Воронеж)

Вопрос 8:
Фунт лиха хлебнул русский нелегал-разведчик, продемонстрировав в
западноевропейском бассейне необщность российского аршина. А каким
образом?

Ответ:
Поплыл "саженками".

Зачет:
По смыслу.

Источник:
Журнал "Esquire", февраль 2009 г. - С. 68.

Автор:
Сергей Финоченко (Воронеж)

Вопрос 9:
С Бенедиктом XVI в Басре, с Джорджем Бушем-младшим в Бишкеке, с Ксенией
Собчак в Новосибирске, с Веркой Сердючкой в Киеве ЭТО произошло как
минимум однажды. Назовите человека, с которым ЭТО происходит во многих
городах ежегодно в начале ноября.

Ответ:
Гай Фокс.

Комментарий:
Перечисленные персонажи, вернее, их чучела, принародно сжигались. "Ночь
Гая Фокса" до сих пор отмечается во многих англоязычных странах 5
ноября.

Источник:
   1. http://www.grani.ru/Society/m.111602.html
   2. http://www.varorud.org/index.php?option=com_content&task=view&id=11281&Itemid=102
   3. http://www.amic.ru/news/?news_id=61557
   4. http://www.utro.ru/news/2007/03/07/630978.shtml
   5. http://ru.wikipedia.org/wiki/Фокс,_Гай

Автор:
Сергей Финоченко (Воронеж)

Вопрос 10:
В книге Бориса Акунина "Пелагия и черный монах" есть персонаж -
сумасшедший физик. Прослушайте один из его монологов: "Я обнаружил лучи.
Проходят через всё. В точности как писал Тото. Про опыт с
фотопластинкой. И Маша писала". Мы не спрашиваем вас фамилию Маши.
Назовите фамилию Тото.

Ответ:
Беккерель.

Источник:
Б. Акунин. Пелагия и черный монах. - М.: OOO "Издательство АСТ", 2002. -
С. 330.

Автор:
Евгений Ткаченко (Тюмень)

Вопрос 11:
Героиня одного романа, приехавшая в Нью-Йорк из Лондона, говорила:
"Больше всего мне нравятся их деньги, я никак не могу привыкнуть, что
они настоящие, и мне кажется, что я...". Закончите ее фразу тремя
словами.

Ответ:
"... играю в "Монополию"".

Источник:
Журнал "XXL", февраль 2008 г. - С. 65.

Автор:
Сергей Финоченко (Воронеж)

Вопрос 12:
В качестве материала для ИКСОВ использовались благородные металлы,
кости, перья, иглы дикобраза. Промышленное производство ИКСОВ началось с
усовершенствования машины по изготовлению обувных деревянных гвоздиков,
которые в России назывались спичками. Назовите ИКСЫ.

Ответ:
Зубочистки.

Комментарий:
А некоторые и сейчас в зубах спичками ковыряются. :-)

Источник:
http://www.nkj.ru/archive/articles/14327/

Автор:
Сергей Финоченко (Воронеж)

Вопрос 13:
"Над нашей вискокурней постоянно роятся ангелы... А мы ежегодно жертвуем
им два [ПРОПУСК]" - так говорит Колин Скот, изобретатель виски "Chivas
Regal 18". Восстановите пропуск.

Ответ:
"... градуса".

Зачет:
"... процента".

Комментарий:
Изначально в бочки для выдержки виски заливается жидкость крепостью до
80°.

Источник:
http://www.rusrep.ru/2009/06/news_viski/

Автор:
Сергей Финоченко (Воронеж)

Вопрос 14:
Внимание, в вопросе есть замена.
   Шутливый комментарий к фотографии в журнале "Maxim" гласил: "На этом
снимке отчетливо видно, как СЫН курит, а папа его наказывает". Мы не
спрашиваем, какому явлению посвящена статья с этой фотографией. Назовите
город, где она была сделана.

Ответ:
Рио-де-Жанейро.

Комментарий:
Молния попала в голову статуи Иисуса Христа в Рио.

Источник:
Журнал "Maxim", июль 2008 г.

Автор:
Сергей Финоченко (Воронеж)


FreeBSD-CVSweb <freebsd-cvsweb@FreeBSD.org>